You are on page 1of 99

WE ARE PROVIDING STUDY MATERIAL FOR INTERNAL MEDICINE

MOH PROMETRIC HAAD DHA SLE QCHP FOR DUABI OMAN SAUDI
QATAR UAE MEDICAL LICENCING EXAMS .STUDY MATERIALS ARE

0
PREPAIRED ON THE BASE OF EXAM SYLLABUS .

1
59
FULL COURSE STUDY MATERIAL INCLUDE 6000 MCQ + CLINICAL

65
VIVA PREPARTION NOTESAND CASE STUDIES

48
CONTACT US : WHATSAP 00918848655910 OMAN 00968 98549513

88
EMAIL : stalin.neel@gmail.com

91
00
INTERNAL MEDICINE PRACTICE QUESTIONS

AP
This free study guide includes: TS
HA
 50 questions to help you prepare for the ABIM Exam
W

 Combination of clinical vignettes and concept review


AL

 Detailed text explanations with visual aids for each question


I
ER
AT
M

www.knowmedge.com
SE
UR
CO
LL
FU

ABIM is a registered trademark of the American Board of Internal Medicine, which neither sponsors nor is affiliated in any way with this product.
1 0
59
65
48
Table of Contents

88
91
Cardiovascular Disease .................................................................... 1

00
Endocrinology & Metabolism ......................................................... 21
Gastroenterology ..........................................................................40

AP
General Internal Medicine .............................................................. 56

TS
Hematology................................................................................... 72
HA
Infectious Disease ..........................................................................88
W

Nephrology/Urology .................................................................... 106


Oncology ..................................................................................... 123
AL

Neurology ................................................................................... 133


I
ER

Dermatology ............................................................................... 143


AT

Pulmonary Disease & Critical Care ............................................... 153


M

Rheumatology / Orthopedics ....................................................... 169


SE
UR
CO
LL
FU
Chapter
K N O W M E D G E

0
1

1
59
65
48
88
91
00
AP
TS
Cardiovascular Disease HA
W
ALI
ER
AT
M
SE
UR
CO
LL
FU

1|Page
K N O W M E D G E

1 0
Question #1

59
Topic: Cardiovascular Disease

65
Subtopic: Hypertension

48
40-year old male with no past medical history presents to the office complaining of fatigue he has been feeling

88
for the past 8 months. He denies any shortness of breath or lightheadedness. He reports mainly that he feels

91
very tired during the day time and feels like he needs to take a nap. He also noted recently that he nearly falls
asleep when driving to work. He denies any vision changes or chest pain. On examination, his heart rate is

00
84/min, blood pressure is 156/60mmHg, and BMI is 33.7kg/m2. Head and neck exam shows unremarkable

AP
fundoscopic exam and increased neck circumference. Cardiac examination shows regular rate and rhythm,
normal S1 and S2, and no gallops or murmurs. Lung exam is clear to auscultation. Abdominal examination

TS
shows obese abdomen, but otherwise unremarkable. Extremities are without edema. Which of the following is
the most appropriate next step?
HA
A. Obtain an echocardiogram
W

B. Start hydrochlorothiazide
AL

C. Perform nocturnal pulse oximetry


I
ER

D. Start metoprolol
AT

E. Obtain an electrocardiogram
M
SE
UR
CO
LL
FU

2|Page
K N O W M E D G E

1 0
Answer #1

59
Topic: Cardiovascular Disease

65
Subtopic: Hypertension

48
Correct Answer: C (Perform nocturnal pulse oximetry)

88
This patient has symptoms (daytime hypersomnolence) and risk factors (obesity, increased neck circumference)

91
for obstructive sleep apnea. Sleep apnea is an important reversible cause of hypertension. Therefore it is
important to evaluate and treat this patient for sleep apnea prior to initiating anti-hypertensive therapies. If

00
treatment for sleep apnea does not result in lower the blood pressure, the next step would be to begin

AP
medications for blood pressure reduction.

TS
Let’s go over the answer choices:
HA
 Choice A (Echocardiogram) is sometimes helpful in the management of hypertensive patients to assess
for left ventricular hypertrophy, but would not be the next step for this patient.
W

 Choice B (Hydrochlorothiazide) is an appropriate first-line agent for essential hypertension, but it is


AL

more important to evaluate and treat this patient for a reversible etiology of his hypertension prior to
I
ER

starting therapy.
AT

 Choice C (Nocturnal pulse oximetry) is the correct choice for screening this patient for obstructive
sleep apnea.
M

 Choice D (Metoprolol) is no longer recommended as a first-line agent for essential hypertension unless
SE

there are certain comorbidities such as prior myocardial infarction or systolic congestive heart failure.
UR

 Choice E (Electrocardiogram) would not be helpful in the initial management of this patient.
CO
LL
FU

3|Page
K N O W M E D G E

01
Topic: Cardiovascular Disease

59
Subtopic: Hypertension

65
48
88
91
00
AP
TS
HA
W
ALI
ER
AT
M
SE
UR
CO
LL
FU

4|Page
K N O W M E D G E

1 0
Question #2

59
Topic: Cardiovascular Disease

65
Subtopic: Ischemic Heart Disease

48
88
91
00
AP
TS
HA
W
I AL

76-year-old male with hypertension, dyslipidemia, and osteoarthritis of the knees, is admitted for
ER

evaluation of chest pain. The chest pain is intermittent, occasionally occurring at rest and not worsened
AT

by exertion. He is pain free on arrival to the emergency room. The patient’s home medications include
aspirin 81mg daily and lisinopril 10mg daily. His vital signs show blood pressure 154/76mmHg and heart
M

rate 76/min. Physical examination is unremarkable. His admitting electrocardiogram is shown above, and
SE

is unchanged from prior EKGs from one, three, and four years. He has had no further symptoms during
the hospital stay. Which of the following is the most appropriate next step in management?
UR
CO

A. Exercise stress test without imaging


B. Exercise stress test with nuclear imaging
C. Pharmacologic stress test with nuclear imaging
LL

D. Exercise stress with echocardiography


FU

E. Cardiology consultation for catheterization

5|Page
K N O W M E D G E

1 0
Answer #2

59
Topic: Cardiovascular Disease

65
Subtopic: Ischemic Heart Disease

48
Correct Answer: Choice B (Aspirin with colchicine)

88
This is a patient admitted with atypical chest pain who undergoes a “rule-out myocardial infarction”

91
protocol. The next step to risk stratify this patient is a stress test. His baseline EKG shows a left-bundle
branch block (LBBB) that is chronic (present on prior ECGs). In general, exercise stress testing of any

00
sort should be avoided if the goal is to diagnose ischemia in a patient with a pre-existing LBBB.

AP
Let's go over the answer choices:

TS
 Choice A (Exercise stress test without imaging) is not correct. A patient must have an
HA
“interpretable” baseline EKG to be able to detect ischemia when the patient exercises. If a patient
has a pre-existing LBBB, then the EKG cannot be used to detect ischemia that develops when a
W

patient exercises. Thus, an EKG stress test is not sufficient in a patient with LBBB, and some
AL

form of imaging must be performed.


I
ER

 Choice B (Exercise stress test with nuclear imaging) and Choice D (Exercise stress with
echocardiography) are not correct. As stated in Choice A, in general, performing exercise stress
AT

test in patients with LBBB is not preferred. Exercising imaging tests in patients with LBBB can
produce false-positive test results because the LBBB causes artifacts with both nuclear images and
M

echocardiograms when done with exercise testing.


SE

 Choice C (Pharmacologic stress test with nuclear imaging) is the correct answer. It is the only
UR

stress test among the answer choices that does not involve exercise. Pharmacologic stress tests are
always done with imaging.
CO

 Choice E (Cardiology consultation for catheterization) is not correct because this patient has
LL

atypical chest pain without any objective evidence of ischemia.


FU

6|Page
K N O W M E D G E

01
Topic: Endocrinology & Metabolism

59
Subtopic: Thyroid Disorders

65
48
88
91
00
AP
TS
HA
W
I AL
ER
AT
M
SE
UR
CO
LL
FU

28 | P a g e
K N O W M E D G E

1 0
Question #9

59
Topic: Endocrinology & Metabolism

65
Subtopic: Diabetes Mellitus

48
35-year-old male presents to your office for a work-related physical. He says he is in good health and voices no

88
complaints at this time. He has no past medical history or surgical history present. Family history is positive for

91
diabetes on his mother’s side. He denies smoking, drinks alcohol socially, and denies illegal drugs.

00
His vital signs include blood pressure of 124/80mmHg, pulse 75/min, temperature 36.8°C, respiratory rate
20/min. Body mass index is 32.0kg/m2. Physical exam is otherwise unremarkable. He gets lab work done and

AP
comes back a week later to discuss the results. His LDL is 101mg/dL, HDL 42mg/dL, total cholesterol and

TS
triglycerides normal. Complete blood count (CBC), renal function and liver function tests are also normal. His
blood glucose on the labs comes back at 108mg/dL. He says he is fasting today. You obtain finger stick glucose
HA
in the office today that shows a glucose level of 110mg/dL.
W

Which of the following is the next best step for this patient?
AL

A. Start metformin
I
ER

B. Start glipizide

C. Start rosiglitazone
AT

D. Start basal insulin


M

E. Encourage lifestyle modifications


SE
UR
CO
LL
FU

29 | P a g e
K N O W M E D G E

1 0
Answer #9

59
Topic: Endocrinology & Metabolism

65
Subtopic: Diabetes Mellitus

48
Correct Answer: Choice E (Encourage lifestyle modifications)

88
Prediabetes is defined as two or more fasting blood sugar values of 100-125mg/dL or a blood sugar of 140-

91
199mg/dL two hours after a 75g glucose load. Diabetes can be diagnosed in patients who have:

00
 Two fasting blood glucose greater than or equal to 126mg/dL OR
 Blood sugar greater than or equal to 200mg/dL 2 hours after a 75g load of oral glucose OR

AP
 A random blood glucose greater than or equal to 200mg/dL in the context of polyuria, polydipsia, or

TS
unintentional weight loss
HA
This patient has prediabetes at this time since both his fasting sugars have been between 100-125mg/dL. The
best recommendation to convey to this patient is diet and exercise as he has a high body mass index, making
W

Choice E (Encourage lifestyle modifications) the best answer. Additionally, he should be told that if he doesn’t
AL

control his prediabetes now, this can lead to diabetes mellitus and will likely require medical treatment in the
future.
I
ER
AT
M
SE
UR
CO
LL
FU

30 | P a g e
K N O W M E D G E

1 0
Question #15

59
Topic: Gastroenterology

65
Subtopic: Small Intestinal Disease

48
23-year-old female goes to medical school in the Caribbean islands to complete her basic science training. Due

88
to her hectic study schedule, she is always eating out at fast food restaurants. After she completes her basic

91
science training that lasted 18 months, she comes back to the United States for her clinical rotations. For a few
months, she complains of diarrhea, abdominal pain and seven pound weight loss. Stool studies are all negative.

00
The patient has mild anemia and increased mean corpuscular volume (MCV). Which of the following would

AP
you expect to find in this patient?

TS
A. Increased folate, decreased vitamin B12

B. Decreased folate, normal vitamin B12


HA
C. Normal folate, decreased vitamin B12
W

D. Decreased folate, decreased vitamin B12


AL

E. None of the above


I
ER
AT
M
SE
UR
CO
LL
FU

50 | P a g e
K N O W M E D G E

1 0
Answer #27

59
Topic: Infectious Disease

65
Subtopic: STDs

48
Correct Answer: Choice E (Secondary syphilis is associated with “nickel and dime” lesions on palms and soles)

88
Stages of syphilis:

91
 Primary syphilis is associated with a painless ulcer that is called a chancre and is diagnosed with

00
darkfield microscopy.

AP
 Secondary syphilis is associated with lesions on the palms and soles that look like nickels and dimes.

TS
 Tertiary syphilis (also known as neurosyphilis) is associated with aortitis and neurological
manifestations.
HA
Choice E (Secondary syphilis is associated with “nickel and dime” lesions on palms and soles) is the correct
W

answer.
AL

Let's go over the other answer choices:


I
ER

 Choice A (Secondary syphilis is associated with foot drop and wide based gait): This is describing tabes
AT

dorsalis, a condition where patient has foot slapping and wide based gait. Tabes dorsalis is actually a
part of neurosyphilis, which makes this choice incorrect.
M
SE

 Choice B (VDRL and RPR are specific for syphilis): MHA-TP and FTA-ABS are specific tests for
syphilis while VDRL and RPR are not so Choice B is incorrect.
UR

 Choice C (Tertiary syphilis is associated with pupils being able to constrict to light): Argyll Robinson
CO

pupil is one that is able to constrict to accommodation and not to light. Choice C is saying that it is able
to constrict to light in tertiary syphilis which is the wrong answer.
LL

 Choice D (Female patient who is pregnant and allergic to penicillin should receive ceftriaxone) : The
FU

best approach to treating a pregnant woman who is allergic to PCN is to first desensitize her and then

97 | P a g e
K N O W M E D G E

1 0
give her PCN, making Choice D incorrect also. In non-pregnant patients with syphilis, PCN

59
desensitization and administration is also the preferred treatment but doxycycline can be given instead.

65
48
88
91
00
AP
TS
HA
W
I AL
ER
AT
M
SE
UR
CO
LL
FU

98 | P a g e
K N O W M E D G E

0
1
Topic: Dermatology

59
65
48
88
91
00
AP
TS
HA
W
ALI
ER
AT
M
SE
UR
CO
LL
FU

146 | P a g e
K N O W M E D G E

0
1
Question #42

59
Topic: Dermatology

65
48
88
91
00
AP
TS
HA
Source: CDC/Emory Univ.; Dr. Sellers
W

Which of the following conditions applies to the above image?


AL

A. Oral hairy leukoplakia


I
ER

B. Lichen planus
AT

C. Geographic tongue
M

D. Black hairy tongue

E. Candida of tongue
SE
UR
CO
LL
FU

147 | P a g e
K N O W M E D G E

1 0
Answer #49

59
Topic: Rheumatology / Orthopedics

65
Subtopic: Crystal-Induced Arthropathy

48
Correct Answer: Choice B (Decreased complement level, increased anti-dsDNA level, increased ESR level)

88
When a patient presents with acute pain located in one joint, you should consider gout, pseudogout, septic

91
arthritis, or trauma.

00
Arthrocentesis values to remember are the following:

AP
 Less than 200/μL WBCs: are normal

TS
 200/μl - 2,000/μL WBCs: suggests a non-inflammatory process, such as osteoarthritis, degenerative
joint disease
HA
 5,000/μL - 50,000/μL WBCs: corresponds to an inflammatory process
W
AL

 Greater than 50,000/μL WBCs: suggests a septic joint


I
ER

Gout and pseudogout are both inflammatory monoarticular arthropathies. This means that arthrocentesis
would yield between 5,000/μL - 50,000/μL WBCs so choice E (WBCs of 6,000/μL) is the correct answer.
AT

Choice A (Positive birefringence) and Choice B (Rhomboid crystals) are seen with pseudogout, not gout.
M

Gout would have needle shaped crystals and negative birefringence. Indomethacin or colchicine are
SE

recommended for acute attack. Allopurinol should never be started in an acute attack and never be
discontinued if the patient is on it already. Since the patient is having an acute attack, we don’t want to start
UR

allopurinol making Choice D (Start indomethacin and allopurinol) incorrect.


CO
LL
FU

174 | P a g e
0
I. INTRODUCTION TO CLINICAL MEDICINE

91
55
QUESTIONS

6
48
88
DIRECTIONS: Choose the one best response to each question.

91
00
I-1. A physician is deciding whether to use a new test to I-5. (Continued)
screen for disease X in his practice. The prevalence of dis- nesses. She reports occasional intermittent chest pain that
ease X is 5%. The sensitivity of the test is 85%, and the is unrelated to exercise but is related to eating spicy food.

AP
specificity is 75%. In a population of 1000, how many pa- The physician’s pretest probability for coronary artery
tients will have the diagnosis of disease X missed by this disease causing these symptoms is low; however, the pa-
test? tient is referred for an exercise treadmill test, which shows
A. 50 TSST depression after moderate exercise. Using Bayes’ theo-
rem, how does one interpret these test results?
HA
B. 42
C. 8 A. The pretest probability is low, and the sensitivity
D. 4 and specificity of exercise treadmill testing in fe-
W

males are poor; therefore, the exercise treadmill


I-2. How many patients will be erroneously told they have test is not helpful in clinical decision making in
diagnosis X on the basis of the results of this test?
AL

this case.
A. 713 B. Regardless of the pretest probability, the abnormal
result of this exercise treadmill testing requires fur-
I

B. 505
ER

C. 237 ther evaluation.


D. 42 C. Because the pretest probability for coronary artery
disease is low, the patient should be referred for fur-
AT

I-3. Which type of health care delivery system encourages ther testing to rule out this diagnosis.
physicians to see more patients but to provide fewer D. Because the pretest probability was low in this case,
M

services? a diagnostic test with a low sensitivity and specificity


is sufficient to rule out the diagnosis of coronary ar-
A. Capitation
E

tery disease.
B. Fee-for-service
E. The testing results suggest that the patient has a very
S

C. Fixed salary compensation


high likelihood of having coronary artery disease
UR

D. Out-of-pocket
and should undergo cardiac catheterization.
I-4. The curve that graphically represents the family of cut-
CO

off points for a positive vs. negative test is a receiver oper- I-6. An effective way to measure the accuracy of a diag-
ating characteristic (ROC) curve. The area under this nostic test is a positive likelihood ratio [sensitivity/(1 –
curve is a quantitative measure of the information con- specificity)], which is also defined as the ratio of the
tent of a test. The ROC axes are probability of a positive test result in a patient with dis-
LL

ease to the probability of a positive test result in a patient


A. negative predictive value vs. (1 – positive predictive without disease. What other piece of information is
FU

value) needed along with a positive likelihood ratio to estimate


B. positive predictive value vs. (1 – negative predictive the possibility of a given disease in a certain patient with a
value) positive test result?
C. sensitivity vs. (1 – specificity)
D. specificity vs. (1 – sensitivity) A. Disease prevalence in the patient’s geographic region
B. Negative predictive value of the test
I-5. A patient is seen in the clinic for evaluation of chest C. Positive predictive value of the test
pain. The patient is 35 years old and has no medical ill- D. Pretest probability of the disease in a patient

1
Copyright © 2008, 2005, 2001, 1998, 1994, 1991, 1987 by The McGraw-Hill Companies, Inc.
Click here for terms of use.
14 I. INTRODUCTION TO CLINICAL MEDICINE — QUESTIONS

I-104. All of the following statements about Gaucher dis- I-107. (Continued)
ease are true except clear cells, 15% lymphocytes, 3% eosinophils, and 2%
monocytes. The peripheral smear is normal. The hemato-

0
A. Bone pain is common.
crit is 34.7%. His erythrocyte sedimentation rate (ESR) is

91
B. Disease frequency is highest in Ashkenazi Jews.
elevated at 57 mm/h. Liver and kidney function are nor-
C. Inheritance is autosomal recessive.
mal. HIV, Epstein-Barr virus (EBV), and cytomegalovirus

55
D. Splenomegaly is rare.
(CMV) testing are negative. Routine blood cultures for
E. The disease is caused by mutations in the gene for
bacteria, chest radiograph, and purified protein derivative

6
acid β-glucosidase.
(PPD) testing are negative. In large groups of patients sim-

48
I-105. The following pedigree is an example of what pat- ilar to this one with fever of unknown origin, which of the
tern of inheritance? following categories comprises the largest group of diag-

88
noses if one is able to be determined?

91
A. Drug or other ingestion
B. Hereditary periodic fever syndromes, such as famil-
ial Mediterranean fever

00
C. Infection
D. Neoplasm

AP
E. Noninfectious inflammatory disease

I-108. Chronic hypoxia causes biochemical changes whereby

TS
oxygen delivery to tissues is not impaired. In comparison
to someone living at sea level, which of the following
HA
changes would be expected in a healthy person acclimated
Solid figure Affected individual
to living at high altitude?
Open figure Unaffected individuals
W

FIGURE I-105 A. Basal temperature <37°C


B. Serum pH >7.45
A. X-linked recessive inheritance C. Increased red blood cell levels of 2,3-diphospho-
AL

B. X-linked dominant inheritance glycerate


C. Autosomal recessive inheritance D. Hemoglobin concentration <10 mg/dL
I

D. Autosomal dominant inheritance E. Arterial PCO2 <30 mmHg


ER

E. Cannot be determined by the limited information


provided in this pedigree I-109. Independent of insurance status, income, age, and
AT

comorbid conditions, African-American patients are less


I-106. Diseases that are inherited in a multifactorial genetic likely to receive equivalent levels of care when compared
fashion (i.e., not autosomal dominant, autosomal reces- to white patients for the following scenarios:
M

sive, or X-linked) and are seen more frequently in persons A. Prescription of analgesic for pain control
bearing certain histocompatibility antigens include
E

B. Referral to renal transplantation


A. gluten-sensitive enteropathy C. Surgical treatment for lung cancer
S

B. neurofibromatosis D. Utilization of cardiac diagnostic and therapeutic


UR

C. adult polycystic kidney disease procedures


D. Wilson’s disease E. All of the above
E. cystic fibrosis
CO

I-110. Which of the following would be present in an indi-


I-107. A 32-year-old man seeks evaluation for ongoing fevers vidual who has lost nondeclarative memory?
of uncertain cause. He first noted a feeling of malaise about A. Inability to recall a spouse’s birthday
LL

3 months ago, and for the past 6 weeks, he has been experi- B. Inability to recall how to tie one’s shoe
encing daily fevers to as high as 39.4°C (103°F). He awak- C. Inability to recognize a photo that was taken at one’s
FU

ens with night sweats once weekly and has lost 4.5 kg. He wedding
complains of nonspecific myalgias and arthralgias. He has D. Inability to recognize a watch as an instrument for
no rashes and reports no ill contacts. He has seen his pri- keeping time
mary care physician on three separate occasions during E. Inability to remember the events of one’s high
this time and has had documented temperatures of 38.7°C school graduation
(101.7°F) while in the physician’s office. Multiple labora-
tory studies have been performed that have shown nonspe- I-111. A 24-year-old woman presents for a routine checkup
cific findings only. A complete blood count showed a white and complains only of small masses in her groin. She states
blood cell count of 15,700/µL with 80% polymorphonu- that they have been present for at least 3 years. On physical
I. INTRODUCTION TO CLINICAL MEDICINE — QUESTIONS 17

I-127. (Continued) I-131. Which of the following is true regarding drug effects
anxious but is alert, oriented, and appropriate. Temperature after an overdose in comparison to a reference dose?
is 36.8°C, blood pressure is 162/84 mmHg, heart rate is 108

0
A. Drug effects begin earlier, peak earlier, and last longer
beats/min, respiratory rate is 22 breaths/min, and oxygen

91
B. Drug effects begin earlier, peak later, and last longer
saturation is 99% on room air. Her pupils are equal and re-
C. Drug effects begin earlier, peak later, and last shorter
active. Cranial nerves are intact. Her oropharynx is slightly

55
D. Drug effects begin later, peak earlier, and last shorter
dry. There is no lymphadenopathy. Lungs are clear. She has
E. Drug effects begin later, peak later, and last longer
a regular heart beat with normal S1, S2, and no extra heart

6
sounds. Her abdomen has normal bowel sounds with slight I-132. A 28-year-old man with bipolar disorder, who is on

48
epigastric tenderness. Her skin is normal without any track lithium, is found in his room 2 days after not showing up
marks or rash. A complete metabolic panel and complete to work. He is arousable but dysarthric and has a mark-

88
blood count are normal. A urine toxicology screen reveals edly abnormal gait when trying to walk. Upon arrival at
heroin metabolites. Further evaluation should include: the emergency department, he has a grand mal seizure.

91
A. arterial blood gas The seizure is not sustained but recurs an hour after 6
B. blood cultures mg lorazepam is infused IV. In the postictal stage, he is

00
C. cerebrospinal fluid (CSF) analysis not arousable to sternal rub and lacks a gag reflex. His se-
D. echocardiogram rum sodium returns at 158 meq/L. In reference to his

AP
E. orifice examination seizures, all of the following are next steps in his manage-
ment except
I-128. Which of the following is a distinguishing feature of

TS
A. barbiturates
amphetamine overdose versus other causes of sympathetic
B. benzodiazepines
overstimulation due to drug overdose or withdrawal?
C. endotracheal intubation
HA
A. Hallucination D. free water replacement
B. Hot, dry, flushed skin and urinary retention E. phenytoin
W

C. History of benzodiazepine abuse


D. Markedly increased blood pressure, heart rate, and I-133. Which of the following statements regarding gastric
end-organ damage in the absence of hallucination decontamination for toxin ingestion is true?
AL

E. Nystagmus A. Activated charcoal’s most common side effect is as-


piration.
I

I-129. Which of the following findings suggests an opiate


ER

B. Gastric lavage via nasogastric tube is preferred over


overdose?
the use of activated charcoal in situations where
A. Anion gap metabolic acidosis with a normal lactate therapeutic endoscopy may also be warranted.
AT

B. Hypotension and bradycardia in an alert patient C. Syrup of ipecac has no role in the hospital setting.
C. Mydriasis D. There are insufficient data to support or exclude a
M

D. Profuse sweating and drooling benefit when gastric decontamination is used more
E. Therapeutic response to naloxone than 1 h after a toxic ingestion.
E

E. All of the above are true.


I-130. A patient with metabolic acidosis, reduced anion
S

gap, and increased osmolal gap is most likely to have I-134. What is the main contributor to the resting energy
UR

which of the following toxic ingestions? expenditure of an individual?


A. Lithium A. Adipose tissue
CO

B. Methanol B. Exercise level


C. Oxycodone C. Lean body mass
D. Propylene glycol D. Resting heart rate
E. Salicylate E. None of the above
LL
FU
0
I. INTRODUCTION TO CLINICAL MEDICINE

91
55
ANSWERS

6
48
I-1. and I-2. The answers are C and C. (Chap. 3) In evaluating the usefulness of a test, it is

88
imperative to understand the clinical implications of the sensitivity and specificity of that
test. By obtaining information about the prevalence of the disease in the population—the

91
specificity and sensitivity—one can generate a two-by-two table, as shown below. This ta-
ble is used to generate the total number of patients in each group of the population:

00
Disease Status

AP
Test Result Present Absent
Positive True-positive False-positive

TS
Negative False-negative True-negative
Total number of patients with disease Total number of patients without disease
HA
The sensitivity of the test is TP/(TP + FN). The specificity is TN/(TN + FP). In this case
the table is filled in as follows:
W

Disease Status
Test Result Present Absent
AL

Positive 42 237
Negative 8 713
I
ER

Total number of patients with disease Total number of patients without disease
= 50 = 950
AT

I-3. The answer is A. (Chap. 3) A capitation system provides physicians with a fixed pay-
ment per patient per year. This has the potential to encourage physicians to take on more
M

patients but to provide patients with fewer services because the physician is liable for
expenses. A fixed salary system encourages physicians to take on fewer patients. A fee-for-
E

service system encourages physicians to provide more services. Out-of-pocket services not
S

covered by insurers are available only to patients with adequate means to receive the service.
UR

I-4. The answer is C. (Chap. 3) A receiver operating characteristic curve plots sensitivity on
the y-axis and (1 – specificity) on the x-axis. Each point on the curve represents a cutoff
point of sensitivity and 1 – specificity. The area under the curve can be used as a quantita-
CO

tive measure of the information content of a test. Values range from 0.5 (a 45° line) repre-
senting no diagnostic information to 1.0 for a perfect test. See Figure I-4.
LL

I-5. The answer is A. (Chap. 3) Bayes’ theorem is used in an attempt to quantify uncer-
tainty by employing an equation that combines pretest probability with the testing char-
FU

acteristics of specificity and sensitivity. The pretest probability quantitatively describes


the clinician’s certainty of a diagnosis after doing a history and physical examination. The
equation is
Pretest probability × test sensitivity
Posttest probability = ---------------------------------------------------------------------------------------------------------------------
Pretest probability × test sensitivity +
( 1 – disease prevalence ) × test false-positive rate

18
I. INTRODUCTION TO CLINICAL MEDICINE — ANSWERS 19

1.0 FIGURE I-4 The receiver operating characteristic


(ROC) curves for three diagnostic exercise tests for de-

True-positive rate (1 – sensitivity)


tection of CAD: exercise ECG, exercise SPECT, and ex-

0
0.8 ercise echo. Each ROC curve illustrates the trade-off

91
that occurs between improved test sensitivity (accurate
detection of patients with disease) and improved test

55
0.6
specificity (accurate detection of patients without dis-
ease), as the test value defining when the test turns

6
0.4 from “negative” to “positive” is varied. A 45° line would

48
indicate a test with no information (sensitivity = speci-
ficity at every test value). The area under each ROC

88
0.2 ECHO curve is a measure of the information content of the
SPECT test. Moving to a test with a larger ROC area (e.g., from
No Imaging
exercise ECG to exercise echo) improves diagnostic ac-

91
0.0 0.2 0.4 0.6 0.8 1.0
curacy. However, these curves are not measured in the
same populations and the effect of referral biases on the

00
False-positive rate (1 – specificity) results cannot easily be discerned. (From KE Fleis-
chmann et al: JAMA 280:913, 1998, with permission.)

AP
In this manner, the uncertainty one faces in clinical decision making is quantified. By in-

TS
serting numbers into the equation, one can see that a low pretest probability combined
with a poorly sensitive and specific test will yield a low posttest probability. However, the
same test result, when combined with a high pretest probability, will yield a high posttest
HA
probability. There have been criticisms of this theorem. Unfortunately, few tests have
only two outcomes: positive and negative. This theorem does not take into account the
W

useful information that is gained from nonbinary test results. Further, it is cumbersome
to calculate the posttest probability for each individual circumstance and patient. Perhaps
AL

the most useful lesson from Bayes’ theorem is to take into account pretest probability
when ordering tests or interpreting test results. To be clinically useful, a clinical scenario
with a low pretest probability will require a test with high sensitivity and specificity. Con-
I
ER

versely, a high pretest probability presentation can be confirmed by a test with only aver-
age sensitivity and specificity.
AT

I-6. The answer is D. (Chap. 3) A positive likelihood ratio can only be interpreted in the
context of a pretest probability of disease. Disease prevalence in a certain region contrib-
M

utes to the patient’s pretest probability. However, other factors such as the patient’s age,
clinical history and risk factors for the disease in question are also important in deter-
mining pretest probability. Armed with an estimated pretest probability and a positive
E

test with a known likelihood ratio, the clinician can estimate a posttest probability of dis-
S

ease. Generally, diagnostic tests are most useful in patients with a medium pretest proba-
UR

bility (25–75%) of having a disease. For example, in a patient with a low pretest
probability of disease, a positive test can be misleading in that the patient’s posttest prob-
ability of disease is still low. The same applies for a patient with a high pretest probability
CO

of disease with a negative test: the negative test usually does not rule out disease. It is
therefore incumbent upon the physician to have a rough estimate of the pretest probabil-
ity of disease, positive likelihood ratio of the diagnostic test, and negative likelihood ratio
LL

of the diagnostic test prior to ordering the test.


FU

I-7. and. I-8. The answers are B and C. (Chap. 3) The goal of a meta-analysis is to summarize
the treatment benefit conferred by an intervention. Risk reduction is frequently expressed
by relative risk or odds ratios; however, clinicians also find it useful to be familiar with the
absolute risk reduction (ARR). This is the difference in mortality (or another endpoint)
between the treatment and the placebo arms. In this case, the absolute risk reduction is
10% – 2% = 8%. From this number, one can calculate the number needed to treat
(NNT), which is 1/ARR. The NNT is the number of patients who must receive the inter-
vention to prevent one death (or another outcome assessed in the study). In this case the
NNT is 1/8% = 12.5 patients.
0
II. NUTRITION

91
55
QUESTIONS

6
48
88
DIRECTIONS: Choose the one best response to each question.

91
00
II-1. A 19-year-old woman with anorexia nervosa under- II-3. A 51-year-old alcoholic man is admitted to the hospi-
goes surgery for acute appendicitis. The postoperative tal for upper gastrointestinal bleeding. From further his-
course is complicated by acute respiratory distress tory and physical examination, it becomes apparent that

AP
syndrome, and she remains intubated for 10 days. She his bleeding is from gingival membranes. He is intoxi-
develops wound dehiscence on postoperative day 10. cated and complains of fatigue. Reviewing his chart you

TS
Laboratory data show a white blood cell count of 4000/ find that he had a hemarthrosis evacuated 6 months ago
µL, hematocrit 35%, albumin 2.1 g/dL, total protein 5.8 and has been lost to follow-up since then. He takes no
g/dL, transferrin 54 mg/dL, and iron-binding capacity 88 medications. Laboratory data show platelets of 250,000,
HA
mg/dL. You are considering initiating nutritional therapy INR of 0.9. He has a diffuse hemorrhagic eruption on his
on hospital day 11. Which of the following is true regard- legs (Figure II-3, Color Atlas).
W

ing the etiology and treatment of malnutrition in this What is the recommended treatment for this patient’s
patient? underlying disorder?
AL

A. She has marasmus, and nutritional support should A. Folate


be started slowly. B. Niacin
I

B. She has kwashiorkor, and nutritional support should C. Thiamine


ER

be aggressive. D. Vitamin C
C. She has marasmic kwashiorkor, kwashiorkor predom- E. Vitamin K
inant, and nutritional support should be aggressive.
AT

D. She has marasmic kwashiorkor, marasmus predom- II-4. While working in the intensive care unit, you admit a
inant, and nutritional support should be slow. 57-year-old woman with acute pancreatitis and oliguric
M

renal failure. Respiratory rate is 26 breaths/min, heart


II-2. You are seeing a patient in follow-up 2 weeks after rate is 125 beats/min, and temperature is 37.2°C. Physical
E

hospitalization. The patient is recovering from nosoco- examination shows marked abdominal tenderness with
S

mial pneumonia due to a resistant Pseudomonas spp. His normoactive bowel sounds. A CT scan shows an inflamed
hospital course was complicated by a deep venous pancreas without hemorrhage. You calculate her
UR

thrombosis. The patient is currently on IV piperacillin/ APACHE-I score to be 28. When deciding on when to ini-
tazobactam and tobramycin via a tunneled catheter, war- tiate nutritional replacement in this patient, which of the
CO

farin, lisinopril, hydrochlorothiazide, and metoprolol. following statements is true?


Laboratory data this morning show an INR of 8.2. At
A. Bowel rest is the cornerstone of treatment for acute
hospital discharge his INR was stable at 2.5. He has no pancreatitis.
history of liver disease. What is the most likely cause of
LL

B. Administering parenteral nutrition within 24 h will


the elevated INR? decrease the risk of infection and mortality.
FU

A. The patient has inadvertently overdosed. C. Enteral feeding supports gut function by secretion
B. The patient has developed a recurrent deep venous of gastrointestinal hormones that stimulate gut
thrombosis, which has affected the laboratory data. trophic activity.
C. The patient is deficient in vitamin K and needs sup- D. In severe systemic response to inflammation, feed-
plementation. ing can be withheld initially because the patient is
D. The warfarin prescription was written incorrectly at likely to have adequate, spontaneous oral intake in
the time of discharge. the first 7 days.

47
Copyright © 2008, 2005, 2001, 1998, 1994, 1991, 1987 by The McGraw-Hill Companies, Inc.
Click here for terms of use.
0
III. ONCOLOGY AND HEMATOLOGY

91
55
QUESTIONS

6
48
88
DIRECTIONS: Choose the one best response to each question.

91
III-1. A 73-year-old male presents to the clinic with 3 months III-4. (Continued)
of increasing back pain. He localizes the pain to the lumbar C. cryoprecipitate

00
spine and states that the pain is worst at night while he is ly- D. recombinant factor VIII
ing in bed. It is improved during the day with mobilization. E. plasmapheresis

AP
Past history is notable only for hypertension and remote cig-
arette smoking. Physical examination is normal. Laboratory III-5. Which of the following statements regarding incidence
studies are notable for an elevated alkaline phosphatase. A of and risk factors for hepatocellular carcinoma is true?
lumbar radiogram shows a lytic lesion in the L3 vertebra.
Which of the following malignancies is most likely? TSA. A chemical toxin produced by Aspergillus species,
aflatoxin B has a strong association with develop-
HA
A. Gastric carcinoma ment of hepatocellular carcinoma and can be found
B. Non-small cell lung cancer in stored grains in hot, humid places.
W

C. Osteosarcoma B. In the United States, the incidence of hepatocellular


D. Pancreatic carcinoma carcinoma is decreasing.
C. Nonalcoholic steatohepatitis is not associated with
AL

E. Thyroid carcinoma
an increased risk for hepatocellular carcinogen.
III-2. Patients from which of the following regions need D. Fewer than 5% of individuals diagnosed with hepa-
I

not be screened for glucose-6-phosphate dehydrogenase


ER

tocellular carcinoma in the United States do not


(G6PD) deficiency when starting a drug that carries a risk have underlying cirrhosis.
for G6PD mediated hemolysis? E. The risk of developing hepatocellular carcinoma in
AT

A. Brazil individuals with hepatitis C infection is 50%.


B. Russia
M

III-6. You are asked to review the peripheral blood smear


C. Southeast Asia
from a patient with anemia. Serum lactate dehydrogenase
D. Southern Europe
is elevated and there is hemoglobinuria. This patient is
E

E. Sub-Saharan Africa
likely to have which physical examination finding? (See
S

F. None of the above


Figure III-6, Color Atlas.)
UR

III-3. All the following are vitamin K–dependent coagula- A. Goiter


tion factors except B. Heme-positive stools
CO

A. factor X C. Mechanical second heart sound


B. factor VII D. Splenomegaly
C. protein C E. Thickened calvarium
LL

D. protein S
E. factor VIII III-7. All of the enzyme deficiencies that lead to porphyrias are
inherited either as autosomal dominant (AD) or autosomal
FU

III-4. A 31-year-old male with hemophilia A is admitted recessive (AR) traits with one exception. Which of the fol-
with persistent gross hematuria. He denies recent trauma lowing most commonly occurs sporadically?
or any history of genitourinary pathology. The examina-
A. 5-ALA dehydratase-deficient porphyria
tion is unremarkable. Hematocrit is 28%. All the follow-
B. Acute intermittent porphyria
ing are treatments for hemophilia A except
C. Erythropoietic porphyria
A. desmopressin (DDAVP) D. Porphyria cutanea tarda
B. fresh-frozen plasma (FFP) E. Variegate porphyria

55
Copyright © 2008, 2005, 2001, 1998, 1994, 1991, 1987 by The McGraw-Hill Companies, Inc.
Click here for terms of use.
0
III. ONCOLOGY AND HEMATOLOGY

91
55
ANSWERS

6
48
III-1. The answer is B. (Chap. 94) Bone pain resulting from metastatic lesions may be diffi-

88
cult to distinguish from degenerative disease, osteoporosis, or disk disease in the elderly.
Generally, these patients present with insidious worsening localized pain without fevers

91
or signs of infection. In contrast to pain related to disk disease, the pain of metastatic dis-
ease is worse when the patient is lying down or at night. Neurologic symptoms related to

00
metastatic disease constitute an emergency. Lung, breast, and prostate cancers account
for approximately 80% of bone metastases. Thyroid carcinoma, renal cell carcinoma,

AP
lymphoma, and bladder carcinoma may also metastasize to bone. Metastatic lesions may
be lytic or blastic. Most cancers cause a combination of both, although prostate cancer is
predominantly blastic. Either lesion may cause hypercalcemia, although lytic lesions

TS
more commonly do this. Lytic lesions are best detected with plain radiography. Blastic le-
sions are prominent on radionuclide bone scans. Treatment and prognosis depend on the
HA
underlying malignancy. Bisphosphonates may reduce hypercalcemia, relieve pain, and
limit bone resorption.
W

III-2. The answer is B. (Chap. 101) Red blood cells utilize glutathione produced by the hex-
ose monophosphate shunt to compensate for increased production of reactive oxygen
AL

species (oxidant stress), usually induced by drugs or toxins. Defects in G6PD are the most
common congenital hexose monophosphate shunt defect. If the red blood cell (RBC) is
unable to maintain an adequate level of glutathione during oxidant stress, hemoglobin
I
ER

precipitates in the RBC, producing Heinz bodies. Because the G6PD gene is on the X
chromosome, almost all afflicted patients are males. G6PD deficiency is widely distrib-
uted throughout regions that are currently or were once highly malarial endemic. It is
AT

common in males of African, African-American, Sardinian, and Sephardic descent. In


most persons with G6PD deficiency, there is no evidence of symptomatic disease. How-
M

ever, infection, ingestion of fava beans, or exposure to an oxidative agent (drug or toxin)
can trigger an acute hemolytic event. Bite cells, Heinz bodies, and bizarre poikilocytes
E

may be evident on smear. The drugs that most commonly precipitate a G6PD crisis in-
clude dapsone, sulfamethoxazole, primaquine, and nitrofurantoin. The anemia is often
S

severe with rapid onset after drug ingestion, and renal failure can occur.
UR

III-3. The answer is E. (Chaps. 110 and 111) Vitamin K is a fat-soluble vitamin that plays an
essential role in hemostasis. It is absorbed in the small intestine and stored in the liver. It
CO

serves as a cofactor in the enzymatic carboxylation of glutamic acid residues on pro-


thrombin-complex proteins. The three major causes of vitamin K deficiency are poor di-
etary intake, intestinal malabsorption, and liver disease. The prothrombin complex
LL

proteins (factors II, VII, IX, and X and protein C and protein S) all decrease with vitamin
K deficiency. Factor VII and protein C have the shortest half-lives of these factors and
FU

therefore decrease first. Therefore, vitamin K deficiency manifests with prolongation of


the prothrombin time first. With severe deficiency, the aPTT will be prolonged as well.
Factor VIII is not influenced by vitamin K.

III-4. The answer is E. (Chaps. 110 and 111) Hemophilia A results from a deficiency of factor
VIII. Replacement of factor VIII is the centerpiece of treatment. Cessation of aspirin or non-
steroidal anti-inflammatory drugs (NSAIDs) is highly recommended. FFP contains pooled
plasma from human sources. Cryoprecipitate refers to FFP that is cooled, resulting in the

71
0
IV. INFECTIOUS DISEASES

91
55
QUESTIONS

6
48
88
DIRECTIONS: Choose the one best response to each question

91
IV-1. Which type of bite represents a potential medical IV-3. (Continued)

00
emergency in an asplenic patient? with 5% band forms and 93% polymorphonuclear cells.
Blood cultures are drawn followed by empirical vancomy-
A. Cat bite
cin therapy. These cultures remain negative for growth 5

AP
B. Dog bite
days later. He remains febrile but hemodynamically stable
C. Fish bite
but does develop a new lesion on his toe similar to those
D. Human bite

TS
on his fingers on hospital day 3. A transthoracic echocar-
IV-2. A 24-year-old man with advanced HIV infection pre- diogram reveals a 1-cm mobile vegetation on the cusp of
his aortic valve and moderate aortic regurgitation. A CT
HA
sents to the emergency department with a tan painless
nodule on the lower extremity (Figure IV-2, Color Atlas). scan of the abdomen shows an enlarged spleen with
He is afebrile and has no other lesions. He does not take wedge-shaped splenic and renal infarctions. What test
W

antiretroviral therapy, and his last CD4+ T cell count was should be sent to confirm the most likely diagnosis?
20/µL. He lives with a friend who has cats and kittens. A
A. Bartonella serology
AL

biopsy shows lobular proliferation of blood vessels lined


by enlarged endothelial cells and a mixed acute and B. Epstein-Barr virus (EBV) heterophile antibody
chronic inflammatory infiltrate. Tissue stains show gram- C. HIV polymerase chain reaction (PCR)
I

D. Peripheral blood smear


ER

negative bacilli. Which of the following is most likely to


be effective therapy for the lesion? E. Q fever serology
AT

A. Azithromycin IV-4. A 36-year-old man with HIV/AIDS (CD4+ lympho-


B. Cephazolin cyte count = 112/µL) develops a scaly, waxy, yellowish,
C. Interferon α patchy, crusty, pruritic rash on and around his nose. The
M

D. Penicillin rest of his skin examination is normal. Which of the fol-


E. Vancomycin lowing is the most likely diagnosis?
S E

IV-3. A 38-year-old homeless man presents to the emer- A. Molluscum contagiosum


gency room with a transient ischemic attack characterized
UR

B. Psoriasis
by a facial droop and left arm weakness lasting 20 min, and C. Reactivation herpes zoster
left upper quadrant pain. He reports intermittent subjec- D. Seborrheic dermatitis
CO

tive fevers, diaphoresis, and chills for the past 2 weeks. He


has had no recent travel or contact with animals. He has IV-5. A 28-year-old woman returns from a 6-week trip to
taken no recent antibiotics. Physical examination reveals a Tanzania in March. She calls your office 2 weeks later
slightly distressed man with disheveled appearance. His
LL

complaining of new symptoms of fever, mild abdominal


temperature is 38.2°C; heart rate is 90 beats per minute; pain, and headache. She feels like she has the flu. What
blood pressure is 127/74 mmHg. He has poor dentition. should you do next?
FU

Cardiac examination reveals an early diastolic murmur


over the left 3d intercostal space. His spleen is tender and 2 A. Ask her to come to the clinic in the next 24 h.
cm descended below the costal margin. He has tender B. Emergently refer her to the emergency department.
painful red nodules on the tips of the third finger of his C. Write her a prescription for oseltamivir and call her
right hand and on the fourth finger of his left hand that are in 24 h to ensure improvement.
new. He has nits evident on his clothes, consistent with D. Write her a prescription for a respiratory fluoro-
body louse infection. White blood cell count is 14,500, quinolone.

103
Copyright © 2008, 2005, 2001, 1998, 1994, 1991, 1987 by The McGraw-Hill Companies, Inc.
Click here for terms of use.
V. DISORDERS OF THE

0
CARDIOVASCULAR SYSTEM

91
55
QUESTIONS

6
48
88
DIRECTIONS: Choose the one best response to each question.

91
V-2. (Continued)

00
V-1. A 46-year-old white female presents to your office with
concerns about her diagnosis of hypertension 1 month ing. It is located infrarenally and measures 4.5 cm. All the
previously. She asks you about her likelihood of develop- following are true about the patient’s diagnosis except

AP
ing complications of hypertension, including renal failure
A. The 5-year risk of rupture of an aneurysm of this
and stroke. She denies any past medical history other than
size is 1 to 2%.
hypertension and has no symptoms that suggest second-

TS
B. Surgical or endovascular intervention is warranted
ary causes. She currently is taking hydrochlorothiazide 25
because of the size of the aneurysm.
mg/d. She smokes half a pack of cigarettes daily and drinks
HA
C. Infrarenal endovascular stent placement is an op-
alcohol no more than once per week. Her family history is
tion if the aneurysm experiences continued growth
significant for hypertension in both parents. Her mother
in light of the location of the aneurysm infrarenally.
died of a cerebrovascular accident. Her father is alive but
W

D. Surgical or endovascular intervention is warranted


has coronary artery disease and is on hemodialysis. Her
if the patient develops symptoms of recurrent ab-
blood pressure is 138/90. Body mass index is 23. She has
AL

dominal or back pain.


no retinal exudates or other signs of hypertensive retinop-
E. Surgical or endovascular intervention is warranted
athy. Her point of maximal cardiac impulse is not dis-
if the aneurysm expands beyond 5.5 cm.
I

placed but is sustained. Her rate and rhythm are regular


ER

and without gallops. She has good peripheral pulses. An V-3. A 45-year-old woman presents to the emergency room
electrocardiogram reveals an axis of –30 degrees with complaining of progressive dyspnea on exertion and
AT

borderline voltage criteria for left ventricular hypertrophy. abrupt onset of painful ulcerations on her toes. She has
Creatinine is 1.0 mg/dL. Which of the following items in noted the symptoms for the past 3 months. The dyspnea
her history and physical examination is a risk factor for a
M

has progressed such that she is only able to walk about 1


poor prognosis in a patient with hypertension? block without stopping. Over this same time, she has no-
ticed a cough that occasionally produces thin, pink-
E

A. Family history of renal failure and cerebrovascular


disease tinged sputum. She also has reports that her breathing is
S

B. Persistent elevation in blood pressure after the initi- worse at night. She sleeps on three pillows but awakens
UR

ation of therapy with dyspnea once or twice nightly. Over the past 2 days,
C. Ongoing tobacco use she has developed painful ulcerations on toes 1 and 4 on
D. Ongoing use of alcohol her left foot. She reports that the areas started as reddish
CO

E. Presence of left ventricular hypertrophy on ECG painful discoloration that ulcerated over the ensuing
days. She denies fevers, chills, or weight loss. She has no
V-2. A 68-year-old male presents to your office for routine history of chest pain, heart disease, or heart murmurs.
LL

follow-up care. He reports that he is feeling well and has She has been in good health until the past 3 months. She
no complaints. His past medical history is significant for takes no medications. Her last dental visit was ~8 months
FU

hypertension and hypercholesterolemia. He continues to ago. On physical examination, she appears in no distress.
smoke a pack of cigarettes daily. He is taking chlorthali- Vital signs: blood pressure of 145/92 mmHg, heart rate of
done 25 mg daily, atenolol 25 mg daily, and pravastatin 40 95 beats/min, respiratory rate of 24 breaths/min, temper-
mg nightly. Blood pressure is 133/85, and heart rate is 66. ature is 37.7°C, and SaO2 is 95% on room air. The cardio-
Cardiac and pulmonary examinations are unremarkable. vascular examination reveals a regular rate and rhythm.
A pulsatile abdominal mass is felt just to the left of the There is a III/VI mid-diastolic murmur with an occa-
umbilicus and measures approximately 4 cm. You confirm sional low-pitched mid-diastolic sound that occurs when
the diagnosis of abdominal aortic aneurysm by CT imag- the patient is in the upright position. The jugular venous

175
Copyright © 2008, 2005, 2001, 1998, 1994, 1991, 1987 by The McGraw-Hill Companies, Inc.
Click here for terms of use.
VI. DISORDERS OF THE

0
RESPIRATORY SYSTEM

91
55
QUESTIONS

6
48
88
DIRECTIONS: Choose the one best response to each question.

91
VI-1. A patient is evaluated in the emergency department VI-4. (Continued)

00
for peripheral cyanosis. Which of the following is not a C. nonsteroidal anti-inflammatory drugs (NSAIDs)
potential etiology? D. isoniazid

AP
E. amiodarone
A. Cold exposure
B. Deep venous thrombosis VI-5. A 26-year-old man presents to the clinic with 3 days

TS
C. Methemoglobinemia of severe sore throat and fever. All of the following sup-
D. Peripheral vascular disease
port the diagnosis of streptococcal pharyngitis except
E. Raynaud’s phenomenon
HA
A. cough
VI-2. Which of the following associations correctly pairs B. fever
W

clinical scenarios and community-acquired pneumonia C. pharyngeal exudates


(CAP) pathogens? D. positive rapid streptococcal throat antigen test
E. tender cervical lymphadenopathy
AL

A. Aspiration pneumonia: Streptococcus pyogenes


B. Heavy alcohol use: atypical pathogens and Staphylo-
coccus aureus VI-6. Which of the following has been shown to decrease
I
ER

duration of nonspecific upper respiratory tract symptoms?


C. Poor dental hygiene: Chlamydia pneumoniae, Kleb-
siella pneumoniae A. Azithromycin
D. Structural lung disease: Pseudomonas aeruginosa, S. B. Echinacea
AT

aureus C. Vitamin C
E. Travel to southwestern United States: Aspergillus spp. D. Zinc
M

E. None of the above


VI-3. A 54-year-old female presents to the hospital because of F. All of the above
E

hemoptysis. She has coughed up approximately 1 teaspoon


of blood for the last 4 days. She has a history of cigarette
S

VI-7. A 24-year-old man presents to the emergency room


smoking. A chest radiogram shows diffuse bilateral infiltrates complaining of shortness of breath and right-sided chest
UR

predominantly in the lower lobes. The hematocrit is 30%, pain. The symptoms began abruptly about 2 hours previ-
and the serum creatinine is 4.0 mg/dL. Both were normal ously. The pain is worse with inspiration. He denies fevers
previously. Urinalysis shows 2+ protein and red blood cell
CO

or chills and has not had any leg swelling. He has no past
casts. The presence of autoantibodies directed against which medical history but smokes 1 pack of cigarettes daily. On
of the following is most likely to yield a definitive diagnosis? physical examination, he is tachypneic with a respiratory
rate of 24 breaths/min. His oxygen saturation is 94% on
LL

A. Glomerular basement membrane


B. Glutamic acid decarboxylase room air. Breath sounds are decreased in the right lung,
C. Phospholipids and there is hyperresonance to percussion. A chest radio-
FU

D. Smooth muscle graph confirms a 50% pneumothorax of the right lung.


E. U1 ribonucleoprotein (RNP) What is the best approach for treatment of this patient?
A. Needle aspiration of the pneumothorax
VI-4. All the following drugs can cause eosinophilic pneu-
B. Observation and administration of 100% oxygen
monia except
C. Placement of a large-bore chest tube
A. nitrofurantoin D. Referral for thoracoscopy with stapling of blebs and
B. sulfonamides pleural abrasion

237
Copyright © 2008, 2005, 2001, 1998, 1994, 1991, 1987 by The McGraw-Hill Companies, Inc.
Click here for terms of use.
VII. DISORDERS OF THE URINARY

0
AND KIDNEY TRACT

91
55
QUESTIONS

6
48
88
DIRECTIONS: Choose the one best response to each question.

91
00
VII-1. A clinic patient who has a diagnosis of polycystic VII-4. (Continued)
kidney disease has been doing research on the Internet. dium is 140 meq/L, BUN is 49 mg/dL. Urine sediment
She is asymptomatic and has no significant family his- shows rare granular casts. His urine sodium is 50 meq/L,

AP
tory. She asks you for screening for intracranial aneu- urine osmolality 287 mosmol and urine creatinine is 35 mg/
rysms. You recommend which of the following? dL. What is the cause of this patient’s acute renal failure?

TS
A. Head CT scan without contrast A. Acute interstitial nephritis
B. CT angiogram B. Acute tubular necrosis
HA
C. Cerebral angiogram C. Glomerulonephritis
D. Magnetic resonance angiogram D. Hepatorenal syndrome
E. No further testing E. Prerenal azotemia
W

VII-2. Which of the following is the most potent stimulus VII-5. The pain associated with acute urinary tract ob-
struction is a result of which of the following?
AL

for hypothalamic production of arginine vasopressin?


A. Hypertonicity A. Compensatory natriuresis
I

B. Hyperkalemia B. Decreased medullary blood flow


ER

C. Hypokalemia C. Increased renal blood flow


D. Hypotonicity D. Vasodilatory prostaglandins
AT

E. Intravascular volume depletion


VII-6. Preoperative assessment of a 55-year-old male pa-
VII-3. A 28-year-old woman with HIV on antiretroviral ther- tient going for coronary angiography shows an estimated
M

apy complains of abdominal pain in the emergency depart- glomerular filtration rate of 33 mL/min per 1.73 m2 and
ment. Laboratory data show a creatinine of 3.2 mg/dL; her poorly controlled diabetes. He is currently on no nephro-
E

baseline creatinine is 1.0 mg/dL. Urinalysis shows large num- toxic medications, and the nephrologist assures you that
he does not currently have acute renal failure. The case is
S

bers of white blood cells and red blood cells without epithelial
cells, leukocyte esterase, or nitrites. Which test is indicated to due to begin in 4 h, and you would like to prevent con-
UR

diagnose the cause of her acute renal failure? trast nephropathy. Which agent will definitely reduce the
risk of contrast nephropathy?
A. Acid-fast stain of the urine
CO

B. Anti-GBM (glomerular base membrane) antibodies A. Dopamine


C. Renal angiogram B. Fenoldopam
D. Renal ultrasound C. Indomethacin
LL

E. Urine electrolytes D. N-acetylcysteine


E. Sodium bicarbonate
FU

VII-4. You are evaluating a 40-year-old patient admitted to


VII-7. All the following forms of glomerulonephritis (GN)
the hospital with cirrhosis and an upper gastrointestinal
have associated normal serum complement C4 levels except
bleed. The bleeding was treated with endoscopy and photo-
coagulation, and the patient is now stable. He required two A. lupus nephritis stage IV
units of packed red blood cells. He was briefly hypotensive B. poststreptococcal GN
upon admission but has remained stable for the past 5 days. C. hemolytic-uremic syndrome
He is becoming oliguric. Laboratory data show a creatinine D. membranoproliferative GN type II
of 4.0 mg/dL, whereas his baseline is 0.8–1.1 mg/dL. So- E. endocarditis-associated GN

283
Copyright © 2008, 2005, 2001, 1998, 1994, 1991, 1987 by The McGraw-Hill Companies, Inc.
Click here for terms of use.
VIII. DISORDERS OF THE

0
GASTROINTESTINAL SYSTEM

91
55
QUESTIONS

6
48
88
DIRECTIONS: Choose the one best response to each question.

91
VIII-3. (Continued)

00
VIII-1. A 46-year-old man is admitted to the hospital for
upper gastrointestinal (GI) bleeding. He has a known his- hours. These are associated with a feeling of excess
tory of peptic ulcer disease, for which he takes a proton- gas. She denies any history of alcohol abuse. She has

AP
pump inhibitor. His last admission for upper GI bleeding no medical history of hypertension or hyperlipid-
was 4 years ago. After fluid resuscitation, he is hemody- emia. On physical examination, she is writhing in dis-
namically stable and his hematocrit has not changed in tress and slightly diaphoretic. Vital signs are: heart
the past 8 h. Upper endoscopy is performed. Which of the
following findings at endoscopy is most reassuring that TSrate 127 beats/min, blood pressure 92/50 mmHg, res-
piratory rate 20 breaths/min, temperature 37.9°C,
HA
the patient will not have a significant rebleeding episode SaO2 88% on room air. Her body mass index is 29 kg/
within the next 3 days? m2. The cardiovascular examination reveals a regular
tachycardia. The chest examination shows dullness to
W

A. Adherent clot on ulcer


percussion at bilateral bases with a few scattered
B. Clean-based ulcer
crackles. On abdominal examination, bowel sounds
AL

C. Gastric ulcer with arteriovenous malformations


are hypoactive. There is no rash or bruising evident
D. Visible bleeding vessel
on inspection of the abdomen. There is voluntary
E. Visible nonbleeding vessel
I

guarding on palpation. The pain with palpation is


ER

VIII-2. Which of the following statements about alcoholic greatest in the periumbilical and epigastric area with-
liver disease is not true? out rebound tenderness. There is no evidence of jaun-
AT

dice, and the liver span is about 10 cm to percussion.


A. Pathologically, alcoholic cirrhosis is often character- Amylase level is 750 IU/L, and lipase level is 1129 IU/L.
ized by diffuse fine scarring with small regenerative Other laboratory values include: aspartate amino
M

nodules. transferase (AST) 168 U/L, alanine aminotransferase


B. The ratio of AST to ALT is often higher than 2. (ALT) 196 U/L, total bilirubin 2.3 mg/dL, alkaline
E

C. Serum aspartate aminotransferase levels are often phosphatase level 268 U/L, lactate dehydrogenase
S

greater than 1000 U/L. LDH 300 U/L, and creatinine 1.9 mg/dL. The hemato-
D. Concomitant hepatitis C significantly accelerates the
UR

crit is 43%, and white blood cell (WBC) count is


development of alcoholic cirrhosis. 11,500/µL with 89% neutrophils. An arterial blood
E. Serum prothrombin times may be prolonged, but gas shows a pH of 7.32, Pa CO2 32 mmHg, and a PaO2 of
CO

activated partial thromboplastin times are usually 56 mmHg. An ultrasound confirms a dilated common
not affected. bile duct with evidence of pancreatitis manifested as
an edematous and enlarged pancreatitis. A CT scan
VIII-3. A 47-year-old woman presents to the emergency
LL

shows no evidence of necrosis. After 3 L of normal sa-


room with severe mid-abdominal pain radiating to
line, her blood pressure comes up to 110/60 mmHg
her back. The pain began acutely and is sharp. She de-
FU

with a heart rate of 105 beats/min. Which of the fol-


nies cramping or flatulence. She has had two episodes
lowing statements best describes the pathophysiology
of emesis of bilious material since the pain began, but
of this disease?
this has not lessened the pain. She currently rates the
pain as a 10 out of 10 and feels the pain is worse in the A. Intrapancreatic activation of digestive enzymes with
supine position. For the past few months, she has had autodigestion and acinar cell injury
intermittent episodes of right upper and mid-epigas- B. Chemoattraction of neutrophils with subsequent
tric pain that occur after eating but subside over a few infiltration and inflammation

307
Copyright © 2008, 2005, 2001, 1998, 1994, 1991, 1987 by The McGraw-Hill Companies, Inc.
Click here for terms of use.
0
IX. RHEUMATOLOGY AND IMMUNOLOGY

91
55
QUESTIONS

6
48
88
DIRECTIONS: Choose the one best response to each question.

91
00
IX-1. A 73-year-old woman with a medical history of obe- IX-3. (Continued)
sity and diabetes mellitus presents to your clinic com- and pericholecystitis necessitating omentectomy and chole-
plaining of right knee pain that has been progressive and cystectomy. However, the pain continued unchanged. He

AP
is worse with walking or standing. She has taken over- currently describes it as periumbilical and radiating into his
the-counter nonsteroidal anti-inflammatory drugs with- groin and legs. It becomes worse with eating. The patient has
out relief. She wants to know what is wrong with her knee also had episodic severe testicular pain, bowel urgency, nau-
and what may have caused it. X-rays are performed and
reveal cartilage loss and osteophyte formation. Which of TSsea, vomiting, and diuresis. He has lost ~22.7 kg over the
preceding 6 months. His past medical history is significant
HA
the following represents the most potent risk factor for of hypertension that has recently become difficult to control.
the development of osteoarthritis?
Medications on admission include aspirin, hydrochlo-
W

A. Age rothiazide, hydromorphone, lansoprazole, metoprolol,


B. Gender and quinapril. On physical examination, the patient ap-
AL

C. Genetic susceptibility pears comfortable. His blood pressure is 170/100 mmHg,


D. Obesity his heart rate is 88 beats/min, and he is afebrile. He has
E. Previous joint injury normal first and second heart sounds without murmurs,
I
ER

and an S4 is present. There are no carotid, renal, abdomi-


IX-2. A 42-year-old obese male presents to your office with nal, or femoral bruits.
complaints of paresthesias in the right hand that are
AT

worst in the fourth and fifth fingers. Symptoms have been His lungs are clear to auscultation. Bowel sounds are nor-
present intermittently for the last 4 months. He has no mal. Abdominal palpation demonstrates minimal diffuse
tenderness without rebound or guarding. No masses are
M

other past medical history and takes no medications. The


examination is significant for an intact neurologic exami- present, and the stool is negative for occult blood. During
nation of the right upper extremity but mild wasting of the examination, the patient develops Raynaud's phenome-
E

the intrinsic muscles on inspection of the right hand. non in his right hand that persists for several minutes. His
S

Laboratories show a normal white blood cell count, he- neurologic examination is intact. Admission laboratory
UR

moglobin, and sedimentation rate. Electrolytes and cre- studies reveal an erythrocyte sedimentation rate of 72 mm/h,
atinine and liver function tests are normal except for a a BUN of 17 mg/dL, and a creatinine of 0.8 mg/dL. The pa-
serum glucose of 148 mg/dL. What is the most likely eti- tient has no proteinuria or hematuria. Tests for antinuclear
CO

ology of this patient’s symptoms? antibodies, anti-double-stranded-DNA antibodies, and an-


tineutrophil cytoplasmic antibodies are negative. Liver
A. Diabetes mellitus function tests are abnormal with an AST of 89 IU/L and an
B. Cholesterol emboli
LL

ALT of 112 IU/L. Hepatitis B surface antigen and e antigen


C. Churg-Strauss disease are positive. Mesenteric angiography demonstrates small
D. Cervical spondylosis
FU

beaded aneurysms of the superior and inferior mesenteric


E. Neurogenic thoracic outlet syndrome veins. What is the most likely diagnosis?
IX-3. A 54-year-old man is admitted for persistent lower ab- A. Hepatocellular carcinoma
dominal and groin pain that began 7 months previously. B. Ischemic colitis
Two months before his present admission, he required ex- C. Microscopic polyangiitis
ploratory laparoscopy for acute abdominal pain and pre- D. Mixed cryoglobulinemia
sumed cholecystitis. This revealed necrotic omental tissue E. Polyarteritis nodosa

345
Copyright © 2008, 2005, 2001, 1998, 1994, 1991, 1987 by The McGraw-Hill Companies, Inc.
Click here for terms of use.
0
X. ENDOCRINOLOGY AND METABOLISM

91
55
QUESTIONS

6
48
88
DIRECTIONS: Choose the one best response to each question.

91
00
X-1. What is the most common cause of hypothyroidism X-4. A 44-year-old woman seeks evaluation for irregular
worldwide? menstrual cycles with heavy menstrual bleeding. She re-
ports that her menses had been regular with 28-day cycles
A. Autoimmune disease

AP
since her early twenties. However, for the past 6 months,
B. Graves’ disease
her cycles have been 22–25 days with heavy associated
C. Iatrogenic causes
bleeding that is unusual for her. She has had rare hot

TS
D. Iodine deficiency
flashes and sleep disturbance. She is requesting assistance
E. Medication side effects
in controlling these symptoms. You suspect she is peri-
HA
X-2. A 23-year-old woman presents to clinic complaining menopausal, and hormonal testing on day 2 of her
of months of weight gain, fatigue, amenorrhea, and wors- menses confirms this suspicion. You are considering
treatment with oral contraceptives for control of her
W

ening acne. She cannot identify when her symptoms be-


gan precisely, but she reports that without a change in her symptoms and to protect against unintended pregnancy.
diet she has noted a 12.3-kg weight gain over the past 6 All of the following would be considered contraindica-
AL

months. She has been amenorrheic for several months. tions to use of oral contraceptive pills except
On examination she is noted to have truncal obesity with A. breast cancer
I
ER

bilateral purplish striae across both flanks. Cushing’s syn- B. cigarette smoking
drome is suspected. Which of the following tests should C. kidney disease
be used to make the diagnosis? D. liver disease
AT

A. 24-h urine free cortisol E. prior history of deep venous thrombosis


B. Basal adrenocorticotropic hormone (ACTH)
M

X-5. All the following are risk factors for the development
C. Corticotropin-releasing hormone (CRH) level at 8 A.M.
D. Inferior petrosal venous sampling of osteoporotic fractures except
E

E. Overnight 1 mg dexamethasone suppression test A. African-American race


S

B. current cigarette smoking


X-3. Secretion of gonadotropin releasing-hormone (GnRH)
UR

C. female sex
normally stimulates release of luteinizing hormone (LH) D. low body weight
and follicle-stimulating hormone (FSH) which promote E. physical inactivity
CO

production and release of testosterone and estrogen.


Which mechanism below best explains how long-acting X-6. All the following drugs are associated with an in-
gonadotropin-releasing hormone agonists (e.g., leupro- creased risk of osteoporosis in adults except
lide) decrease testosterone levels in the management of
LL

prostate cancer? A. cyclosporine


B. dilantin
FU

A. GnRH agonists also promote production of sex hor- C. heparin


mone–binding globulin, which decreases the avail- D. prednisone
ability of testosterone E. ranitidine
B. Negative feedback loop between GnRH and LH/FSH
C. Sensitivity of LH and FSH to pulse frequency of GnRH X-7. A 34-year-old woman presents to your clinic with a va-
D. Translocation of the cytoplasmic nuclear receptor riety of complaints that have been worsening over the past
into the nucleus with constitutive activation of year or so. She notes fatigue, amenorrhea, and weight
GnRH gain. She states that her primary physician diagnosed her

379
Copyright © 2008, 2005, 2001, 1998, 1994, 1991, 1987 by The McGraw-Hill Companies, Inc.
Click here for terms of use.
0
XI. NEUROLOGIC DISORDERS

91
55
QUESTIONS

6
48
88
DIRECTIONS: Choose the one best response to each question.

91
00
XI-1. Delirium, an acute confusional state, is a common XI-2. (Continued)
disorder that remains a major cause of morbidity and D. Sumatriptan, 50 mg orally, at the onset of an attack
mortality in the United States. Which patient is at the E. Surgical consultation for microvascular decompres-

AP
highest risk for developing delirium? sion of the trigeminal nerve
A. A 36-year-old man admitted to the medical ward XI-3. You are seeing your patient with polymyositis in fol-

TS
with a deep venous thrombosis low-up. He has been taking prednisone at high doses for 2
B. A 55-year-old man postoperative day 2 from a total months, and you initiated mycophenolate mofetil at the
HA
colectomy last clinic visit for a steroid-sparing effect. He began a
C. A 68-year-old woman admitted to the intensive care steroid taper 2 weeks ago. His symptoms were predomi-
unit (ICU) with esophageal rupture nantly in the lower extremities and face, and he has im-
W

D. A 74-year-old woman in the preoperative clinic be- proved considerably. He no longer needs a cane and his
fore hip surgery voice has returned to normal. Laboratory data show a
AL

E. An 84-year-old man living in an assisted living facility creatine kinase (CK) of 1300 U/L, which is unchanged
from 2 months ago. What is the most appropriate next
XI-2. A 46-year-old man presents for evaluation of severe
I

step in this patient’s management?


ER

unilateral headache. He states that he has had episodes of


intermittent headache for the past 3 years. He describes A. Continue current management
the headaches as a stabbing pain located near his right B. Continue high-dose steroids with no taper
AT

temple. They occur abruptly and last up to 3 h at a time, C. Switch mycophenolate to methotrexate
during which he feels incapacitated, rating the pain as a D. Repeat muscle biopsy
M

10 out of 10. Most of the time, the headaches begin in the XI-4. A patient complains of numbness in his neck. Over
early morning hours. When they occur, he finds it impos- months, the numbness has become more pronounced
E

sible to sleep. He feels that rubbing his head improves the and involves a dense area bilaterally from the sternal
pain but has noticed no other factors that relieve the pain.
S

notch to the area behind the ear. On examination, scalp


Specifically, he has had no improvement with acetamino-
UR

sensation, cranial nerve function, and upper extremity


phen, naprosyn, or oxycodone. When the headaches oc- motor examination are normal. The patient has de-
cur, he develops nasal congestion and tearing on the side creased pain and temperature sensation in the distribu-
CO

of the pain. He believes the headaches occur in cycles. He tion of C4. Vibration sense is normal. Cranial and caudal
will have the headaches almost daily for up to 2 weeks at a to the affected area, sensation is intact. Bladder and anal
time, but then have no headaches at all for as long as 3 sphincter function are also normal. What is the most
months. He has decided to seek medical advice because
LL

likely cause of this patient’s neurologic disorder?


he is worried about the possibility of a brain tumor be-
cause of the severity of the headaches. He takes no medi- A. Amyotrophic lateral sclerosis
FU

cines regularly. His vital signs and physical examination B. Disc herniation
are normal. What is the best approach to treatment of C. Intramedullary tumor
these headaches? D. Knife or bullet injury
E. Neurosyphilis
A. Fluticasone nasal spray and loratadine, 10 mg orally
B. Indomethacin, 25 mg three times daily XI-5. A 56-year-old male is admitted to the intensive care unit
C. Oxygen at 10–12 L/min by nasal cannula at the on- with a hypertensive crisis after cocaine use. Initial blood
set of an attack pressure is 245/132. On physical examination the patient is

421
Copyright © 2008, 2005, 2001, 1998, 1994, 1991, 1987 by The McGraw-Hill Companies, Inc.
Click here for terms of use.
0
XII. DERMATOLOGY

91
55
QUESTIONS

6
48
88
DIRECTIONS: Choose the one best response to each question.

91
00
XII-1. All of the following statements regarding atopic der- XII-4. (Continued)
matitis are true except chest, upper arms, and thighs, for example, can become hir-
sute. In the same person, however, hair growth on the scalp
A. 80% of children with atopic dermatitis also coex-

AP
resembles male-pattern balding (androgenic alopecia). What
press allergic rhinitis or asthma.
best describes the reason for the different effects of andro-
B. Exacerbations and remissions are typical.
gens on hair growth on the scalp compared to other areas?

TS
C. Patients with atopic dermatitis often have increased
levels of circulating IgE. A. Androgen levels in the scalp are lower than they are
HA
D. The adult form of atopic dermatitis is commonly lo- in other parts of the body.
calized to the hands. B. Hirsutism causes stress, which leads to male-pattern
E. When one parent is affected with atopic dermatitis, balding.
W

there is an 80% prevalence of atopic dermatitis in C. In the scalp, androgen excess causes hair to spend
the children. less time in the growth phase.
AL

D. Sun-exposed areas tend to be less androgen-sensitive.


XII-2. A patient with acne vulgaris is considering treatment
with isotretinoin. Her acne has been refractory to oral an- XII-5. A 22-year-old woman comes to your office con-
I
ER

tibiotics and topical washes. In addition to counseling re- cerned about sun exposure. She brings a few sun block
garding the teratogenic effects of this medication, which creams into your office and wants to know which one is
other side effect has been documented with isotretinoin best for preventing wrinkling and blotchiness. She is less
AT

for the treatment of acne vulgaris? concerned about sunburn because she is trying to get a
better tan. Blocking which ultraviolet rays will achieve
A. Bradycardia
M

her desired result?


B. Cutaneous lymphomas
C. Fugue state A. UV-A
E

D. Hypertriglyceridemia B. UV-B
S

E. Tooth discoloration C. Both UV-A and UV-B equally


D. Neither UV-A nor UV-B
UR

XII-3. A 42-year-old man recently returned from a 2-week


antelope hunting/camping trip to Tanzania. Near the end XII-6. Identify this skin lesion (see Color Atlas, Figure XII-6).
CO

of the trip, he noted a painful sore on his foot that has be-
A. Herpes simplex virus
come worse and is pictured in the figure (Color Atlas, Fig-
B. Molluscum contagiosum
ure XII-3). He is concerned about the possibility of
C. Scabies
trypanosomiasis. Which of the following is most likely to
LL

D. Tinea corporis
yield a diagnosis?
XII-7. A 48-year-old diabetic man presents to the emergency
FU

A. Light microscopy of fluid from the skin lesion


B. Polymerase chain reaction (PCR) from the serum department complaining of blistering of the skin. One
C. Serology from the serum week prior, he was seen by his cardiologist, where he was
D. Stool analysis for parasites started on a new blood pressure medication. His major
E. Urinary antigen testing complaints are blisters on his arms, chest, and face and
pruritus. On physical examination, he is afebrile. The
XII-4. Androgen excess in women affects terminal hair growth “blisters” are actually shallow erosions associated with ery-
patterns. In states of androgen excess, the upper lip, chin, thema, scale, and crust formation, Nikolsky’s sign is

457
Copyright © 2008, 2005, 2001, 1998, 1994, 1991, 1987 by The McGraw-Hill Companies, Inc.
Click here for terms of use.
2.
CARDIOLOGY QUESTIONS AND ANSWER S

1 0
59
65
QUESTIONS 2. A 78-year-old woman comes to your office with irregu-

48
lar palpitations, shortness of breath, and fatigue. She
Multiple Choice (choose the best answer) has type 2 diabetes mellitus and hypertension, both of

88
which she has kept under excellent control since she had
a minor stroke at age 69. She has no other known medi-
A R R H Y T H M I A S A N D C L I N I C A L SY N D RO M E S

91
cal problems of note. Examination findings are normal
except that she has a soft, barely audible right carotid
1. You are asked to see a 22-year-old man who under-
bruit. The electrocardiogram (ECG) from a week ago is

00
went a routine electrocardiographic (ECG) examina-
shown (Figure 2.Q2). A transthoracic echocardiogram
tion as part of a preemployment physical examination
shows normal left ventricular function and moderate
for a large manufacturing company. Other than having

AP
left atrial enlargement. You order another ECG today
a fairly sedentary lifestyle, he is healthy and reports
since her regular heart rate confirms normal sinus
no symptoms. Cardiac examination findings are nor-

TS
rhythm. What anticoagulation therapy should you rec-
mal. You view the 12-lead ECG shown in Figure 2.Q1.
ommend for this patient at this time?
Which of the following would be the best management
a. Warfarin (goal international normalized ratio, 2.0–3.0)
HA
strategy for this patient at this time?
b. Aspirin 325 mg
a. Perform an electrophysiology study and catheter ablation.
c. Catheter ablation of this arrhythmia to eliminate stroke
b. Observe.
W

risk
c. Start therapy with metoprolol 25 mg twice daily.
d. Aspirin 325 mg and clopidogrel 75 mg
d. Start therapy with procainamide 150 mg twice daily.
AL

e. No anticoagulation since she is now in normal sinus rhythm


e. Start therapy with digoxin 0.125 mg daily.
and at low risk
I
ER
AT
M

I aVR V1 V4
SE
UR

II aVL V2 V5
CO
LL

III aVF V3 V6
FU

II

Figure 2.Q1

9
I aVR V1 V4

1 0
59
II aVL V2 V5

65
III aVF V3 V6

48
88
II

91
00
V1

AP
V5

TS
Figure 2.Q2 HA
3. An 85-year-old man has had 2 separate syncopal epi- On examination, the patient was fully orientated
sodes that occurred suddenly and without warning. On and appeared younger than his age. His heart rate and
W

the first occasion, he was talking to his daughter on the blood pressure were satisfactory, but his systolic pres-
phone—then he was lying on the floor. Recovery was sure decreased by 10 mm Hg on standing. Bilateral
immediate, but he injured his left hand. On the second carotid bruits and a harsh 3/6 ejection murmur were
AL

occasion, he was in the kitchen making a sandwich. His present. The murmur was loudest at the left sternal
daughter was present and was able to break his fall. She border, with some radiation into the neck. Reversed
I
ER

noted that both arms made jerking movements, which splitting of the second heart sound (separation during
resolved when he was placed into a recovery position expiration rather than inspiration) was also present, but
on his left side. On both occasions, a full recovery no diastolic murmur was heard. There was no evidence
AT

occurred within a few minutes without sequelae. of jugular venous distention, and his lungs were clear,
M
SE

I aVR V1 V4
UR

II aVL V2 V5
CO

III aVF V3 V6
LL
FU

V1

II

V5

Figure 2.Q3

10 • M AYO C L I N I C I N T E R N A L M E D I C I N E B OA R D R E VI EW: Q U E S T I O N S A N D A N S W E R S
but a small amount of peripheral edema was present. a. Add sotalol.
Gait was normal and there were no neurologic signs of b. Add dronedarone.
note. The electrocardiogram is shown in Figure 2.Q3. c. Refer for catheter ablation.
This patient’s presentation is most consistent with d. Continue the current regimen.
which of the following?

0
a. Cardiogenic syncope C A R D I AC P H Y S I C A L E X A M I NAT I O N

1
b. A possible seizure disorder

59
c. Stroke or transient ischemic attack 6. A 30-year-old woman is referred for a pre-insurance
d. Neurocardiogenic syncope examination. She has no complaints. She is active

65
e. Orthostatic response and runs with her large dog about 1 hour daily (4–5
miles). She has no exercise limitations and was hiking

48
4. A 55-year-old woman with nonischemic dilated car-
diomyopathy and exertional dyspnea (New York Heart at an elevation of 3,030 m in the Rockies last summer.
On examination, her pulse rate is 50 beats per minute,

88
Association [NYHA] functional class III) and 2-pillow
orthopnea underwent placement of an implantable blood pressure is 120/80 mm Hg, and she is thin. On
cardiac examination, there is a normal first heart sound

91
cardioverter-defibrillator 4 years ago for sudden cardiac
arrest prophylaxis. Now she returns and asks whether and normal splitting; an early low-pitched diastolic
sound is consistent with a third heart sound (S3). There

00
cardiac resynchronization therapy may be appropri-
ate. Her current medications are lisinopril 5 mg daily, is a grade 1 to 2 midsystolic murmur at the left upper
carvedilol 6.25 mg twice daily, spironolactone 25 mg sternal border. The murmur disappears with the Valsalva

AP
daily, and furosemide 40 mg daily. maneuver. Carotid upstrokes are normal, and the jugu-
Physical examination and diagnostic study findings lar venous pressure is normal. No systolic clicks can be
appreciated at rest or with the squat-to-stand maneuver.

TS
were as follows: heart rate 97 beats per minute, blood
pressure 142/70 mm Hg, jugular venous pressure 12 cm The electrocardiogram shows sinus bradycardia (heart
rate 48 beats per minute) but is otherwise normal. What
HA
water, lungs clear, lateral displacement of the cardiac
impulse, audible third heart sound, and no murmurs; further cardiac testing needs to be done?
a. Cardiac catheterization to measure a gradient across the
electrocardiography showed left bundle branch block
W

mitral valve
(LBBB) with QRS duration of 130 ms, and transtho-
b. Stress echocardiography
racic echocardiography showed left ventricular ejection
c. Treadmill ECG
AL

fraction (LVEF) of 20%. What is the most appropriate


d. No additional testing
management for this patient at this point?
e. Holter monitoring
I

a. There is no indication for cardiac resynchronization therapy


ER

(CRT)—the symptoms are not severe enough. 7. Which of the following statements about shunts in the
b. There is no indication for CRT—the QRS complex is not atrial septum is true?
AT

wide enough. a. The auscultatory findings associated with atrial septal defect
c. Upgrade to CRT now. (ASD) are paradoxical splitting of the second heart sound
d. Up-titrate the dosages of lisinopril and carvedilol. (S2) and a diastolic flow rumble heard midsternum.
M

b. Patent foramen ovale (PFO) is uncommon in healthy adults


5. A 32-year-old woman who received a diagnosis of peri-
and is found in approximately 2.5% of adults.
SE

partum cardiomyopathy (New York Heart Association


c. A fixed splitting of the S2 and a right ventricular (RV) lift are
class II) after the birth of her second child presents
found in ASD with significant shunting.
with symptomatic paroxysmal atrial fibrillation (AF).
UR

d. Secundum ASD and PFO require subacute bacterial endo-


Typically, she experiences approximately 4 episodes of
carditis prophylaxis.
AF per year, each lasting 2 to 4 hours. During episodes,
CO

she notes palpitations, shortness of breath, and fatigue, 8. A 30-year-old woman comes to your office for evalua-
with heart rates around 90 beats per minute, but she tion. She appears healthy and is not limited by physi-
is able to continue working in her home. Although cal activity. She says that she has occasional chest pains
LL

she has generally done well from a heart failure (HF) that occur at rest, often with a change in position. Her
standpoint, she was hospitalized 3 weeks ago with blood pressure is 110/70 mm Hg, and her heart rate is
acute decompensated HF. Current medications include
FU

76 beats per minute and regular. Auscultatory findings


carvedilol 25 mg twice daily, lisinopril 20 mg daily, are a midsystolic click followed by a 2/6 apical systolic
furosemide 40 mg daily, and digoxin 0.125 mg daily. murmur that radiates to her axilla. Which of the fol-
Physical examination and diagnostic study findings lowing dynamic physical examination findings would
were as follows: heart rate 60 beats per minute (regular confirm your suspicion of mitral valve prolapse?
rhythm), blood pressure 100/70 mm Hg, jugular pres- a. There is no change in the click with squatting or standing.
sure not elevated, and lungs clear; electrocardiography b. A passive leg raise makes the click-murmur occur earlier.
showed normal sinus rhythm and left bundle branch c. A squat maneuver makes the click-murmur occur later.
block. Which of the following would you recommend d. The mitral regurgitant murmur does not change with posi-
at this time? tional maneuvers.

2 . C A R D I O L O GY Q U E S T I O N S A N D A N S W E R S • 11
9. A 25-year-old man, who is slender and athletic, comes C O N G E S T I VE H E A RT FA I LU R E
to your office because of an episode of syncope imme-
diately after playing basketball. He exercises diligently 13. A 72-year-old man presents with progressively increas-
but says that he has experienced mild dyspnea on exer- ing dyspnea and edema of the lower extremities. He
tion over the past 2 months. On physical examination, denies having chest discomfort or palpitations. He

0
his blood pressure is 175/90 mm Hg, his carotid pulse experiences dyspnea on minimal exertion, orthopnea,

1
is bounding, and there is an early systolic click imme- and occasional episodes of paroxysmal nocturnal dysp-

59
diately followed by a late peaking 3/6 systolic murmur nea. He had a myocardial infarction 10 years ago and
heard best at the right second intercostal space and underwent primary percutaneous coronary interven-

65
radiating to the carotids. There is no dynamic increase tion then. His residual left ventricular ejection fraction
in murmur intensity with the Valsalva maneuver and (LVEF) after that event was 30% by echocardiography.

48
no change in the timing of the click with squatting. He has adhered to his medical regimen. He has been
The apical impulse is sustained. A radial femoral delay hospitalized for decompensated heart failure repeat-

88
is suspected on examination. What should be the next edly over the past 3 years. He received an implantable
step in the evaluation or management of this patient? cardioverter-defibrillator with cardiac resynchroniza-

91
a. Coronary angiogram tion therapy 2 years before the current presentation.
b. Use of a 24-hour Holter monitor He has a history of hypertension and hyperlipidemia.

00
c. Computed tomographic (CT) scan of the chest His medical regimen consists of enalapril 10 mg twice
d. Reassurance with recommendations for warm-up and daily, carvedilol 6.25 mg twice daily, simvastatin 20 mg

AP
cooldown routines with exertion daily, aspirin 81 mg daily, spironolactone 25 mg daily,
e. Use of a 24-hour blood pressure monitor and furosemide 80 mg daily.
Examination findings were as follows: afebrile, heart

TS
10. A 65-year-old patient with a history of coronary artery rate 62 beats per minute, blood pressure 90/50 mm Hg,
bypass graft surgery 1 year ago presents with fatigue, significantly elevated jugular venous pressures with
HA
dyspnea, and progressive lower extremity edema. prominent v waves, lungs clear, a 3/6 pansystolic murmur
Examination reveals a blood pressure of 120/70 mm at the apex, a third heart sound (S3), and pitting edema
Hg and a pulse of 77 beats per minute. The lungs are
W

(3+). Laboratory test results were as follows: hemoglo-


clear. The heart is quiet, with normal first and second bin 10 g/dL, sodium 126 mEq/L, potassium 4.5 mEq/L,
heart sounds and no murmurs. The jugular venous pres- serum urea nitrogen 42 mg/dL, and creatinine 1.6 mg/
AL

sure ( JVP) is elevated at midneck (approximately 20 cm dL. The electrocardiogram showed a paced rhythm. A
water). There is an increase in the JVP with inspiration recent exercise stress test done with sestamibi imaging
I

with a rapid descent. What does this combination of


ER

showed a large fixed defect in the anteroseptal wall with


findings strongly suggest? no evidence of reversible ischemia and an LVEF of 20%.
a. Heart failure due to biventricular dysfunction The patient was admitted to the hospital for intravenous
AT

b. Superior vena cava syndrome diuretic therapy. The edema improved significantly, but
c. Constrictive pericarditis he continued to experience dyspnea on minimal exertion.
M

d. Failure of the bypass grafts and recurrent coronary artery His creatinine increased to 2.0 mg/dL. What should be
disease the next step in this patient’s management?
SE

11. A patient comes to you for evaluation of peripheral a. Increase the carvedilol dosage to 12.5 mg twice daily.
edema and palpitations. On examination, a loud grade b. Optimize cardiac resynchronization therapy.
c. Add metolazone 5 mg daily.
UR

3/6 holosystolic murmur is heard at the left lower ster-


nal border, and an intermittent third heart sound (S3) d. Perform a positron emission tomographic viability scan.
is heard that seems to be present only with inspiration. e. Refer for implantation of a left ventricular assist device
CO

A large v wave is noted in the jugular venous pressure (LVAD).


( JVP). What is your diagnosis?
a. Superior vena cava syndrome 14. Which statement is false regarding serum brain natri-
uretic peptide (BNP) levels?
LL

b. Atrial fibrillation
c. Constrictive pericarditis a. Serum BNP levels may be in the reference range in patients
FU

d. Significant pulmonary stenosis with advanced heart failure.


e. Significant tricuspid regurgitation b. A persistently elevated serum BNP level despite optimal
medical therapy is a marker of poor prognosis in heart
12. Which of the following is true about the ausculta- failure.
tory finding of a fourth heart sound (S4) on physical c. Optimization of medical therapy based on serum BNP lev-
examination? els decreases hospitalizations in heart failure.
a. Often associated with hypertension d. Serum N-terminal pro-BNP has a longer half-life than
b. Normal in young children serum BNP.
c. Normal in athletes e. The serum BNP level is frequently elevated in idiopathic
d. A hallmark of the onset of atrial fibrillation constrictive pericarditis.

12 • M AYO C L I N I C I N T E R N A L M E D I C I N E B OA R D R E VI EW: Q U E S T I O N S A N D A N S W E R S
0
ANSWER S

1
59
65
48
1. Answer b. be permanent pacemaker implantation. The other causes of
syncope are most unlikely in this patient.
The ECG is typical for Wolff-Parkinson-White syndrome.

88
Since the patient is asymptomatic, there is no indication 4. Answer d.
for drug therapy or an electrophysiology study unless he

91
is in a high-risk occupation (eg, pilot, military member). CRT is indicated for patients who have LVEF less than 35%,
With the onset of symptoms (palpitations, class I indica- LBBB with QRS duration of more than 120 ms, and conges-

00
tion), the preferred approach would be an electrophysiol- tive heart failure in NYHA class III or IV despite receiving
ogy study and catheter ablation (>95% success rate with a the maximally tolerated medical therapy. The most appro-
priate management is to up-titrate the dosages of lisinopril

AP
low risk of complications).
and carvedilol now and reevaluate the patient over the next
2. Answer a. 3 to 6 months to determine whether CRT is indicated.

TS
The clinical decision on how best to treat the patient’s 5. Answer d.
atrial fibrillation (rate control or rhythm control) should
HA
be based on symptom burden and adequacy of rate control, The patient is well compensated from an HF standpoint.
independently of stroke risk, as determined by calculating Her episodes of AF are infrequent and not particularly long
her CHADS2 (congestive heart failure, hypertension, in duration. Therefore, use of a daily antiarrhythmic drug
W

age >75 years, diabetes mellitus, and previous stroke) risk (eg, sotalol) is not warranted (toxicity could occur over the
score. Her CHADS2 score is greater than 2. According to long term with limited benefit). Dronedarone is contrain-
AL

current guidelines, warfarin is indicated. dicated in patients with HF or recent decompensation of


HF. (See Køber et al in the “Suggested Reading” list.)
3. Answer a.
I
ER

6. Answer d.
The symptoms and presentation are consistent with car-
diogenic syncope and conduction system disease (there is This patient is asymptomatic and presents with bradycardia
AT

electrocardiographic evidence of high-grade atrioventricu- that may be readily explained by her athletic conditioning.
lar block and left bundle branch block). Treatment would Her murmur is systolic, soft (less than grade 2), and heard
M

Presence of cardiac murmur


SE
UR

Systolic murmur Diastolic or continuous murmur


CO

Grade 3 or higher
Grade 1 or 2 &
holosystolic or late
midsystolic
systolic
LL
FU

Asymptomatic Other signs or


& no associated symptoms of Echocardiography
findings cardiac disease

Catheterization &
No further workup
angiography if required

Figure 2.A6. (Adapted from Bonow RO, Carabello B, De Leon AC Jr, Edmunds LH Jr, Fedderly BJ, Freed MD, et al. ACC/AHA guidelines for the
management of patients with valvular heart disease: a report of the American College of Cardiology/American Heart Association. Task Force on Practice
Guidelines [Committee on Management of Patients with Valvular Heart Disease]. J Am Coll Cardiol. 1998 Nov;32[5]:1486–588. Used with permission.)

18
in the pulmonary position, and it disappears with the is not indicated in ASD or PFO. Chest radiography may
Valsalva maneuver. If you were worried about hypertrophic be helpful. Classically, with large shunts radiography shows
cardiomyopathy, the response to this maneuver fairly well enlargement of the right atrium (bulging of the right heart
argues against it. There are no associated systolic clicks of border), RV (decreased retrosternal clear space), and pul-
mitral valve prolapse or bicuspid aortic valve, no diastolic monary artery and increased pulmonary vascular markings.

0
snaps to suggest mitral stenosis, and the electrocardiogram Common anatomical types of ASD are ostium secundum

1
is normal with the exception of the slow heart rate. Thus, ASD, ostium primum ASD, and sinus venosus ASD. Sinus

59
the murmur is most likely a benign flow murmur due to a venosus ASD is commonly accompanied by anomalous
thin chest wall, and you are likely hearing pulmonary out- pulmonary venous return. Ostium primum ASD is com-

65
flow, anatomically located just below the left upper sternal monly associated with cleft mitral valve and other complex
border. An S3 is a normal finding in a young athlete. There congenital heart diseases. (See Levin et al, O’Toole et al,

48
is no need for further testing (Figure 2.A6). and Wilson et al in the “Suggested Reading” list.)

88
7. Answer c. 8. Answer c.
Several clinical findings in ASD provide clues to the condi- This 30-year-old patient presents with atypical chest pain,

91
tion: 1) RV lift or sternal lift; 2) palpable pulmonary artery which may be associated with mitral valve prolapse. Mitral
pulse consistent with pulmonary artery volume and pres- valve prolapse is essentially a mismatch of the left ven-

00
sure overload of the RV (associated with significant shunt- tricular cavity size and the mitral valve leaflets, which are
ing); 3) fixed splitting of the S2 (the hallmark of ASD); 4) redundant. The classic bedside maneuver is the squat-to-

AP
accentuation of the tricuspid valve closure leading to split- stand maneuver; however, any maneuver that increases the
ting of the first heart sound; 5) midsystolic pulmonary ejec- left ventricular cavity size will delay midsystolic clicks and
tion murmur due to increased volume of flow through the the mitral regurgitant murmur (Figure 2.A8). The Valsalva

TS
RV outflow tract; and 6) if a large shunt is present, a mid- to maneuver decreases venous return, which would result in
late-diastolic rumble may also be appreciated. PFO is usu- a smaller left ventricle and thus an earlier mitral valve pro-
HA
ally not associated with any significant physical examination lapse. A postextrasystolic beat (due to the compensatory
findings, and it is a relatively common finding in autopsy pause), the passive leg raise, and squat (all would improve
W

studies; on average, PFO is seen in 25% of otherwise left ventricular filling) should actually delay midsystolic
healthy people. Subacute bacterial endocarditis prophylaxis clicks and murmur.
AL

Supine S1
S2
I

C
ER
AT
M
SE

Standing S1
UR

S2
C
CO
LL
FU

Squatting S1
S2
C

Figure 2.A8. C indicates click; S1, first heart sound; S2, second heart sound. (Adapted from Shaver JA, Leonard JJ, Leon DF. Examination of the heart.
Part 4: Auscultation of the heart. Dallas [TX]: American Heart Association; c1990. p. 13. Used with permission.)

2 . C A R D I O L O GY Q U E S T I O N S A N D A N S W E R S • 19
9. Answer c. 13. Answer e.
This patient, with a systolic click murmur in the absence The patient presents with stage D congestive heart fail-
of dynamic change, likely has bicuspid aortic valve (AV) ure. He has several poor prognostic indicators, including
stenosis. This diagnosis is more commonly associated with recurrent hospitalizations for heart failure exacerbations

0
a coarctation of the aorta whose flow murmur is best heard despite optimal medical therapy, low blood pressure, ane-

1
over the posterior aspect between the shoulder blades, mia, renal insufficiency, and a decreasing LVEF. His blood

59
slightly to the left of the midline. In the presence of hyper- pressure would not allow an increase in β-blocker dose.
tension and radial-femoral delay, there is enough evidence Optimization of cardiac resynchronization therapy is

65
on examination to support imaging of the aorta to rule out unlikely to help patients with stage D heart failure and a
coarctation. A CT scan (or magnetic resonance imaging) low output state. Adding a thiazide diuretic when edema

48
of the chest would show the coarctation. Another appro- has improved and renal insufficiency has worsened would
priate test would be transthoracic echocardiography to not be an appropriate next step. The patient had a remote

88
confirm the diagnosis of a bicuspid AV and to assess the history of myocardial infarction and had no active angina
degree of aortic stenosis. Echocardiography is often used to pectoris; hence, determining myocardial viability with a

91
view the descending thoracic aorta to determine the degree view toward revascularization is unlikely to provide much
of coarctation; however, if the aorta is not well visualized benefit toward improving the patient’s symptoms of heart

00
by echocardiography, other imaging is indicated to rule failure or survival. Patients with dilated cardiomyopathy,
out the coarctation. A coronary angiogram is unlikely to an LVEF of 25% or less, and New York Heart Association
be helpful unless it is coupled with an aortogram, which (NYHA) class IV symptoms should be considered for

AP
might provide the diagnosis, but there may be an increased LVAD implantation after optimization of medical and
danger due to the potential for coarctation, which could be electrical therapy. LVADs have been shown to prolong sur-

TS
difficult to cross with the catheter. This patient most likely vival, improve NYHA functional class, and improve qual-
does not have coronary artery disease. The murmur and ity of life.
HA
clinical presentation are not benign, and a thorough evalu-
ation is warranted. This patient’s syncopal event is related 14. Answer e.
to aortic stenosis or coarctation with significant stenosis Although serum BNP levels correlate with clinical sever-
W

and the inability to adequately increase cardiac output, ity, some patients with advanced heart failure have normal
especially with postexercise vasodilatation. BNP levels. Several large clinical trials have shown that
AL

a persistently elevated BNP level despite ongoing medi-


10. Answer c.
cal therapy is a marker of poor prognosis. A BNP-guided
I

Constrictive pericarditis is a relatively infrequent compli- approach to titrate medical therapy has been shown to
ER

cation of open heart surgery, yet cardiac surgery is now the decrease hospitalizations compared with standard therapy,
number one cause of constrictive pericarditis causing heart but the benefit may result from more intense medical sur-
AT

failure. The heart failure often begins slowly, with symptoms veillance and subsequent up-titration of medical therapy;
of dyspnea, peripheral edema, and signs of elevated neck veins therefore, its routine use for this purpose cannot be rec-
ommended. Although the serum BNP level is frequently
M

and the Kussmaul sign—elevation of JVP with inspiration.


Another finding often reported on the JVP is rapid descent. elevated in constrictive pericarditis due to cardiac surgery
or radiotherapy, it is frequently within the normal range in
SE

11. Answer e. patients with idiopathic constrictive pericarditis.


The presence of a prominent v wave in the JVP profile is
UR

15. Answer c.
consistent with significant tricuspid regurgitation. The
right-sided S3 may be intermittent and more prominent The patient has severe heart failure with preserved ejection
CO

with the inspiratory phase of respiration. Mitral regurgita- fraction. The cause of heart failure in this patient is cardiac
tion will produce a v wave into the pulmonary circulation, amyloidosis, which is consistent with the low voltage on
which can be seen with Swan-Ganz catheter tracings, but the the electrocardiogram and the thickened left ventricular
v wave is generally not transmitted to the JVP. Constrictive walls with a classic speckled pattern on the echocardio-
LL

pericarditis has very rapid y descents. Pulmonary stenosis is gram. The concomitant presence of peripheral neuropathy
associated with a giant a wave due to increased atrial con- is consistent with a diagnosis of familial amyloidosis. In
FU

traction caused by right ventricular pressure overload. In familial amyloidosis, the source of the amyloidogenic vari-
superior vena cava (SVC) syndrome, the pulsatile waves are ant of the transthyretin protein is the liver. Hence, the best
often lost because of extra-cardiac obstruction of the SVC. treatment option for this patient is combined heart and
liver transplant.
12. Answer a.
16. Answer c.
An S4 indicates increased left ventricular stiffness and is
commonly heard in adults with hypertension. It is caused The patient has dilated cardiomyopathy. The most com-
by the increased filling in late diastole during atrial contrac- mon cause of dilated cardiomyopathy, especially in a person
tion. An S4 is never normal. with atherosclerotic risk factors, is coronary artery disease.

20 • M AYO C L I N I C I N T E R N A L M E D I C I N E B OA R D R E VI EW: Q U E S T I O N S A N D A N S W E R S
FU
LL
CO
UR
SE
M
AT
ER
IAL
W
HA
TS
This page intentionally left blank

AP
00
91
88
48
65
59
1 0
3.
GASTROENTEROLOGY AND HEPATOLOGY

0
QUESTIONS AND ANSWER S

1
59
65
48
QUESTIONS a. Oral prednisone
b. Bismuth subsalicylate

88
Multiple Choice (choose the best answer) c. Budesonide
d. Ciprofloxacin

91
e. Metronidazole
C O L O N A N D PA N C R E A S

00
4. A 55-year-old woman presented to the emergency
1. A 52-year-old man presents for colon cancer screening.
department with a 6-hour history of epigastric pain,
He is asymptomatic, and he states that he is not aware of
nausea, and vomiting. She has a history of hyperten-

AP
any family member with a prior history of either colon
sion and hyperlipidemia. Her medications are aspirin
cancer or colon polyps. The patient undergoes colonos-
81 mg daily, lisinopril 10 mg daily, and simvastatin
copy and is found to have the following polyps and histo-

TS
20 mg daily. Her heart rate is 105 beats per minute,
logic diagnoses: 2-mm cecal polyp and 4-mm transverse
her blood pressure is 100/60 mm Hg, and her tem-
polyp (tubular adenomas, low-grade dysplasia), 6-mm
perature is 36.4°C. On clinical examination, the patient
HA
sigmoid polyp (tubulovillous adenoma, low-grade dys-
has moderate epigastric tenderness and reduced bowel
plasia), and 3-mm rectal polyp (hyperplasia). If those 4
sounds. Laboratory test results include the following:
lesions were completely removed, what should you rec-
W

hemoglobin 13 g/dL, white blood cell count 18×109/L,


ommend for ongoing colon surveillance?
amylase 2,563 U/L, lipase 5,637 U/L, aspartate amin-
a. Colonoscopy in 1 year
AL

otransferase (AST) 350 U/L, alanine aminotransferase


b. Colonoscopy in 3 years
(ALT) 250 U/L, and bilirubin 1.1 mg/dL. What should
c. Colonoscopy in 5 years
I

you recommend next?


ER

d. Yearly fecal occult blood test and colonoscopy in 5 years


a. Computed tomographic (CT) scan of the abdomen with
e. Computed tomographic colonography in 5 years
intravenous (IV) contrast medium
2. A 35-year-old woman with ulcerative colitis of 5 years’
AT

b. Ultrasonography of the abdomen


duration presents with a red eye. She has no pain or c. Emergent endoscopic retrograde cholangiopancreatography
headache, and her vision is normal. Her stools have (ERCP)
M

been somewhat looser over recent weeks. She has been d. Plain abdominal radiography
taking mesalamine at her usual dosage of 2.4 g daily, e. IV fluid, bowel rest, and observation
SE

but she has missed doses occasionally. She recently quit


5. A 50-year-old man presents to the emergency depart-
smoking. What is the most likely cause of her red eye?
ment with left lower abdominal pain. He has not had
UR

a. Uveitis
fever or a change in bowel habit. He is eating with-
b. Giant cell arteritis
out difficulty. He has never had similar symptoms in
c. Episcleritis
CO

the past. He has not undergone colon cancer screen-


d. Nicotine withdrawal and insomnia
ing. He has no comorbid conditions. On examina-
e. Viral conjunctivitis
tion, he has mild tenderness in the left lower abdomen
3. A 60-year-old woman presents with a 6-week history of without peritoneal signs. The white blood cell count
LL

diarrhea. She has had 6 to 8 large-volume watery stools is 12.5×10 9/L. Computed tomography (CT) shows
daily. There has been no blood in the stool. She has not changes consistent with diverticulitis without abscess.
FU

had fever or other systemic symptoms. Recently, she has What should you recommend as the next step for this
not taken antibiotics or changed medication. Clinical patient?
examination findings were normal. Laboratory test a. Hospital admission, bowel rest, and intravenous antibiotics
results were unremarkable. Stool examination for fecal b. Outpatient antibiotics
leukocytes was negative. On colonoscopy, the colon c. Colonoscopy
appeared normal. On biopsy, a thickened subepithe- d. Surgical consultation
lial collagen band was apparent. What initial therapy e. CT colonography
should you recommend?

27
0
ANSWER S

1
59
65
48
1. Answer b. without difficulty, outpatient management may be pur-
sued. If there were evidence of a complication (eg, abscess),
Hyperplastic polyps do not infer an increased risk of colon

88
or if the patient could not tolerate oral intake, hospital-
cancer. Therefore, the rectal lesion is of no clinical significance.
ization would be necessary. Since this is the patient’s first
The patient had 3 clinically significant polyps. Although each

91
episode of diverticulitis, and it is uncomplicated, surgery is
lesion was smaller than 1 cm, the interval to the next colonos-
not indicated. Colonoscopy is contraindicated with acute
copy would be 3 years because there were 3 or more polyps

00
diverticulitis, but since this patient has not undergone
and because 1 of the lesions had a villous component.
colon cancer screening, it would be reasonable to perform
2. Answer c. colonoscopy 2 to 4 weeks after the acute symptoms have

AP
resolved.
A patient with inflammatory bowel disease who has a red
eye most likely has either episcleritis or uveitis. Uveitis is

TS
6. Answer d.
associated with pain in the eye. Episcleritis is typically pain-
less. Giant cell arteritis typically does not manifest with red This patient has iron deficiency anemia that is likely due to
HA
eye and is usually associated with headache or visual loss. the antral erosions that appear to have been induced by the
nonsteroidal anti-inflammatory drug. However, he also has
3. Answer b. a gastric polyp that is a gastric adenoma; similar to colonic
W

The clinical presentation and the colonoscopic and his- adenomas, gastric adenomas are deemed premalignant and
tologic findings are typical of microscopic colitis (in this require full endoscopic removal. The patient should have
AL

case, collagenous colitis). Initial therapy for mild to mod- another EGD now with polypectomy since the polyp was
erate disease (3–6 bowel movements daily) is usually with simply biopsied and not fully removed during his previ-
I

ous EGD. To do no further testing would be inadequate


ER

either an antidiarrheal, such as loperamide hydrochloride,


or bismuth subsalicylate. In more severe cases (>6 bowel because this polyp could continue to grow and progress
movements daily), budesonide may be considered for ini- to gastric cancer. While gastric erosions can be caused by
AT

tial therapy. Prednisone would be used only in cases of H pylori, this patient’s histologic examination was negative
microscopic colitis that did not respond to the aforemen- for H pylori, and he was not taking any medications that
could lead to false-negative testing for H pylori (proton
M

tioned therapies.
pump inhibitor, antibiotics, etc); therefore, further test-
4. Answer e. ing for H pylori would not be needed. If the gastric polyp
SE

The patient has acute pancreatitis as evidenced by the had been malignant, endoscopic ultrasonography would
clinical presentation and the elevated levels of amylase and be needed to assess the depth of invasion and to complete
UR

lipase. The cause is most likely gallstone disease (AST and locoregional staging, but that is not needed for an ade-
ALT were moderately elevated). The most important initial noma of this size. To wait to repeat the EGD for 1 year is
not recommended since the polyp could continue to grow,
CO

step in the management of patients with acute pancreatitis


is to ensure excellent hydration to optimize pancreatic per- progress to cancer, or cause bleeding, all of which could be
fusion and thereby decrease the risk of pancreatic necrosis. prevented by removal now.
LL

The patient does not have evidence of cholangitis (absence


7. Answer c.
of fever, pain localizing to the right upper quadrant, and
FU

jaundice), so there is no indication for emergent ERCP. This patient has features suggestive of gastroesophageal
CT scan of the abdomen with IV contrast medium may be disease. He has not only progressive heartburn but also the
indicated later in the clinical course to evaluate for pancre- extra-esophageal feature of hoarseness. Since he has not
atic necrosis, but it is not indicated at initial presentation. fully improved with PPI therapy and the EGD was negative
Ultrasonography of the abdomen is a reasonable step but for esophagitis, a 24-hour ambulatory pH probe would be
only after IV fluid resuscitation has been initiated. helpful to document whether he truly has acid reflux and
whether there is symptom correlation. When this test incor-
5. Answer b.
porates impedance testing, symptoms can be correlated with
This patient is presenting with a first episode of uncom- acid-mediated reflux or with non–acid-mediated reflux (eg,
plicated diverticulitis. Since he is tolerating oral intake seen with someone receiving PPI therapy). At this point,

31
4.
PULMONARY DISEASES QUESTIONS AND ANSWER S

1 0
59
65
QUESTIONS a. Haloperidol

48
b. Lorazepam
Multiple Choice (choose the best answer) c. Hydromorphone

88
d. Midazolam
e. Fentanyl
CRITICAL CARE MEDICINE

91
3. A 58-year-old man comes to the emergency depart-
1. A 59-year-old man underwent a radical prostatectomy
ment with clumsiness of his right (dominant) hand that

00
for prostate cancer 24 hours ago. Since hospital admis-
began abruptly 3 hours earlier. He has hypertension,
sion, he has received a total of 3.5 L of intravenous iso-
for which he takes a β-blocker. His temperature is 37°C,
tonic crystalloid and has had a total urine output of 2.5 L.

AP
his heart rate is 90 beats per minute, his respirations are
The hospital physician on duty examined him earlier
16 per minute, and his blood pressure is 145/90 mm Hg.
this evening for evaluation of substernal chest pain.
Oxygen saturation as measured by pulse oximetry is

TS
The patient’s electrocardiogram (ECG) was normal. He
97% with room air. Physical examination reveals an
received intravenous furosemide and oral nitroglycerin
intact sensorium, no obvious cranial nerve deficits, 3/5
and had 1.0 L of urine output. He became symptomati-
HA
motor strength in the right upper arm with inability
cally worse, so he was transferred to the intensive care
to perform repetitive hand movements (ataxia), and
unit for further evaluation. When you initially examine
4/5 motor strength in the right lower leg. No sensory
W

the patient, his blood pressure is 150/100 mm Hg, his


deficits are noted. Emergent computed tomography of
heart rate is 115 beats per minute, his respiration rate
the head does not show a stroke. All screening labora-
AL

is 24 breaths per minute, and his temperature is 37°C.


tory study results are normal except for a serum choles-
He reports chest pain and shortness of breath. There is
terol level of 250 mg/dL. It is now 4 hours after onset
no jugular vein distention. You hear bibasilar crackles
I

of symptoms. Which of the following is the definitive


ER

posteriorly and a harsh systolic murmur across the pre-


treatment?
cordium. His extremities are well perfused with good
a. Alteplase
pulses. An ECG shows sinus tachycardia and delayed
AT

b. Aspirin
precordial transition, but no other changes are noted.
c. Clopidogrel
While you wait for results of additional diagnostic
M

d. Continuous-infusion eptifibatide
studies, which of the following should you administer
e. Continuous-infusion heparin
now?
SE

a. Furosemide
4. A 52-year-old man is admitted to the hospital because
b. Metoprolol
of acute dyspnea and a syncopal episode during which
UR

c. Nitroglycerin
he fell. He is hypotensive and requires intubation and
d. Aspirin
norepinephrine to maintain adequate blood pressure
e. Heparin
and oxygenation. A computed tomographic (CT) scan
CO

2. A 32-year-old man underwent surgical repair and fixa- of the chest shows a saddle pulmonary embolism, and
tion of multiple traumatic orthopedic injuries 3 days the patient is administered tissue plasminogen activa-
ago. He also has bilateral lung contusions and has tor (tPA) in the emergency department. An echocar-
LL

required mechanical ventilation. He did not have any diogram shows right ventricular dilatation and failure.
head injuries. His condition is improving, and physi- Ten hours later, the norepinephrine dosage has been
FU

ologically he is ready to begin being weaned from the incrementally increased from 25 to 55 mcg per min-
ventilator. His oxygenation is good. However, when the ute. Oxygen saturation as measured by pulse oximetry
propofol infusion is decreased, he manifests a severe, is 98% with a fraction of inspired oxygen of 0.80. His
agitated delirium with ventilator dyssynchrony and an heart rate is 125 beats per minute, his blood pressure is
inability to follow commands. Neurologic examination 100/50 mm Hg, and his central venous pressure is 1 to
findings are otherwise normal. Laboratory test results 2 mm Hg. The hemoglobin level has decreased from
are normal, computed tomography of the head is nor- 13 g/dL on admission to 5 g/dL. Activated partial
mal, and the electrocardiogram is normal. Which of the thromboplastin time is 65 seconds. A formal reading
following should you order now? of the admission CT scan shows left rib fractures in

35
addition to pulmonary embolism. A subsequent chest D I FF US E LU N G D I S E A S E A N D O C CU PAT I O NA L
radiograph is clear. A subsequent echocardiogram LU N G D I S E A S E
shows a poorly contracting right ventricle. Gastric aspi-
8. A 25-year-old female nonsmoker presents with a 1-week
rate is clear, and the stool is negative for occult blood. In
history of mild cough and dyspnea after a flulike illness
addition to transfusion, which of the following should

0
with fever, arthralgias, and tender erythematous lesions
you recommend?

1
on the anterior aspects of the legs. She has no history
a. Perform a CT scan of the abdomen.

59
of asthma or significant medical illnesses. No environ-
b. Repeat the dose of tPA.
mental or occupational high-risk exposures are noted.
c. Stop the heparin and place an inferior vena cava (IVC)

65
Examination reveals clear lung fields and no other
filter.
abnormalities. A chest radiograph shows prominent
d. Continue heparin and place an IVC filter.

48
bilateral hilar lymphadenopathy without parenchymal
e. Perform a surgical embolectomy.
infiltrates. What should you do next?

88
5. A 25-year-old man is admitted to the intensive care unit a. Set up blood cultures.
(ICU) with decelerating injury after a motor vehicle col- b. Perform human immunodeficiency virus serology testing.

91
lision. The patient required on-scene mechanical extrica- c. Determine the erythrocyte sedimentation rate.
tion from the vehicle. He has bilateral lower extremity d. Perform Lyme serology testing.

00
fractures that required surgical intervention before he e. Observe and repeat the chest radiograph in 12 weeks.
arrived in the ICU. Thus far, he has received 10 L of crys-
9. A 62-year-old man, a former smoker, presents with a
talloid and 2 units of packed red blood cells. Over the

AP
2-year history of progressive dry cough and dyspnea.
first 4 hours, his blood pressure and urine output decrease
He has no extrapulmonary symptoms. No occupational
and partially respond to an additional 4 L of crystal-
or environmental exposures are noted. Findings on

TS
loid. Currently, his blood pressure is 80/50 mm Hg, his
examination include bibasilar coarse rales and digital
heart rate is 110 beats per minute, his respiratory rate is
clubbing. A chest radiograph shows prominent inter-
HA
18 breaths per minute, and he is normothermic. Other
stitial infiltrates in the middle and lower lung fields.
than his lower extremity injuries, no abnormal findings
The antinuclear antibody titer is borderline elevated
are noted on his examination. His hemoglobin is 9 g/dL
W

at 1:40. Serum protein electrophoresis shows a poly-


and his coagulation values are normal. His total creatine
clonal gammopathy. The rheumatoid factor titer is also
kinase (CK) is 800 U/L. A computed tomographic scan
borderline elevated at 1:40. A high-resolution com-
AL

of the abdomen from the emergency department is nor-


puted tomographic (CT) scan of the chest shows sub-
mal. What should you do next?
pleural honeycombing with thickened alveolar septa
I

a. Administer a colloid fluid bolus.


ER

in both lower lobes with bilateral mediastinal 1.5-cm


b. Obtain an echocardiogram.
lymph nodes. No ground-glass opacities are present.
c. Administer methylprednisolone.
Which treatment is most likely to result in clinical
AT

d. Perform a focused assessment with sonography for trauma


improvement?
(FAST).
a. Azathioprine
e. Begin a bicarbonate infusion.
M

b. Systemic corticosteroids
6. The incidence of transfusion-related acute lung injury c. Cyclophosphamide
SE

(TRALI) is greatest with transfusion of which of the d. Systemic corticosteroids with azathioprine
following? e. No treatment
a. Fresh frozen plasma
UR

10. A 52-year-old man, a current smoker (75 pack-years),


b. Packed red blood cells
is examined for acute dyspnea and right-sided chest
c. Pooled platelets
pain. He denies having fever, chills, sweats, cough,
CO

d. Single-donor platelets
sputum production, or hemoptysis. On auscultation
e. Salt-poor albumin
of the lungs, diminished breath sounds are heard
7. Which of the following best describes the effects of throughout, with more on the right than the left. The
LL

critical illness on physiologic sleep? chest radiograph and computed tomographic scan
a. Opiate-benzodiazepine combinations promote physiologic of the chest show scattered interstitial changes with
FU

sleep in patients receiving mechanical ventilatory support. cystic and nodular abnormalities, which are more
b. In critical illness, the proportion of rapid eye movement prominent in the mid and upper lung zones, and a
(REM) sleep is decreased. right-sided pneumothorax. What is the most likely
c. In critical illness, sleep has predominant waveforms consis- diagnosis?
tent with deep sleep. a. Lymphangioleiomyomatosis
d. The total duration of sleep during a 24-hour period is b. Pulmonary Langerhans cell histiocytosis (histiocytosis X)
increased. c. Cystic fibrosis
e. The total duration of sleep during a 24-hour period is mark- d. Aspiration pneumonia
edly decreased. e. Idiopathic pulmonary fibrosis

36 • M AYO C L I N I C I N T E R N A L M E D I C I N E B OA R D R E VI EW: Q U E S T I O N S A N D A N S W E R S
5.
INFECTIOUS DISEASES QUESTIONS AND ANSWER S

1 0
59
65
QUESTIONS no clubbing or cyanosis. The total leukocyte count is

48
14.4×109/L. Results of serum chemistry tests are nor-
Multiple Choice (choose the best answer) mal. The chest radiograph shows a new dense consolida-

88
tion in the left upper lung field. Which of the following
antimicrobial regimens would you initiate?
P N EU MO N I A , Z O O N O S E S , T R AVE L , A N D

91
a. Azithromycin orally
B I OT E R RO R I S M b. Ceftriaxone intravenously and azithromycin orally

00
c. Ciprofloxacin intravenously
1. A 67-year-old man with uncomplicated type 2 diabe-
d. Ampicillin intravenously and clindamycin orally
tes mellitus and hypertension presents to his primary
e. Meropenem intravenously

AP
care physician with a 3-day history of fever, productive
cough, and shortness of breath. He has been in good 3. A 44-year-old male business executive is admitted to
health otherwise, has never been hospitalized, and the intensive care unit for fevers, diarrhea, cough, and

TS
has good social support at home. Examination find- shortness of breath progressing over the past 48 hours.
ings include the following: temperature 39.1°C, blood His past medical history includes cigarette smok-
HA
pressure 110/75 mm Hg, heart rate 78 beats per min- ing and weekly binge drinking but no other chronic
ute with regular rate and rhythm, and respiratory rate medical problems. He is from St Louis, Missouri, but
26 breaths per minute. Oxygen saturation is 94% with was attending a conference in Chicago, Illinois, 2 days
W

room air. He is awake and alert and oriented to person, before admission. Abdominal pain, diarrhea, cough,
place, and time; he responds to questions appropriately. and shortness of breath developed 2 days before admis-
AL

Inspiratory crackles are audible in the right lower lung sion. He had mentioned that other people from the
field, his abdomen has active bowel sounds and is not convention had similar illnesses. He has had no other
I

tender, and his extremities have no clubbing or cyano- recent travel and has not had any exposure to animals.
ER

sis. Which of the following should be done next? Antimicrobial therapy has been started and blood
a. Outpatient observation only with follow-up in 2 days and sputum cultures are pending. Examination find-
AT

b. Outpatient consultation with an infectious diseases ings include the following: temperature 39.2°C, blood
specialist pressure 85/60 mm Hg, heart rate 72 beats per min-
M

c. Hospital admission and treatment with levofloxacin ute with regular rate and rhythm, and respiratory rate
d. Intensive care unit admission and treatment with 40 breaths per minute. Oxygen saturation is 90% with
SE

levofloxacin 10 L of oxygen per minute by face mask. He is drowsy and


e. Outpatient treatment with levofloxacin does not reliably follow commands, and he is oriented
to person only. Diffuse bilateral crackles are audible on
UR

2. A 57-year-old woman is admitted to an internal medi-


lung auscultation, his abdomen has active bowel sounds
cine hospital ward with a 3-day history of fever, cough,
and is not tender, and his extremities have no club-
progressively purulent sputum, and shortness of breath.
CO

bing or cyanosis. Remarkable diagnostic test results


She has been well otherwise, has never been hospital-
were leukocytosis (16.2×109/L) and hyponatremia
ized, and has a past medical history of well-controlled
(126 mEq/L). A chest radiograph showed bilateral patchy
type 2 diabetes mellitus and hypertension. She has not
infiltrates. Gram staining of a sputum showed scant poly-
LL

had any recent antimicrobial exposure, foreign travel,


morphonuclear cells and no bacteria. What is the most
or animal exposure. She has no known drug allergies.
likely cause of the patient’s respiratory infection?
FU

Examination findings include the following: tempera-


a. Legionella pneumophila
ture 38.7°C, blood pressure 125/75 mm Hg, heart rate
b. Respiratory syncytial virus (RSV)
78 beats per minute with regular rate and rhythm, and
c. Enterovirus
respiratory rate 26 breaths per minute. Oxygen satu-
d. Chlamydophila psittaci
ration is 95% with 2 L of oxygen per minute by nasal
e. Coccidioides immitis
cannula. She is awake and alert and oriented to person,
place, and time. Inspiratory crackles are audible in the 4. Which of the following is a recommended intervention
left upper lung field, her abdomen has active bowel to reduce the risk of ventilator-associated pneumonia
sounds and is not tender, and her extremities have among patients receiving mechanical ventilation?

45
0
ANSWER S

1
59
65
48
1. Answer e. Legionella infections can progress rapidly and are often asso-
ciated with diarrhea and other gastrointestinal tract symp-
Initial assessment of the severity of community-acquired

88
toms. Alcohol abuse is a known epidemiologic risk factor,
pneumonia is important for internal medicine physicians to
and Legionella often causes outbreaks among persons with
reduce unnecessary hospitalization and to identify patients

91
common-source exposure. Relative bradycardia and mental
who are at higher risk of death or who need more immediate
status changes frequently occur in patients with Legionella
intervention. Illness severity scores such as the pneumonia

00
infection. Leukocytosis, hyponatremia, and patchy bilat-
severity index (PSI) and CURB-65 (confusion, urea nitro-
eral infiltrates occur frequently in Legionella infections,
gen, respiratory rate, blood pressure, and 65 years or older)
but those features may not be as helpful for discriminating

AP
have been developed to help in the decision for site of care
between other microbiologic causes of community-acquired
for patients with community-acquired pneumonia. In the
pneumonia. RSV is unlikely to cause respiratory illness of
outpatient setting, the modified CRB-65 (confusion, respi-

TS
this severity in an adult without immunocompromising
ratory rate, blood pressure, and 65 years or older) score is
conditions. Enterovirus is a cause of encephalitis but is not
useful since it does not require laboratory or radiographic
a cause of community-acquired pneumonia. When C psit-
HA
evidence to determine the severity score. In this question, the
taci causes atypical community-acquired pneumonia, it is
patient is not confused, does not have a respiratory rate of
generally not rapidly progressive or associated with relative
30 breaths per minute or more, does not have a systolic blood
W

bradycardia or gastrointestinal tract symptoms. The patient


pressure less than 90 mm Hg or a diastolic blood pressure of
has not been to a region where C immitis is endemic.
60 mm Hg or less, but is 65 years or older. His CRB-65 score
AL

is 1. Patients with scores of 0 or 1 can generally be treated as 4. Answer e.


outpatients if they can reliably take oral antimicrobials and
I

have outpatient support resources. (See Mandell et al and This question relates to knowledge of modifiable risk fac-
ER

Capelastegui et al in the “Suggested Reading” list.) tors to reduce the risk of ventilator-associated pneumo-
nia among patients receiving mechanical ventilation. The
2. Answer b.
AT

correct answer and incorrect answers (modified to be


This question is related to appropriate initial antimicrobial negative) were taken directly from the guideline recom-
mendations on health care–associated pneumonia. (See
M

treatment of community-acquired pneumonia in a patient


with medical comorbidities who requires hospitalization but American Thoracic Society and Infectious Diseases Society
of America in the “Suggested Reading” list.)
SE

not intensive care unit admission. Recommended treatment


is either a respiratory fluoroquinolone or a β-lactam antibiotic
in combination with a macrolide antibiotic. Azithromycin 5. Answer c.
UR

alone is not a recommended regimen in patients with medical This question is based on an understanding of the epidemi-
comorbidities, including diabetes mellitus. Ciprofloxacin does ology of hospital-acquired pneumonia and how it relates
CO

not have sufficient coverage for Streptococcus pneumoniae and is to the choice of empirical antimicrobials for its treatment.
thus not considered a respiratory fluoroquinolone. Although The hospital prevalence of methicillin-resistant S aureus
intravenous ampicillin would be an acceptable β-lactam anti- and multidrug-resistant gram-negative organisms, such as
biotic antimicrobial choice, clindamycin is not a macrolide P aeruginosa, A baumannii, and the enterobacteriaceae,
LL

antibiotic and the patient does not have risk factors to war- need to be considered when deciding on an antimicrobial
rant empirical coverage of methicillin-resistant Staphylococcus regimen. The only correct pair of choices is S aureus and
FU

aureus. Meropenem is a very broad-spectrum antimicrobial P aeruginosa. Enterococci and C albicans are respiratory
and is not recommended for routine use for hospitalized pathogens in only select hosts and are very rare causes of
patients with community-acquired pneumonia who do not hospital-acquired pneumonia. Although influenza virus,
have risk factors for infection with Pseudomonas aeruginosa. S pneumoniae, and H influenzae can cause hospital-acquired
(See Mandell et al in the “Suggested Reading” list.) pneumonia, they generally are involved in a relatively small
3. Answer a. proportion of nosocomial cases and their resistance pro-
files are generally those of the community-acquired strains.
This question involves recognition of Legionella as a cause (See American Thoracic Society and Infectious Diseases
of community-acquired pneumonia. In healthy hosts, Society of America in the “Suggested Reading” list.)

52
FU
LL
CO
UR
SE
M
AT
ER
IAL
W
HA
TS
This page intentionally left blank

AP
00
91
88
48
65
59
1 0
6.
RHEUMATOLOGY QUESTIONS AND ANSWER S

1 0
59
65
QUESTIONS becomes fatigued if she tries to comb her hair. She is

48
a nonsmoker and denies having any respiratory com-
Multiple Choice (choose the best answer) plaints. On examination, her temperature is 37.5°C.

88
Her right radial pulse is decreased compared with the
N O NA RT I CU L A R R H EUM AT I S M A N D
left, and the blood pressure in her right arm is decreased

91
VA S C U L I T I S
compared with the left. Laboratory studies show mild
normochromic anemia with a hemoglobin of 11.2 g/dL

00
1. A 72-year-old woman presents with complaints of (reference range >12.0 g/dL), mildly elevated erythro-
new-onset headache. She is fatigued and has lost about cyte sedimentation rate (ESR) at 36 mm/h (reference
2.3 kg over the past month. The patient describes daily range <29 mm/h), negative antineutrophil cytoplasmic

AP
bitemporal headaches. On further questioning, she autoantibody (ANCA) test, and normal blood chemis-
describes morning stiffness lasting about 1 hour and try panel results. Urinalysis and chest radiograph find-

TS
aching in the shoulders and hips. She reports transient ings are normal. Which of the following is the most
loss of vision in her right eye for about 30 minutes yes- likely diagnosis?
terday before her vision returned. Physical examination a. Polymyalgia rheumatica (PMR)
HA
findings are unremarkable for her age. Initial labora- b. Giant cell arteritis (GCA)
tory studies show a mild normochromic anemia and c. Buerger disease
W

elevated erythrocyte sedimentation rate (ESR) (101 d. Wegener granulomatosis


mm/h). Which would be the most appropriate next e. Takayasu arteritis
AL

step?
a. Request a temporal artery biopsy.
4. A 59-year-old man has a 1-week history of low back dis-
b. Prescribe prednisone 1 mg/kg orally daily.
comfort. He says that he has been moving furniture over
I

the past several weeks, but he does not recall a specific


ER

c. Prescribe prednisone 5 mg orally 3 times daily.


d. Request computed tomography (CT) of the head.
injury. The pain is worse if he is active; it improves if he
e. Refer the patient to a neurologist.
is at rest. He denies having pain radiating to the legs.
AT

The patient has been taking ibuprofen 400 mg twice


2. A 55-year-old man presents with a 3-week history of left daily with food, and this seems to help. He has no prior
knee pain. He does not recall an injury. He says that the
M

history of lower back pain. Neurologic examination


front of his leg is tender. On examination, he is afebrile findings are normal, with a downgoing Babinski sign,
and overweight. There is no left knee effusion, but the
SE

equal and symmetrical knee jerks, and normal strength


patient is markedly tender over the anterior medial tibia in the lower extremities. He has somewhat diffuse ten-
just distal to the knee joint. This area seems somewhat derness over the lumbar spine. Laboratory study results
UR

puff y but is not particularly erythematous. The patient are normal for the complete blood cell count, erythro-
has good range of motion in both hips, but the area over cyte sedimentation rate, and blood chemistry panel.
the left trochanteric bursa is tender. What would be the What would be the most appropriate recommendation
CO

most appropriate step for pain relief ? at this point?


a. Inject the left knee joint with corticosteroid and local a. Bed rest for 2 weeks
anesthetic. b. Radiograph of the lumbar spine
LL

b. Inject the left trochanteric bursa with corticosteroid and c. Electromyographic (EMG) study
local anesthetic. d. Neurologic consultation
FU

c. Inject the left pes anserine bursa with corticosteroid and e. HLA-B27 testing
local anesthetic.
d. Prescribe propoxyphene 65 mg orally every 6 hours.
5. A 26-year-old woman presents to your office because
e. Prescribe prednisone 60 mg orally daily for 7 days.
she aches all over. She tells you that this condition has
been present for several years but has gotten worse over
3. A 26-year-old woman presents for evaluation of the past 6 months. She has problems getting to sleep and
low-grade fevers that have been present about does not feel rested when she wakes up. She denies hav-
6 months. She also has lost 3.6 kg and has noted some ing depression. She is stiff for 5 minutes in the morning
arthralgias and myalgias. She says that her right arm and has not noticed any joint swelling. On examination,

59
7.
ENDOCRINOLOGY QUESTIONS AND ANSWER S

1 0
59
65
QUESTIONS 3. A 72-year-old man has new-onset atrial fibrillation and

48
abnormal results on thyroid tests (thyrotropin <0.01
Multiple Choice (choose the best answer) mIU/L, free thyroxine 2.3 ng/dL). He begins antiar-

88
rhythmic therapy and is referred to you. He reports palpi-
T H Y RO I D D I S E A S E tations and weight loss for 3 months along with redness,

91
1. A 62-year-old farmer is evaluated as a new patient. He swelling, and pain over his eyes. His past medical history
has a 20-year history of hypothyroidism and hypogo- is significant for hypertension, congestive heart failure,

00
nadism. He takes levothyroxine (88 mcg daily) and and ongoing nicotine dependence. On examination, his
testosterone gel 1% (2.5 g daily) and feels well. He heart rate is irregular at 92 beats per minute, his blood
weighs 75 kg and has a small thyroid. Physical exami- pressure is 148/86 mm Hg, and his lungs are clear. The

AP
nation findings are unremarkable. Laboratory test thyroid is nontender, about twice the normal size, and
results include the following: thyrotropin 0.2 mIU/L increased in consistency. He has bilateral exophthalmos

TS
(reference range, 0.3–5.0 mIU/L) and free thyroxine with red and significantly swollen eyelids and injected
1.3 ng/dL (reference range, 0.8–1.8 ng/dL). You decide conjunctiva. There is pitting edema (trace) of the lower
extremities and an area of brownish thickening of the
HA
to decrease the levothyroxine dosage to 75 mcg daily.
After 2 months, thyrotropin was 0.1 mIU/L, free thy- skin over the pretibial areas. What is the best manage-
roxine was 0.9 ng/dL, and he feels tired and cold. Which ment for his thyroid dysfunction at this point?
W

of the following should you do now? a. Observation, antiarrhythmic treatment, and reevaluation in
a. Decrease the levothyroxine dosage to 50 mcg daily and 6 weeks for possible hypothyroidism
AL

recheck in 6 weeks. b. Propylthiouracil (PTU)


b. Stop levothyroxine altogether and evaluate thyroid uptake c. Methimazole treatment
d. Total or near-total thyroidectomy
I

for hyperthyroidism.
ER

c. Discontinue the testosterone gel since it increases the e. Radioactive iodine treatment
thyroxine-binding globulins.
AT

d. Check his thyroglobulin to distinguish between primary D I A B ET E S M E L L IT US , H Y P O G LYC E M I A , A N D


and secondary hypothyroidism. HYPERLIPIDEMIA
e. Continue levothyroxine at 125 mcg daily, get his old records,
M

4. A 54-year-old woman comes to your office for her rou-


and consider cortisol testing.
tine 6-month follow-up for the management of type
SE

2. A 29-year-old woman has a 7-year history of primary 2 diabetes mellitus. She has no immediate concerns.
hypothyroidism due to Hashimoto thyroiditis. She has Over the past year, she has consulted a dietitian and has
been treated with 125 mcg of levothyroxine with a good been trying to adhere to her diabetic diet. She has also
UR

clinical and biochemical response so far. A few months started a walking program. Her medications include
ago, she decided to start taking oral contraceptives metformin 1,000 mg twice daily, simvastatin 20 mg
CO

because she had persistently irregular periods related daily, lisinopril 20 mg daily, a multivitamin daily, and
to polycystic ovary syndrome. Her periods are regular a calcium supplement daily. She monitors her blood
now, but she feels tired and has cold intolerance. Her glucose levels at home in the morning 2 or 3 times per
LL

thyrotropin is 10.8 mIU/L; free thyroxine is 1.1 ng/dL. week, and the metered glucose values (in milligrams per
Which of the following should you do next? deciliter) are in the mid 100s. She denies having hypo-
FU

a. Check the titer of thyroperoxidase antibodies to assess the glycemia. She works outside the home but eats meals
severity of Hashimoto thyroiditis. at regular times. On physical examination, her blood
b. Advise the patient to improve her adherence to the medical pressure is 135/80 mm Hg, her heart rate is 75 beats
therapy and recheck in 2 to 3 months. per minute, her height is 160 cm, her weight is 85 kg,
c. Advise the patient to take the oral contraceptive 12 hours and her body mass index is 33. Other examination find-
after taking levothyroxine, and recheck in 2 to 3 months. ings are unremarkable. Laboratory test results include
d. Add liothyronine to the levothyroxine for a better clinical hemoglobin A1c (HbA1c) 8.5% and normal serum crea-
response. tinine and liver transaminase values. What is your best
e. Increase the levothyroxine dose to 137 mcg, and recheck management option to improve this patient’s glycemic
thyrotropin in 2 to 3 months. control?

67
0
ANSWER S

1
59
65
48
1. Answer e. eye condition, lead to a transient increase in thyroid hor-
mone values, and not achieve euthyroidism for another 2
The replacement dose of levothyroxine is relatively low for

88
to 3 months. Surgery would be associated with additional
this patient’s weight (the usual dose is 1.6 mcg/kg body
risks because this patient has several cardiac comorbidities.
weight), and it would be unlikely to cause iatrogenic thyro-

91
The safest and most effective treatment for him would be
toxicosis. The free thyroxine level decreased with a decrease
antithyroid drug treatment. Methimazole would be the
in the levothyroxine dosage without a significant change in

00
first choice. PTU is a second-line drug because of its asso-
the thyrotropin level. The patient has symptoms of hypo-
ciation with severe liver failure and death, but PTU is still
thyroidism, and he has hypogonadism. His old records
preferred in the first trimester of pregnancy and during

AP
might explain why he has central hypothyroidism. If those
thyroid storm. (See Bahn et al and Bartalena et al in the
records are not available, testing his adrenal axis and imag-
“Suggested Reading” list.)
ing his pituitary would be the next step. For the same

TS
reason, decreasing the dosage to 50 mcg daily for a 75-kg 4. Answer c.
person or evaluating him for hyperthyroidism would not
HA
address his symptoms or biochemical response to the first Metformin and sulfonylurea medications have been exten-
change in dosage. Androgens actually decrease the level of sively studied as monotherapy and combination therapy
thyroxine-binding globulins. (See Daly et al and Singer et for type 2 diabetes mellitus. For the overweight or obese
W

al in the “Suggested Reading” list.) patient, metformin is the recommended first-line therapy
because of its insulin-sensitizing effects, low risk of hypo-
AL

2. Answer e. glycemia, and neutral impact on weight. Sulfonylurea


Increased estrogen levels, which were likely present in medications provide additional glucose lowering to the
I

this patient during oral contraceptive therapy, lead to patient who has inadequate glycemic control with optimal
ER

an increase in thyroxine-binding globulins, and that doses of metformin and a recognized risk of hypoglycemia
requires an increase in the levothyroxine replacement and weight gain. No additional glucose-lowering effect
AT

dose in hypothyroid patients to maintain the free thy- is observed with metformin doses higher than 2,000 mg
roxine levels unchanged. Thyroperoxidase antibodies are daily. The use of rosiglitazone is restricted because of recog-
nized risks of heart failure and cardiovascular events, and it
M

not pathogenic, and their titer does not correlate with


the replacement dose. The patient’s hypothyroidism had is not appropriate for this patient. (See Nathan et al in the
“Suggested Reading” list.)
SE

been well controlled with a stable dosage of levothyroxine


for several years, so that nonadherence is probably not a
5. Answer d.
problem. The estrogen action is sustained, is not related
UR

to the timing of the administration of levothyroxine This patient meets the body mass index (BMI) criterion
and oral contraceptive, and does not interfere with the for obesity (BMI >30) and is at risk for type 2 diabetes
CO

absorption of levothyroxine. The combination of levothy- mellitus over the next 5 years. Many patients are unaware
roxine and liothyronine has not been proved to be more of their risk for progression toward type 2 diabetes mel-
effective clinically than levothyroxine monotherapy, but litus. Interventions to delay progression to type 2 diabe-
it has been associated with significantly more iatrogenic tes mellitus should be discussed. Lifestyle changes with
LL

thyrotoxicosis in clinical trials; therefore, it is not recom- dietary modification, regular physical activity, and mod-
mended. (See Ain et al and Sawka et al in the “Suggested est weight loss can substantially lower his risks for pro-
FU

Reading” list.) gressing to type 2 diabetes mellitus. Several medications


have been studied in type 2 diabetes mellitus prevention,
3. Answer c.
including metformin, rosiglitazone, acarbose, and orlistat.
This patient has thyrotoxicosis with Graves ophthalmopa- However, rosiglitazone was associated with significant
thy and dermopathy, which are pathognomonic for Graves risk for weight gain and heart failure despite the younger
disease. Therefore, the possibility of thyroiditis is effec- population studied (mean age, 55 years) in the Diabetes
tively excluded. The presence of moderate to severe oph- Reduction Assessment With Ramipril and Rosiglitazone
thalmopathy in a smoker is a relative contraindication for Medication (DREAM) trial. Since the DREAM trial, the
the use of radioactive iodine, which might exacerbate his use of rosiglitazone has been significantly restricted because

74
8.
ONCOLOGY QUESTIONS AND ANSWER S

1 0
59
65
QUESTIONS not tender. Findings from mammography, breast ultra-

48
sonography, and computed tomography of the chest are
Multiple Choice (choose the best answer) negative except for the presence of right axillary ade-

88
1. An otherwise healthy 32-year-old man asks you about nopathy. Biopsy of a lymph node shows a moderately
screening tests for colon cancer. He reports that his differentiated adenocarcinoma of unknown primary

91
2 older sisters had colon cancer at age 40 and 42, his origin. What should be the next step in evaluation of
mother had colon cancer at age 40 and endometrial this patient?

00
a. Perform a mediastinoscopy.
cancer at age 45, and a maternal aunt had breast cancer
b. Assume that non–small cell carcinoma is present, and treat
at a young age. He notes that there was not a history of
with cisplatin-based chemotherapy.
colon polyps in any family member. What should you

AP
c. Clarify the histogenetic origin of the tumor by testing for
recommend for this patient?
tumor markers: carcinoembryonic antigen, cancer antigen
a. Annual fecal occult blood testing

TS
15–3, and neuron-specific enolase.
b. Colonoscopy now and every 1 to 2 years thereafter
d. Perform breast magnetic resonance imaging (MRI).
c. Colonoscopy with random biopsies to look for inflamma-
e. Recommend bilateral mastectomies.
HA
tory bowel disease—if not present, follow routine screening
recommendations for average-risk Americans 4. A 38-year-old woman presents for intermittent abdom-
d. Prophylactic colectomy
W

inal pain and bloating that has been getting worse for
e. Reassurance only, since no polyps were found in family the past several months. She has been reading medical
members information on the Internet and is very concerned about
AL

2. A 72-year-old male smoker with a 42-pack-year history ovarian cancer. She has no family history of malig-
presents with anorexia, cough, and altered mental sta- nancy and has been otherwise healthy. She is not tak-
I
ER

tus. A chest radiograph shows a right-sided mass that, on ing any medications. Physical examination findings are
bronchoscopy, is identified as squamous cell carcinoma. remarkable only for some tenderness to movement of
On physical examination, the patient is thin, cachec- the uterus. No pelvic masses are detected. She requests
AT

tic, dehydrated, and disoriented with no focal neuro- a serum cancer antigen (CA) 125 test; the result is 86
logic deficits. The calcium level is elevated (15 mg/dL), U/mL (reference range <35 U/mL). How should you
M

creatinine is 2.5 mg/dL, and albumin is 2.2 g/dL. advise her at this time?
A bone scan shows only some degenerative changes. a. Recommend combination chemotherapy with cisplatin and
SE

What is the most appropriate next step in the manage- paclitaxel.


ment of this patient? b. Tell her that this degree of CA 125 elevation occurs only in
ovarian cancer.
UR

a. Cisplatin-based chemotherapy
b. Radiotherapy to the brain c. Tell her that although the CA 125 elevation is concerning,
c. Intravenous fluids and bisphosphonates multiple conditions can cause such an elevation, and further
CO

d. Dexamethasone 100 mg given as an intravenous push investigation is warranted.


e. Emergent magnetic resonance imaging of the head d. Recommend exploratory laparotomy with total abdomi-
nal hysterectomy, bilateral salpingo-oophorectomy, pelvic
3. A 55-year-old woman with a history of hypertension, a
LL

lymphadenectomy, omentectomy, and aggressive surgical


30-pack-year history of smoking, mild chronic obstruc- debulking of all disease.
tive pulmonary disease, and moderate obesity presents
FU

e. Recommend only observation now, and recheck the CA 125


with a right axillary mass. She has a family history of level in 3 months.
coronary artery disease and strokes. Current medica-
tions include a statin, a diuretic, and a β-blocker. On 5. A 38-year-old woman with recently diagnosed
examination, she is moderately obese and in no dis- node-positive breast cancer presents to the emergency
tress. Her lungs have increased sound in the expiratory department with a temperature of 38.3°C. She reports
phase diffusely but no frank wheezing or other sounds. having mild chills and fever but denies having nausea,
Findings from examination of the heart, abdomen, and vomiting, diarrhea, cough, or dysuria. Seven days ago
breasts are unremarkable. On lymph node examina- she received her third cycle of doxorubicin and cyclo-
tion, a palpable right axillary mass is firm, mobile, and phosphamide chemotherapy; thus far, she has tolerated

79
9.
HEMATOLOGY QUESTIONS AND ANSWER S

1 0
59
65
QUESTIONS 2. A 45-year-old woman is admitted to the surgical service

48
with severe arterial insufficiency of the right second toe.
Multiple Choice (choose the best answer) She has no prior medical history and takes no medica-

88
tions. Physical examination findings are normal except
A N E M I A S A N D MY E L O I D M A L I G NA N C I E S
for mild splenomegaly and signs of early gangrene in the

91
right second toe. All pulses are full and equal through-
1. A 67-year-old man is evaluated for exertional dyspnea. out. Diagnostic testing results are shown in Table 9.Q2.

00
He recalls that 3 years ago he was told that he had ane-
mia. In reviewing his records, you note that at that time Table 9.Q2
his hemoglobin level was 9.5 g/dL and his hematocrit

AP
was 33% with an increased mean corpuscular volume COMPONENT FINDING
(MCV); the remainder of his complete blood cell count Hemoglobin, g/dL 13.2

TS
was normal. On physical examination, he had conjunc-
tival pallor, normal heart and lung findings, no lymph- Hematocrit, % 39
adenopathy, no hepatomegaly or splenomegaly, and no
HA
Leukocyte count, ×109/L 15.5
petechiae or ecchymoses. Diagnostic testing results are Segmented neutrophils, % 78
shown in Table 9.Q1. Band cells, % 4
W

Lymphocytes, % 20
Table 9.Q1 Monocytes, % 5
Basophils, % 2
AL

COMPONENT FINDING Eosinophils, % 1

Platelet count, ×109/L 1,300


I

Hemoglobin, g/dL 7.5


ER

Hematocrit, % 23 Mean corpuscular volume, f L 88

Erythrocyte sedimentation 28
AT

Mean corpuscular volume, f L 110 (reference range, 86–98)


rate, mm/h
Leukocyte count, ×109/L 2.1
Leukocyte alkaline 110 (reference range, 13–130)
M

Neutrophils, % 20
Lymphocytes, % 70 phosphatase score
Monocytes, % 6
SE

Serum ferritin Within reference range


Basophils, % 3
Eosinophils, % 1 Serum iron Within reference range
UR

9
Platelet count, ×10 /L 64 Serum total iron-binding Within reference range
capacity
Reticulocyte count, % of 0.3 (reference range, 0.5–1.5)
CO

erythrocytes Peripheral blood film Increased large platelets with


some clustering; leukocytes and
Absolute reticulocyte 10.0 (reference range, 29.5–87.3) erythrocytes are unremarkable
count, ×109/L
LL

Bone marrow aspiration and Increased cellularity with increased


Peripheral blood film Dimorphic erythrocyte population biopsy and atypical megakaryocytes in
with pronounced macrocytes
FU

clusters; reticulin staining is normal


Lactate dehydrogenase, U/L 150 (reference range, 140–280) Chromosomal analysis Normal female karyotype (46XY)

Which of the following is the most likely explanation


for these findings? Which of the following is the most likely diagnosis?
a. Acute myeloid leukemia (AML) a. Essential thrombocythemia
b. Vitamin B12 deficiency b. Vasculitis
c. Hemolytic anemia c. Philadelphia chromosome–negative chronic myeloid leuke-
d. Myelodysplastic syndrome (MDS) mia (CML)
e. Primary myelofibrosis d. Primary myelofibrosis (PMF)

83
10.
NEPHROLOGY QUESTIONS AND ANSWER S

1 0
59
65
QUESTIONS Which of the following contributed to his

48
hyponatremia?
Multiple Choice (choose the best answer) a. HCTZ

88
b. Decreased intravascular volume
c. Beer drinking
E L E C T RO LY T E D I S O R D E R S

91
d. Chronic kidney disease
1. A 68-year-old man, a formerly heavy smoker, presents e. Syndrome of inappropriate secretion of antidiuretic

00
with a blood-tinged cough and weakness. He uses an hormone
ipratropium bromide inhaler. On physical examination,
his vital signs are normal, his jugular venous pressure is 3. An 80-year-old woman who is a nursing home resident

AP
normal, and he has no edema. His laboratory values are with hypertension, multi-infarct dementia, dysphasia,
as follows: sodium 125 mEq/L, potassium 3.6 mEq/L, and atrial fibrillation was admitted because her men-
tal status changed. Physical examination findings and

TS
bicarbonate 25 mEq/L, serum urea nitrogen 7 mg/dL,
and serum creatinine 0.9 mg/dL. What should you laboratory test results are shown in Table 10.Q3.
obtain next?
HA
a. Serum osmolality value Table 10.Q3
b. Urine osmolality value
W

COMPONENT FINDING
c. Thyrotropin and morning cortisol values
d. Computed tomographic (CT) scan of the chest Blood pressure, mm Hg 106/58
AL

e. Magnetic resonance imaging of the brain


Pulse, beats per minute 90 (irregular)
2. A 56-year-old man with a history of hypertension and
I

drinking alcoholic beverages (mostly beer) presents Respiratory rate, breaths per minute 24
ER

with episodic confusion, weakness, and imbalance. For Oxygen saturation with room air, % 97
the past 3 weeks, he has been taking hydrocholorothi-
AT

azide (HCTZ) 25 mg once daily. On physical examina- Weight, kg 60


tion, his blood pressure is 103/50 mm Hg, his pulse is Mucosal surfaces Dry
110 beats per minute, his respiratory rate is 20 breaths
M

per minute, his jugular venous pressure is normal, the Mental status Sleepy, oriented to name only
liver edge is palpable, and he has no edema. Laboratory
SE

Lung fields Clear, no edema


test results are shown in Table 10.Q2.
Sodium, mEq/L 176
UR

Potassium, mEq/L 3.5


Table 10.Q2
Chloride, mEq/L 129
CO

COMPONENT FINDING
Bicarbonate, mEq/L 35
Sodium, mEq/L 105
Serum creatinine, mg/dL 1.2
LL

Potassium, mEq/L 3.2


Urine osmolality, mOsm/kg 655
Chloride, mEq/L 70
FU

Bicarbonate, mEq/L 30
What is her estimated water deficit?
Serum creatinine, mg/dL 1.3 a. 6.5 L
b. 7.3 L
Serum osmolality, mOsm/kg 224
c. 9.3 L
Urine osmolality, mOsm/kg 410 d. 10.5 L
e. 2.4 L
Thyrotropin Within reference range
Morning cortisol Within reference range

93
11.
ALLERGY QUESTIONS AND ANSWER S

1 0
59
65
QUESTIONS Within minutes, the area becomes red, painful, pru-

48
ritic, and swollen. During the next several hours, the
Multiple Choice (choose the best answer) redness and swelling spread to the elbow. He denies

88
1. A 28-year-old woman presents with year-round symp- having other symptoms; specifically, he denies having
toms of nasal congestion, rhinorrhea, and sneezing. dyspnea, light-headedness, nausea, vomiting, or diar-

91
Although the symptoms are present year-round, they rhea. There are no skin manifestations elsewhere. What
worsen in the spring and fall. Her symptoms are bilat- is the most appropriate management?

00
a. Apply ice to the arm, provide symptomatic relief, and review
eral and have responded partially to treatment with
strategies to avoid stinging insects.
over-the-counter antihistamines. In an effort to control
b. Perform skin testing to bee only; if results are positive,
her symptoms, which medication should not be used as

AP
administer immunotherapy.
part of her treatment plan?
c. Perform skin testing to apids (honeybee) and vespids (yel-
a. Nonsedating antihistamine

TS
low jacket, wasp, and hornet); if any of the results are posi-
b. Leukotriene antagonist
tive, administer immunotherapy.
c. Topical decongestant
d. Perform an in vitro test for IgE specific for apids (honeybee)
HA
d. Intranasal corticosteroid
and vespids (yellow jacket, wasp, and hornet); if results are
e. Saline nasal spray
positive, administer immunotherapy.
W

2. A 26-year-old woman is treated for Neisseria gonor- e. Administer 0.3 mL epinephrine 1:1,000 intramuscularly
rhoeae infection. She has had 2 episodes of Neisseria and observe.
AL

meningitidis infection. As a child, she had 1 episode


of pneumonia. She has no history of skin infections or 5. A 31-year-old woman presents in her fifth month of
abscesses. What type of defect is the most likely under- pregnancy with increasing asthma symptoms. Her
I

asthma had been well controlled throughout pregnancy


ER

lying immunologic abnormality?


a. C2
but worsened over the past 4 days with the onset of an
b. C5–9
upper respiratory tract infection. She has daily symp-
AT

c. IgG
toms and nighttime awakenings due to dyspnea. She
d. Neutrophil chemotaxis
uses her albuterol inhaler every 3 hours and has adhered
M

e. Lymphocyte count
to her usual inhaler regimen of budesonide 2 puffs daily.
She denies having fever or purulent mucus. On exami-
3. A 22-year-old man presents with a 10-year history of
SE

nation, she has scattered expiratory wheezes through-


recurrent angioedema, particularly of the lips and eyes. out all lung fields, a respiratory rate of 18 breaths per
Throat swelling occurred on 3 occasions. The episodes minute, oxygen saturation of 97%, and forced expira-
UR

seem to occur randomly, approximately monthly, and tory volume in 1 second (FEV1) of 64% of the predicted
there is no association with food, medications (includ- value, which improves to 72% after albuterol. How
ing over-the-counter products), or environmental should you change her therapy?
CO

exposures. His father reports having similar symptoms a. Add montelukast 10 mg daily.
for years. The patient is otherwise healthy. Current b. Increase the dosage of budesonide to 2 puffs twice daily.
physical examination findings are normal, although he c. Start the use of amoxicillin 500 mg 3 times daily.
LL

brings in photographs that show marked angioedema d. Discontinue the use of the budesonide inhaler.
of his orbits and lips. What type of abnormality does e. Add prednisone 40 mg daily for 5 days.
FU

this patient have?


a. C4
6. A 43-year-old woman reports that she had pruritic
b. C1q
hives and wheezing after her second dose of intravenous
c. Tryptase
penicillin approximately 10 years ago. She has avoided
d. Lymphocyte count
penicillin since then and has not had any further drug
e. 24-hour urine N-methylhistamine
reactions. Currently, she is being treated for endocardi-
tis, and high-dose penicillin is considered the drug of
4. A 32-year-old man is stung on the distal right forearm choice. What recommendation can be given for penicil-
by what he describes as a “bee.” No stinger is visible. lin use?

105
12.
PSYCHIATRY QUESTIONS AND ANSWER S

1 0
59
65
QUESTIONS her hands are now raw from all the scrubbing. With fur-

48
ther queries, she says that she feels compelled to count
Multiple Choice (choose the best answer) ceiling tiles in your examination room and she worries

88
1. A 72-year-old man has a history of poorly controlled irrationally about her children’s health. She has sleep
type 2 diabetes mellitus, coronary heart disease with QT difficulties, and her anxiety is so high throughout the

91
prolongation (corrected QT interval 482 ms), and major day that she has difficulty working. Which of the fol-
depression. He presents to your office with a 2-month lowing would not be an appropriate recommendation

00
history of worsening depressive symptoms, including for this patient?
a. Treat with bupropion titrated to 150 mg twice daily.
severely depressed mood, tearfulness, decreased con-
b. Treat with clonazepam 0.5 mg twice daily as a bridge until
centration, hopelessness, middle insomnia, and passive

AP
other treatment is effective.
suicidal ideation without a specific intent or plan. He
c. Treat with fluoxetine 20 mg daily.
has been taking sertraline 150 mg daily (the dosage was

TS
d. Taper and eliminate her intake of caffeine.
recently increased from 100 mg daily) without much
e. Recommend cognitive behavioral psychotherapy (CBT).
benefit. He has been experiencing life stresses including
HA
financial strain and his wife’s declining health. Which 4. A 42-year-old woman presents to your office again
of the following would not be a reasonable next step in after 16 years of intermittent severe left lower quadrant
managing his depression? abdominal pain. She denies having weight loss, fever, or
W

a. Cross-taper his medication from sertraline to amitriptyline. chills. The cause of her symptoms is not apparent from
b. Cross-taper his medication from sertraline to bupropion. previous workups, which included a complete blood
AL

c. Perform a preanesthetic medical examination and refer the cell count, electrolyte evaluations, urinalysis, computed
patient to a psychiatrist for consideration of electroconvul- tomographic scan of the abdomen and pelvis, colonos-
I

sive therapy (ECT). copy, and gynecologic examination. She has previ-
ER

d. Refer the patient to a psychologist for psychotherapy. ously been thoroughly evaluated for episodic dizziness,
e. Ask the patient about his access to firearms and enlist family headaches, flulike syndromes, back pain, and pain with
AT

members to remove them from the home. intercourse. The results of all these workups were nega-
2. A 33-year-old woman has recently received a diagnosis tive. What is the most likely diagnosis?
M

a. Conversion disorder
of bipolar disorder and is now taking lithium carbonate
b. Somatization disorder
600 mg orally twice daily. She wants to know some of
SE

c. Hypochondriasis
the long-term risks of taking this medication. Which of
d. Body dysmorphic disorder
the following is false about long-term management of
e. Factitial disorder
UR

bipolar disorder with lithium?


a. Nonsteroidal anti-inflammatory drugs (NSAIDs) should 5. A 56-year-old man with a history of schizophrenia has
be avoided because of nephrotoxicity associated with taking a history of mild congestive heart failure and osteoar-
CO

the combination of an NSAID and lithium. thritis. He harbors the paranoid belief that his wife is
b. There is an increased risk of birth defects in children of poisoning him, and he gets special messages from the
women taking lithium during pregnancy. television. These symptoms have been partially con-
LL

c. Thyroid function needs to be monitored because of the trolled with a second-generation antipsychotic, olan-
potential for thyrotoxicity. zapine 5 mg orally daily. You recommend increasing the
FU

d. Acetaminophen should be avoided because of hepatotoxic- olanzapine dosage to 10 mg daily. Which of the follow-
ity associated with taking the combination of acetamino- ing is false about the use of olanzapine in this patient?
phen and lithium. a. He is at increased risk of neuroleptic malignant syndrome.
e. Lithium doses may need to be decreased temporarily if an b. He is at increased risk of parkinsonism.
acute illness leads to dehydration. c. He is at increased risk of tardive dyskinesia.
d. He is at increased risk of diabetes mellitus.
3. A 32-year-old woman presents with anxiety about
e. He is at increased risk of anorexia.
germs. She washes her hands 30 to 40 times a day, and

109
P1: SBT
0521779407pre CUNY1086/Karliner 0 521 77940 7 June 27, 2007 10:22

10
59
65
Contents

48
88
Preface page xxix

91
Abdominal Aortic Aneurysm (AAA) . . . . . . . . . . . . . . . . . . . . . . . . . 1
Abscesses and Fistulas . . . . . . . . . . . . . . . . . . . . . . . . . . . . . . . . . . 5

00
Acanthosis Nigricans . . . . . . . . . . . . . . . . . . . . . . . . . . . . . . . . . . . 7

AP
Acne . . . . . . . . . . . . . . . . . . . . . . . . . . . . . . . . . . . . . . . . . . . . . . . 9
Acromegaly . . . . . . . . . . . . . . . . . . . . . . . . . . . . . . . . . . . . . . . . . 12
TS
Actinic Keratosis . . . . . . . . . . . . . . . . . . . . . . . . . . . . . . . . . . . . . . 15
HA

Acute Acalculous Cholecystitis . . . . . . . . . . . . . . . . . . . . . . . . . . . 16


Acute Bacterial Meningitis . . . . . . . . . . . . . . . . . . . . . . . . . . . . . . 18
W

Acute Fatty Liver of Pregnancy . . . . . . . . . . . . . . . . . . . . . . . . . . . 21


L

Acute Heart Failure . . . . . . . . . . . . . . . . . . . . . . . . . . . . . . . . . . . . 23


IA

Acute Lymphoblastic Leukemia . . . . . . . . . . . . . . . . . . . . . . . . . . . 29


ER

Acute Myeloblastic Leukemia . . . . . . . . . . . . . . . . . . . . . . . . . . . . 37


AT

Acute Myocarditides . . . . . . . . . . . . . . . . . . . . . . . . . . . . . . . . . . . 41
Acute Pancreatitis . . . . . . . . . . . . . . . . . . . . . . . . . . . . . . . . . . . . . 45
M

Acute Pericarditis . . . . . . . . . . . . . . . . . . . . . . . . . . . . . . . . . . . . . 48
SE

Acute Renal Failure . . . . . . . . . . . . . . . . . . . . . . . . . . . . . . . . . . . . 50


UR

Acute Respiratory Acidosis . . . . . . . . . . . . . . . . . . . . . . . . . . . . . . 55


Acute Respiratory Alkalosis . . . . . . . . . . . . . . . . . . . . . . . . . . . . . . 56
CO

Acute Respiratory Failure/Monitoring . . . . . . . . . . . . . . . . . . . . . . 58


Adenocarcinoma of the Ampulla of Vater . . . . . . . . . . . . . . . . . . . . 62
LL

Adenovirus . . . . . . . . . . . . . . . . . . . . . . . . . . . . . . . . . . . . . . . . . . 63
FU

Adrenal Insufficiency . . . . . . . . . . . . . . . . . . . . . . . . . . . . . . . . . . 66
Adrenal Tumors . . . . . . . . . . . . . . . . . . . . . . . . . . . . . . . . . . . . . . 70
Adult Optic Neuropathies . . . . . . . . . . . . . . . . . . . . . . . . . . . . . . . 74
Age Related Maculopathy . . . . . . . . . . . . . . . . . . . . . . . . . . . . . . . 80

ix
P1: SBT
0521779407pre CUNY1086/Karliner 0 521 77940 7 June 27, 2007 10:22

x Contents

Air Embolus . . . . . . . . . . . . . . . . . . . . . . . . . . . . . . . . . . . . . . . . . 83

10
Alcohol Abuse, Dependence, and Withdrawal . . . . . . . . . . . . . . . . 85

59
Alcoholic Liver Disease (ALD) . . . . . . . . . . . . . . . . . . . . . . . . . . . . 89

65
Allergic Rhinitis . . . . . . . . . . . . . . . . . . . . . . . . . . . . . . . . . . . . . . 93

48
Alopecia . . . . . . . . . . . . . . . . . . . . . . . . . . . . . . . . . . . . . . . . . . . . 96
Alpha-1-Antitrypsin . . . . . . . . . . . . . . . . . . . . . . . . . . . . . . . . . . 100

88
Alzheimer’s Disease . . . . . . . . . . . . . . . . . . . . . . . . . . . . . . . . . . 101

91
Amaurosis Fugax . . . . . . . . . . . . . . . . . . . . . . . . . . . . . . . . . . . . 103

00
Amblyopia . . . . . . . . . . . . . . . . . . . . . . . . . . . . . . . . . . . . . . . . . 104
Amebiasis . . . . . . . . . . . . . . . . . . . . . . . . . . . . . . . . . . . . . . . . . . 106

AP
Amebic Liver Abscess . . . . . . . . . . . . . . . . . . . . . . . . . . . . . . . . . 109
TS
Amyloidosis . . . . . . . . . . . . . . . . . . . . . . . . . . . . . . . . . . . . . . . . 111
Amyotrophic Lateral Sclerosis (ALS) . . . . . . . . . . . . . . . . . . . . . . 115
HA

Analgesic Nephropathy And NSAID-induced ARF . . . . . . . . . . . . 117


W

Anaphylaxis . . . . . . . . . . . . . . . . . . . . . . . . . . . . . . . . . . . . . . . . 120
Anemias Secondary to Systemic Disease . . . . . . . . . . . . . . . . . . . 122
L
IA

Anisakiasis . . . . . . . . . . . . . . . . . . . . . . . . . . . . . . . . . . . . . . . . . 125
ER

Anorectal Tumors . . . . . . . . . . . . . . . . . . . . . . . . . . . . . . . . . . . . 126


Anorexia Nervosa . . . . . . . . . . . . . . . . . . . . . . . . . . . . . . . . . . . . 129
AT

Antibiotic-Associated and Pseudomembranous Colitis . . . . . . . . . 133


M

Antiphospholipid Antibodies . . . . . . . . . . . . . . . . . . . . . . . . . . . . 135


SE

Aortic Coarctation . . . . . . . . . . . . . . . . . . . . . . . . . . . . . . . . . . . . 138


Aortic Dissection . . . . . . . . . . . . . . . . . . . . . . . . . . . . . . . . . . . . 140
UR

Aortic Insufficiency (AI) . . . . . . . . . . . . . . . . . . . . . . . . . . . . . . . . 143


Aortitis . . . . . . . . . . . . . . . . . . . . . . . . . . . . . . . . . . . . . . . . . . . . 146
CO

Aphasia . . . . . . . . . . . . . . . . . . . . . . . . . . . . . . . . . . . . . . . . . . . 149
Aplastic Anemia . . . . . . . . . . . . . . . . . . . . . . . . . . . . . . . . . . . . . 150
LL

Apparent Mineralocorticoid Excess . . . . . . . . . . . . . . . . . . . . . . . 153


FU

Appendicitis . . . . . . . . . . . . . . . . . . . . . . . . . . . . . . . . . . . . . . . . 153
Arterial Embolus . . . . . . . . . . . . . . . . . . . . . . . . . . . . . . . . . . . . . 156
Ascariasis . . . . . . . . . . . . . . . . . . . . . . . . . . . . . . . . . . . . . . . . . . 160
Ascites . . . . . . . . . . . . . . . . . . . . . . . . . . . . . . . . . . . . . . . . . . . . 162
P1: SBT
0521779407pre CUNY1086/Karliner 0 521 77940 7 June 27, 2007 10:22

Contents xi

Aspergillosis . . . . . . . . . . . . . . . . . . . . . . . . . . . . . . . . . . . . . . . . 166

10
Asthma . . . . . . . . . . . . . . . . . . . . . . . . . . . . . . . . . . . . . . . . . . . . 171

59
Atelectasis . . . . . . . . . . . . . . . . . . . . . . . . . . . . . . . . . . . . . . . . . 176

65
Atherosclerotic Occlusive Disease . . . . . . . . . . . . . . . . . . . . . . . . 179

48
Atopic Dermatitis . . . . . . . . . . . . . . . . . . . . . . . . . . . . . . . . . . . . 187
Atrial Fibrillation (AF) . . . . . . . . . . . . . . . . . . . . . . . . . . . . . . . . . 188

88
Atrial Flutter . . . . . . . . . . . . . . . . . . . . . . . . . . . . . . . . . . . . . . . . 191

91
Atrial Premature Complexes . . . . . . . . . . . . . . . . . . . . . . . . . . . . 194

00
Atrial Septal Defect (ASD) . . . . . . . . . . . . . . . . . . . . . . . . . . . . . . 195
Atrial Tachycardia (AT) . . . . . . . . . . . . . . . . . . . . . . . . . . . . . . . . 197

AP
Atrioventricular Block . . . . . . . . . . . . . . . . . . . . . . . . . . . . . . . . . 198
TS
Autoimmune Hepatitis (AIH) . . . . . . . . . . . . . . . . . . . . . . . . . . . . 201
Autonomic Dysfunction . . . . . . . . . . . . . . . . . . . . . . . . . . . . . . . 204
HA

Autosomal Dominant Polycystic Renal Disease . . . . . . . . . . . . . . 206


W

Autosomal Recessive Polycystic Renal Disease . . . . . . . . . . . . . . . 208


AV Reentrant Tachycardia (AVRT) . . . . . . . . . . . . . . . . . . . . . . . . 209
L
IA

AV-Nodal Reentrant Tachycardia (AVNRT) . . . . . . . . . . . . . . . . . . 211


ER

Back or Neck Pain . . . . . . . . . . . . . . . . . . . . . . . . . . . . . . . . . . . . 213


Bacterial Arthritis . . . . . . . . . . . . . . . . . . . . . . . . . . . . . . . . . . . . 214
AT

Bacterial Pneumonia . . . . . . . . . . . . . . . . . . . . . . . . . . . . . . . . . . 218


M

Balanitis . . . . . . . . . . . . . . . . . . . . . . . . . . . . . . . . . . . . . . . . . . . 220
SE

Bartter’s Syndrome . . . . . . . . . . . . . . . . . . . . . . . . . . . . . . . . . . . 223


Bell’s Palsy . . . . . . . . . . . . . . . . . . . . . . . . . . . . . . . . . . . . . . . . . 224
UR

Benign Disorders of the Rectum and Anus . . . . . . . . . . . . . . . . . . 225


Benign Prostatic Hyperplasia . . . . . . . . . . . . . . . . . . . . . . . . . . . . 228
CO

Benign Tumors of the Liver . . . . . . . . . . . . . . . . . . . . . . . . . . . . . 232


Biliary Tract Motility Disorders . . . . . . . . . . . . . . . . . . . . . . . . . . 235
LL

Biotin Deficiency . . . . . . . . . . . . . . . . . . . . . . . . . . . . . . . . . . . . 238


FU

Bladder Tumors . . . . . . . . . . . . . . . . . . . . . . . . . . . . . . . . . . . . . 239


Blastocystis Hominis Infection . . . . . . . . . . . . . . . . . . . . . . . . . . 242
Blastomyces Dermatitidis . . . . . . . . . . . . . . . . . . . . . . . . . . . . . . 243
Blepharitis . . . . . . . . . . . . . . . . . . . . . . . . . . . . . . . . . . . . . . . . . 246
P1: SBT
0521779407pre CUNY1086/Karliner 0 521 77940 7 June 27, 2007 10:22

xii Contents

Brain Abscess . . . . . . . . . . . . . . . . . . . . . . . . . . . . . . . . . . . . . . . 248

10
Brain Death . . . . . . . . . . . . . . . . . . . . . . . . . . . . . . . . . . . . . . . . 249

59
Bronchiectasis . . . . . . . . . . . . . . . . . . . . . . . . . . . . . . . . . . . . . . 250

65
Bronchiolitis . . . . . . . . . . . . . . . . . . . . . . . . . . . . . . . . . . . . . . . . 253

48
Bronchitis, Acute . . . . . . . . . . . . . . . . . . . . . . . . . . . . . . . . . . . . 257
Budd-Chiari Syndrome . . . . . . . . . . . . . . . . . . . . . . . . . . . . . . . . 259

88
Buerger’s Disease . . . . . . . . . . . . . . . . . . . . . . . . . . . . . . . . . . . . 261

91
Bulimia Nervosa . . . . . . . . . . . . . . . . . . . . . . . . . . . . . . . . . . . . . 263

00
Bullous Pemphigoid . . . . . . . . . . . . . . . . . . . . . . . . . . . . . . . . . . 266
Candidiasis . . . . . . . . . . . . . . . . . . . . . . . . . . . . . . . . . . . . . . . . . 270

AP
Candidiasis: Oral . . . . . . . . . . . . . . . . . . . . . . . . . . . . . . . . . . . . 272
TS
Carcinoid . . . . . . . . . . . . . . . . . . . . . . . . . . . . . . . . . . . . . . . . . . 273
Cardiac Arrest . . . . . . . . . . . . . . . . . . . . . . . . . . . . . . . . . . . . . . . 277
HA

Cardiac Trauma . . . . . . . . . . . . . . . . . . . . . . . . . . . . . . . . . . . . . 280


W

Cardiac Tumors . . . . . . . . . . . . . . . . . . . . . . . . . . . . . . . . . . . . . 281


Cat Scratch Disease . . . . . . . . . . . . . . . . . . . . . . . . . . . . . . . . . . . 286
L
IA

Cecal Volvulus . . . . . . . . . . . . . . . . . . . . . . . . . . . . . . . . . . . . . . 288


ER

Celiac Sprue and Malabsorption . . . . . . . . . . . . . . . . . . . . . . . . . 289


Cellulitis . . . . . . . . . . . . . . . . . . . . . . . . . . . . . . . . . . . . . . . . . . . 292
AT

Cerebrovascular Disease & Stroke . . . . . . . . . . . . . . . . . . . . . . . . 294


M

Cervical Spine Disorders . . . . . . . . . . . . . . . . . . . . . . . . . . . . . . . 297


SE

Chediak Higashi Syndrome . . . . . . . . . . . . . . . . . . . . . . . . . . . . . 298


Cheilitis . . . . . . . . . . . . . . . . . . . . . . . . . . . . . . . . . . . . . . . . . . . 299
UR

Cholangiocarcinoma . . . . . . . . . . . . . . . . . . . . . . . . . . . . . . . . . . 299
Choledochal Cysts . . . . . . . . . . . . . . . . . . . . . . . . . . . . . . . . . . . 302
CO

Cholestasis . . . . . . . . . . . . . . . . . . . . . . . . . . . . . . . . . . . . . . . . . 305
Cholestasis of Pregnancy . . . . . . . . . . . . . . . . . . . . . . . . . . . . . . . 308
LL

Chordae Tendineae Rupture . . . . . . . . . . . . . . . . . . . . . . . . . . . . 309


FU

Chromium Deficiency . . . . . . . . . . . . . . . . . . . . . . . . . . . . . . . . . 310


Chronic Acalculous Cholecystitis . . . . . . . . . . . . . . . . . . . . . . . . . 311
Chronic Bundle Branch Block and Hemi-blocks . . . . . . . . . . . . . . 312
Chronic Coronary Artery Disease (CAD) . . . . . . . . . . . . . . . . . . . . 314
P1: SBT
0521779407pre CUNY1086/Karliner 0 521 77940 7 June 27, 2007 10:22

Contents xiii

Chronic Granulomatous Disease . . . . . . . . . . . . . . . . . . . . . . . . . 317

10
Chronic Heart Failure . . . . . . . . . . . . . . . . . . . . . . . . . . . . . . . . . 318

59
Chronic Kidney Disease . . . . . . . . . . . . . . . . . . . . . . . . . . . . . . . 327

65
Chronic Lymphocytic Leukemia . . . . . . . . . . . . . . . . . . . . . . . . . 333

48
Chronic Myelogenous Leukemia . . . . . . . . . . . . . . . . . . . . . . . . . 336
Chronic Obstructive Pulmonary Disease . . . . . . . . . . . . . . . . . . . 340

88
Chronic Pancreatitis . . . . . . . . . . . . . . . . . . . . . . . . . . . . . . . . . . 343

91
Chronic Renal Failure . . . . . . . . . . . . . . . . . . . . . . . . . . . . . . . . . 346

00
Chronic Respiratory Acidosis . . . . . . . . . . . . . . . . . . . . . . . . . . . . 351
Chronic Respiratory Alkalosis . . . . . . . . . . . . . . . . . . . . . . . . . . . 352

AP
Chronic Respiratory Failure . . . . . . . . . . . . . . . . . . . . . . . . . . . . . 353
TS
Churg Strauss Disease . . . . . . . . . . . . . . . . . . . . . . . . . . . . . . . . . 357
Clostridium Infections . . . . . . . . . . . . . . . . . . . . . . . . . . . . . . . . 360
HA

Cluster Headache . . . . . . . . . . . . . . . . . . . . . . . . . . . . . . . . . . . . 362


W

Coccidioides Immitis . . . . . . . . . . . . . . . . . . . . . . . . . . . . . . . . . 363


Colon Polyps and Tumors . . . . . . . . . . . . . . . . . . . . . . . . . . . . . . 367
L
IA

Coma . . . . . . . . . . . . . . . . . . . . . . . . . . . . . . . . . . . . . . . . . . . . . 370
ER

Complications of Human Immunodeficiency Virus Type 1


(HIV-1) Infection . . . . . . . . . . . . . . . . . . . . . . . . . . . . . . . . . . . . . 371
AT

Condyloma Acuminata . . . . . . . . . . . . . . . . . . . . . . . . . . . . . . . . 388


M

Confusion & Delirium . . . . . . . . . . . . . . . . . . . . . . . . . . . . . . . . . 389


Congenital Qualitative Platelet Disorders . . . . . . . . . . . . . . . . . . . 390
SE

Conjunctival Tumors . . . . . . . . . . . . . . . . . . . . . . . . . . . . . . . . . . 393


UR

Constipation and Fecal Impaction . . . . . . . . . . . . . . . . . . . . . . . . 394


Constrictive Pericarditis . . . . . . . . . . . . . . . . . . . . . . . . . . . . . . . 397
CO

Contact Dermatitis . . . . . . . . . . . . . . . . . . . . . . . . . . . . . . . . . . . 398


Contraception for the Internist . . . . . . . . . . . . . . . . . . . . . . . . . . 399
LL

Corneal Ulcer . . . . . . . . . . . . . . . . . . . . . . . . . . . . . . . . . . . . . . . 403


FU

Coronary Artery Injury . . . . . . . . . . . . . . . . . . . . . . . . . . . . . . . . 406


Coronary Syndromes, Acute . . . . . . . . . . . . . . . . . . . . . . . . . . . . 407
Coronavirus/Common Cold . . . . . . . . . . . . . . . . . . . . . . . . . . . . 413
Cough . . . . . . . . . . . . . . . . . . . . . . . . . . . . . . . . . . . . . . . . . . . . 414
P1: SBT
0521779407pre CUNY1086/Karliner 0 521 77940 7 June 27, 2007 10:22

xiv Contents

Crigler-Najjer Syndrome Type 1 . . . . . . . . . . . . . . . . . . . . . . . . . . 417

10
Crigler-Najjer Type 2 . . . . . . . . . . . . . . . . . . . . . . . . . . . . . . . . . . 419

59
Crohn’s Disease . . . . . . . . . . . . . . . . . . . . . . . . . . . . . . . . . . . . . 420

65
Croup . . . . . . . . . . . . . . . . . . . . . . . . . . . . . . . . . . . . . . . . . . . . . 424

48
Cryptococcus Neoformans . . . . . . . . . . . . . . . . . . . . . . . . . . . . . 426
Cryptosporidiosis . . . . . . . . . . . . . . . . . . . . . . . . . . . . . . . . . . . . 430

88
Crystal-Induced Arthritis . . . . . . . . . . . . . . . . . . . . . . . . . . . . . . . 432

91
Cushing’s Syndrome . . . . . . . . . . . . . . . . . . . . . . . . . . . . . . . . . . 435

00
Cutaneous Larva Migrans . . . . . . . . . . . . . . . . . . . . . . . . . . . . . . 439
Cutaneous Lupus Erythematosus (LE) . . . . . . . . . . . . . . . . . . . . . 441

AP
Cutaneous Vasculitis . . . . . . . . . . . . . . . . . . . . . . . . . . . . . . . . . . 444
TS
Cyclosporiasis . . . . . . . . . . . . . . . . . . . . . . . . . . . . . . . . . . . . . . . 447
Cyst of Tunica Albuginea of Testis . . . . . . . . . . . . . . . . . . . . . . . . 449
HA

Cystic Fibrosis . . . . . . . . . . . . . . . . . . . . . . . . . . . . . . . . . . . . . . 449


W

Cystic Fibrosis . . . . . . . . . . . . . . . . . . . . . . . . . . . . . . . . . . . . . . 452


Cysticercosis . . . . . . . . . . . . . . . . . . . . . . . . . . . . . . . . . . . . . . . . 455
L
IA

Cystinuria . . . . . . . . . . . . . . . . . . . . . . . . . . . . . . . . . . . . . . . . . . 458
ER

Cystitis and Pyelonephritis . . . . . . . . . . . . . . . . . . . . . . . . . . . . . 458


Cytomegalovirus . . . . . . . . . . . . . . . . . . . . . . . . . . . . . . . . . . . . . 461
AT

Deep Venous Thrombosis . . . . . . . . . . . . . . . . . . . . . . . . . . . . . . 463


M

Dementia . . . . . . . . . . . . . . . . . . . . . . . . . . . . . . . . . . . . . . . . . . 467
SE

Dent’s Disease . . . . . . . . . . . . . . . . . . . . . . . . . . . . . . . . . . . . . . 468


Dermatofibroma . . . . . . . . . . . . . . . . . . . . . . . . . . . . . . . . . . . . . 469
UR

Diabetes Insipidus . . . . . . . . . . . . . . . . . . . . . . . . . . . . . . . . . . . 470


Diabetes Mellitus, Type 1 . . . . . . . . . . . . . . . . . . . . . . . . . . . . . . . 472
CO

Treatment Goals . . . . . . . . . . . . . . . . . . . . . . . . . . . . . . . . . . . . . 474


Diabetes Mellitus, Type 2 . . . . . . . . . . . . . . . . . . . . . . . . . . . . . . . 475
LL

Diabetic Retinopathy . . . . . . . . . . . . . . . . . . . . . . . . . . . . . . . . . 479


FU

Diarrhea . . . . . . . . . . . . . . . . . . . . . . . . . . . . . . . . . . . . . . . . . . . 482
Dientamoeba Fragilis Infection . . . . . . . . . . . . . . . . . . . . . . . . . . 485
Diphtheria . . . . . . . . . . . . . . . . . . . . . . . . . . . . . . . . . . . . . . . . . 487
Disorders of Neutrophil Function . . . . . . . . . . . . . . . . . . . . . . . . 489
P1: SBT
0521779407pre CUNY1086/Karliner 0 521 77940 7 June 27, 2007 10:22

Contents xv

Disseminated Intravascular Coagulation, Thrombotic

10
Thrombocytopenic Purpura, Hemolytic Uremic Syndrome

59
(DIC/TTP/HUS) . . . . . . . . . . . . . . . . . . . . . . . . . . . . . . . . . . . . . 490
Diverticulitis and Diverticular Disease . . . . . . . . . . . . . . . . . . . . . 494

65
Drug Allergy . . . . . . . . . . . . . . . . . . . . . . . . . . . . . . . . . . . . . . . . 497

48
Drug and Toxin-induced Liver Diseases . . . . . . . . . . . . . . . . . . . . 500

88
Drug Eruptions . . . . . . . . . . . . . . . . . . . . . . . . . . . . . . . . . . . . . . 503

91
Dubin-Johnson Syndrome . . . . . . . . . . . . . . . . . . . . . . . . . . . . . . 508
Dyspepsia . . . . . . . . . . . . . . . . . . . . . . . . . . . . . . . . . . . . . . . . . . 509

00
Dysphagia . . . . . . . . . . . . . . . . . . . . . . . . . . . . . . . . . . . . . . . . . 512

AP
Dyspnea . . . . . . . . . . . . . . . . . . . . . . . . . . . . . . . . . . . . . . . . . . . 514
Echinococcosis . . . . . . . . . . . . . . . . . . . . . . . . . . . . . . . . . . . . . . 517
TS
Ehlers-Danlos Syndrome . . . . . . . . . . . . . . . . . . . . . . . . . . . . . . . 520
HA

Encephalitis . . . . . . . . . . . . . . . . . . . . . . . . . . . . . . . . . . . . . . . . 522
Enteral and Parenteral Nutrition . . . . . . . . . . . . . . . . . . . . . . . . . 524
W

Enterobiasis (Pinworm) . . . . . . . . . . . . . . . . . . . . . . . . . . . . . . . 530


L

Enteroviruses . . . . . . . . . . . . . . . . . . . . . . . . . . . . . . . . . . . . . . . 532
IA

Epididymitis and Orchitis . . . . . . . . . . . . . . . . . . . . . . . . . . . . . . 535


ER

Epilepsies . . . . . . . . . . . . . . . . . . . . . . . . . . . . . . . . . . . . . . . . . . 536
AT

Episcleritis and Scleritis . . . . . . . . . . . . . . . . . . . . . . . . . . . . . . . 537


Erectile Dysfunction . . . . . . . . . . . . . . . . . . . . . . . . . . . . . . . . . . 539
M

Erysipelas and Cellulitis . . . . . . . . . . . . . . . . . . . . . . . . . . . . . . . 543


SE

Erythema Multiforme Major; Aka Stevens Johnson Syndrome


(see also Drug Eruptions) . . . . . . . . . . . . . . . . . . . . . . . . . . . . . . 545
UR

Erythema Nodosum . . . . . . . . . . . . . . . . . . . . . . . . . . . . . . . . . . 548


CO

Esophageal Cancer . . . . . . . . . . . . . . . . . . . . . . . . . . . . . . . . . . . 550


Esophageal Infections and Inflammation . . . . . . . . . . . . . . . . . . . 551
LL

Esophageal Motor Disorders . . . . . . . . . . . . . . . . . . . . . . . . . . . . 556


FU

Essential Tremor . . . . . . . . . . . . . . . . . . . . . . . . . . . . . . . . . . . . . 559


Excessive Daytime Sleepiness . . . . . . . . . . . . . . . . . . . . . . . . . . . 560
Exfoliative Dermatitis . . . . . . . . . . . . . . . . . . . . . . . . . . . . . . . . . 561
Eyelid Lesions . . . . . . . . . . . . . . . . . . . . . . . . . . . . . . . . . . . . . . . 562
Fanconi Syndrome . . . . . . . . . . . . . . . . . . . . . . . . . . . . . . . . . . . 563
P1: SBT
0521779407pre CUNY1086/Karliner 0 521 77940 7 June 27, 2007 10:22

xvi Contents

Fever of Unknown Origin . . . . . . . . . . . . . . . . . . . . . . . . . . . . . . 564

10
Fibromas . . . . . . . . . . . . . . . . . . . . . . . . . . . . . . . . . . . . . . . . . . 568

59
Fibromyalgia . . . . . . . . . . . . . . . . . . . . . . . . . . . . . . . . . . . . . . . 568

65
Filariasis . . . . . . . . . . . . . . . . . . . . . . . . . . . . . . . . . . . . . . . . . . . 571

48
Folic Acid/Cobalamin (Vitamin B12) Deficiency . . . . . . . . . . . . . . 575
Folliculitis and Furunculosis . . . . . . . . . . . . . . . . . . . . . . . . . . . . 580

88
Food Allergies . . . . . . . . . . . . . . . . . . . . . . . . . . . . . . . . . . . . . . . 582

91
Food Poisoning . . . . . . . . . . . . . . . . . . . . . . . . . . . . . . . . . . . . . . 586

00
Foreign Bodies and Bezoars . . . . . . . . . . . . . . . . . . . . . . . . . . . . . 589
Fulminant Hepatic Failure . . . . . . . . . . . . . . . . . . . . . . . . . . . . . . 592

AP
Galactosemia . . . . . . . . . . . . . . . . . . . . . . . . . . . . . . . . . . . . . . . 596
TS
Gallbladder Cancers . . . . . . . . . . . . . . . . . . . . . . . . . . . . . . . . . . 599
Gallstone Disease . . . . . . . . . . . . . . . . . . . . . . . . . . . . . . . . . . . . 600
HA

Gastric Adenocarcinoma . . . . . . . . . . . . . . . . . . . . . . . . . . . . . . . 604


W

Gastric Carcinoids . . . . . . . . . . . . . . . . . . . . . . . . . . . . . . . . . . . . 605


Gastric Lymphoma . . . . . . . . . . . . . . . . . . . . . . . . . . . . . . . . . . . 606
L
IA

Gastric Stromal Tumor . . . . . . . . . . . . . . . . . . . . . . . . . . . . . . . . 607


ER

Gastroenteritis . . . . . . . . . . . . . . . . . . . . . . . . . . . . . . . . . . . . . . 608
Gastroesophageal Reflux Disease . . . . . . . . . . . . . . . . . . . . . . . . . 610
AT

Gastrointestinal Bleeding . . . . . . . . . . . . . . . . . . . . . . . . . . . . . . 613


M

Gastropathy . . . . . . . . . . . . . . . . . . . . . . . . . . . . . . . . . . . . . . . . 617
SE

Geographic Tongue . . . . . . . . . . . . . . . . . . . . . . . . . . . . . . . . . . . 621


Giant Cell Arteritis . . . . . . . . . . . . . . . . . . . . . . . . . . . . . . . . . . . 622
UR

Giardiasis . . . . . . . . . . . . . . . . . . . . . . . . . . . . . . . . . . . . . . . . . . 623
Gilbert’s Syndrome . . . . . . . . . . . . . . . . . . . . . . . . . . . . . . . . . . . 625
CO

Gitelman’s Syndrome . . . . . . . . . . . . . . . . . . . . . . . . . . . . . . . . . 626


Glaucoma . . . . . . . . . . . . . . . . . . . . . . . . . . . . . . . . . . . . . . . . . . 626
LL

Glomerular Diseases . . . . . . . . . . . . . . . . . . . . . . . . . . . . . . . . . . 630


FU

Glucocorticoid-Remediable Aldosteronism . . . . . . . . . . . . . . . . . 634


Glycogen Storage Disease . . . . . . . . . . . . . . . . . . . . . . . . . . . . . . 635
Goiter . . . . . . . . . . . . . . . . . . . . . . . . . . . . . . . . . . . . . . . . . . . . . 638
Gonorrhea . . . . . . . . . . . . . . . . . . . . . . . . . . . . . . . . . . . . . . . . . 640
P1: SBT
0521779407pre CUNY1086/Karliner 0 521 77940 7 June 27, 2007 10:22

Contents xvii

Gordon Syndrome (Pseudohypoaldosteronism Type 2) . . . . . . . . 644

10
Granuloma Annulare . . . . . . . . . . . . . . . . . . . . . . . . . . . . . . . . . 645

59
Granulomatous Liver Disease . . . . . . . . . . . . . . . . . . . . . . . . . . . 646

65
Granulomatous Vasculitis . . . . . . . . . . . . . . . . . . . . . . . . . . . . . . 647

48
Haemophilus Infections . . . . . . . . . . . . . . . . . . . . . . . . . . . . . . . 651
Hairy Cell Leukemia . . . . . . . . . . . . . . . . . . . . . . . . . . . . . . . . . . 653

88
Hantavirus Pulmonary Syndrome . . . . . . . . . . . . . . . . . . . . . . . . 654

91
Hartnup’s Disease . . . . . . . . . . . . . . . . . . . . . . . . . . . . . . . . . . . . 656

00
Head and Neck Cancer . . . . . . . . . . . . . . . . . . . . . . . . . . . . . . . . 657
Head Trauma . . . . . . . . . . . . . . . . . . . . . . . . . . . . . . . . . . . . . . . 662

AP
Hearing Loss . . . . . . . . . . . . . . . . . . . . . . . . . . . . . . . . . . . . . . . . 663
TS
Helicobacter Pylori . . . . . . . . . . . . . . . . . . . . . . . . . . . . . . . . . . . 665
Hemiballismus . . . . . . . . . . . . . . . . . . . . . . . . . . . . . . . . . . . . . . 669
HA

Hemochromatosis . . . . . . . . . . . . . . . . . . . . . . . . . . . . . . . . . . . 670
W

Hemophilia A and B . . . . . . . . . . . . . . . . . . . . . . . . . . . . . . . . . . 673


Hepatic Encephalopathy . . . . . . . . . . . . . . . . . . . . . . . . . . . . . . . 677
L
IA

Hepatic Veno-occlusive Disease . . . . . . . . . . . . . . . . . . . . . . . . . 680


ER

Hepatitis A and E . . . . . . . . . . . . . . . . . . . . . . . . . . . . . . . . . . . . 682


Hepatitis B . . . . . . . . . . . . . . . . . . . . . . . . . . . . . . . . . . . . . . . . . 685
AT

Hepatitis C . . . . . . . . . . . . . . . . . . . . . . . . . . . . . . . . . . . . . . . . . 688
M

Hepatitis Delta Virus . . . . . . . . . . . . . . . . . . . . . . . . . . . . . . . . . . 693


SE

Hepatorenal Syndrome . . . . . . . . . . . . . . . . . . . . . . . . . . . . . . . . 694


Hereditary Elliptocytosis (HE) and Hereditary
UR

Pyropoikilocytosis (HPP) . . . . . . . . . . . . . . . . . . . . . . . . . . . . . . . 696


Hereditary Spherocytosis . . . . . . . . . . . . . . . . . . . . . . . . . . . . . . 698
CO

Hernia . . . . . . . . . . . . . . . . . . . . . . . . . . . . . . . . . . . . . . . . . . . . 700
Herpes Labialis . . . . . . . . . . . . . . . . . . . . . . . . . . . . . . . . . . . . . . 701
LL

Herpes Simplex . . . . . . . . . . . . . . . . . . . . . . . . . . . . . . . . . . . . . . 702


FU

Herpes Type 1/Type 2 . . . . . . . . . . . . . . . . . . . . . . . . . . . . . . . . . 703


Herpes Zoster . . . . . . . . . . . . . . . . . . . . . . . . . . . . . . . . . . . . . . . 707
Hidradenitis Suppurativa . . . . . . . . . . . . . . . . . . . . . . . . . . . . . . 708
High-Oxygen-Affinity Hemoglobins . . . . . . . . . . . . . . . . . . . . . . . 710
P1: SBT
0521779407pre CUNY1086/Karliner 0 521 77940 7 June 27, 2007 10:22

xviii Contents

Hirsutism (Dermatology) . . . . . . . . . . . . . . . . . . . . . . . . . . . . . . 711

10
Hirsutism (Endocrinology) . . . . . . . . . . . . . . . . . . . . . . . . . . . . . 714

59
Histoplasma Capsulatum . . . . . . . . . . . . . . . . . . . . . . . . . . . . . . 718

65
Homocystinuria . . . . . . . . . . . . . . . . . . . . . . . . . . . . . . . . . . . . . 722

48
Hookworm . . . . . . . . . . . . . . . . . . . . . . . . . . . . . . . . . . . . . . . . . 725
Horner’s Syndrome . . . . . . . . . . . . . . . . . . . . . . . . . . . . . . . . . . . 727

88
Human Herpes 6 . . . . . . . . . . . . . . . . . . . . . . . . . . . . . . . . . . . . . 728

91
Human Immunodeficiency Virus Type 1 (HIV-1) . . . . . . . . . . . . . 729

00
Huntington’s Disease . . . . . . . . . . . . . . . . . . . . . . . . . . . . . . . . . 735
Hydatid Cyst Disease . . . . . . . . . . . . . . . . . . . . . . . . . . . . . . . . . 736

AP
Hydrocele . . . . . . . . . . . . . . . . . . . . . . . . . . . . . . . . . . . . . . . . . . 738
TS
Hypercalcemia . . . . . . . . . . . . . . . . . . . . . . . . . . . . . . . . . . . . . . 738
Hypercholesterolemia . . . . . . . . . . . . . . . . . . . . . . . . . . . . . . . . . 742
HA

Hyperemesis Gravidarum . . . . . . . . . . . . . . . . . . . . . . . . . . . . . . 746


W

Hyperkalemia . . . . . . . . . . . . . . . . . . . . . . . . . . . . . . . . . . . . . . . 747
Hypermagnesemia . . . . . . . . . . . . . . . . . . . . . . . . . . . . . . . . . . . 750
L
IA

Hypernatremia . . . . . . . . . . . . . . . . . . . . . . . . . . . . . . . . . . . . . . 751
ER

Hyperoxaluria . . . . . . . . . . . . . . . . . . . . . . . . . . . . . . . . . . . . . . . 754
Hyperphosphatemia . . . . . . . . . . . . . . . . . . . . . . . . . . . . . . . . . . 757
AT

Hypersensitive Carotid Syndrome and Syncope . . . . . . . . . . . . . . 759


M

Hypertension . . . . . . . . . . . . . . . . . . . . . . . . . . . . . . . . . . . . . . . 760
SE

Hyperthermia . . . . . . . . . . . . . . . . . . . . . . . . . . . . . . . . . . . . . . . 767
Hyperthyroidism . . . . . . . . . . . . . . . . . . . . . . . . . . . . . . . . . . . . 770
UR

Hypertriglyceridemia . . . . . . . . . . . . . . . . . . . . . . . . . . . . . . . . . 773
Hypocalcemia . . . . . . . . . . . . . . . . . . . . . . . . . . . . . . . . . . . . . . 777
CO

Hypoglycemia . . . . . . . . . . . . . . . . . . . . . . . . . . . . . . . . . . . . . . 780
Hypogonadism in Men . . . . . . . . . . . . . . . . . . . . . . . . . . . . . . . . 783
LL

Hypokalemia . . . . . . . . . . . . . . . . . . . . . . . . . . . . . . . . . . . . . . . 786
FU

Hypomagnesemia . . . . . . . . . . . . . . . . . . . . . . . . . . . . . . . . . . . . 789
Hyponatremia . . . . . . . . . . . . . . . . . . . . . . . . . . . . . . . . . . . . . . 791
Hypophosphatemia . . . . . . . . . . . . . . . . . . . . . . . . . . . . . . . . . . 794
Hypopituitarism . . . . . . . . . . . . . . . . . . . . . . . . . . . . . . . . . . . . . 796
P1: SBT
0521779407pre CUNY1086/Karliner 0 521 77940 7 June 27, 2007 10:22

Contents xix

Hypothermia . . . . . . . . . . . . . . . . . . . . . . . . . . . . . . . . . . . . . . . 799

10
Hypothyroidism . . . . . . . . . . . . . . . . . . . . . . . . . . . . . . . . . . . . . 802

59
Ichthyosis . . . . . . . . . . . . . . . . . . . . . . . . . . . . . . . . . . . . . . . . . . 804

65
Idiopathic (Immune) Thrombocytopenic Purpura (ITP) . . . . . . . . 808

48
Immune Hemolytic Anemia . . . . . . . . . . . . . . . . . . . . . . . . . . . . 810
Immunodeficiency Disorders, Congenital . . . . . . . . . . . . . . . . . . 818

88
Impetigo . . . . . . . . . . . . . . . . . . . . . . . . . . . . . . . . . . . . . . . . . . . 822

91
Infectious Diarrheas . . . . . . . . . . . . . . . . . . . . . . . . . . . . . . . . . . 825

00
Infective Endocarditis . . . . . . . . . . . . . . . . . . . . . . . . . . . . . . . . . 829
Influenza . . . . . . . . . . . . . . . . . . . . . . . . . . . . . . . . . . . . . . . . . . 833

AP
Influenza, Avian . . . . . . . . . . . . . . . . . . . . . . . . . . . . . . . . . . . . . 834
TS
Inguinofemoral Hernia . . . . . . . . . . . . . . . . . . . . . . . . . . . . . . . . 836
Inner Ear Disorders Producing Vertigo . . . . . . . . . . . . . . . . . . . . . 837
HA

Insomnia . . . . . . . . . . . . . . . . . . . . . . . . . . . . . . . . . . . . . . . . . . 840
W

Interstitial Lung Diseases . . . . . . . . . . . . . . . . . . . . . . . . . . . . . . 841


Intertrigo . . . . . . . . . . . . . . . . . . . . . . . . . . . . . . . . . . . . . . . . . . 845
L
IA

Intervertebral Disc Disease . . . . . . . . . . . . . . . . . . . . . . . . . . . . . 847


ER

Intestinal Flukes . . . . . . . . . . . . . . . . . . . . . . . . . . . . . . . . . . . . . 848


Intestinal Motor Disorders . . . . . . . . . . . . . . . . . . . . . . . . . . . . . 850
AT

Intracranial Hypertension . . . . . . . . . . . . . . . . . . . . . . . . . . . . . . 853


M

Intracranial Hypotension . . . . . . . . . . . . . . . . . . . . . . . . . . . . . . 854


SE

Intraocular Infection . . . . . . . . . . . . . . . . . . . . . . . . . . . . . . . . . . 855


Intraocular Tumors . . . . . . . . . . . . . . . . . . . . . . . . . . . . . . . . . . . 859
UR

Intravascular, Non-immune Hemolytic Anemia –


Microangiopathic Hemolytic Anemias . . . . . . . . . . . . . . . . . . . . . 860
CO

Iron Deficiency . . . . . . . . . . . . . . . . . . . . . . . . . . . . . . . . . . . . . . 866


Irritable Bowel Syndrome . . . . . . . . . . . . . . . . . . . . . . . . . . . . . . 870
LL

Ischemic Bowel Disease . . . . . . . . . . . . . . . . . . . . . . . . . . . . . . . 874


FU

Isosporiasis . . . . . . . . . . . . . . . . . . . . . . . . . . . . . . . . . . . . . . . . 877
Jaw Swelling and Masses . . . . . . . . . . . . . . . . . . . . . . . . . . . . . . . 878
Keloids . . . . . . . . . . . . . . . . . . . . . . . . . . . . . . . . . . . . . . . . . . . . 882
Laceration or Myocardial Perforation . . . . . . . . . . . . . . . . . . . . . . 883
P1: SBT
0521779407pre CUNY1086/Karliner 0 521 77940 7 June 27, 2007 17:50

xx Contents

Laryngotracheitis . . . . . . . . . . . . . . . . . . . . . . . . . . . . . . . . . . . . 884

10
Laxative Abuse . . . . . . . . . . . . . . . . . . . . . . . . . . . . . . . . . . . . . . 885

59
Legionella Infections . . . . . . . . . . . . . . . . . . . . . . . . . . . . . . . . . . 887

65
Leishmaniasis, Cutaneous . . . . . . . . . . . . . . . . . . . . . . . . . . . . . . 889

48
Leishmaniasis, Visceral . . . . . . . . . . . . . . . . . . . . . . . . . . . . . . . . 891
Leptospirosis, Relapsing Fever and Rat-bite Fever . . . . . . . . . . . . 893

88
Leukocytosis: Neutrophil . . . . . . . . . . . . . . . . . . . . . . . . . . . . . . 896

91
Leukopenia . . . . . . . . . . . . . . . . . . . . . . . . . . . . . . . . . . . . . . . . 900

00
Leukoplakia . . . . . . . . . . . . . . . . . . . . . . . . . . . . . . . . . . . . . . . . 904
Lichen Planus . . . . . . . . . . . . . . . . . . . . . . . . . . . . . . . . . . . . . . . 905

AP
Liddle’s Syndrome . . . . . . . . . . . . . . . . . . . . . . . . . . . . . . . . . . . 907
TS
Liver Fluke Infections . . . . . . . . . . . . . . . . . . . . . . . . . . . . . . . . . 907
Liver Flukes . . . . . . . . . . . . . . . . . . . . . . . . . . . . . . . . . . . . . . . . 909
HA

Liver Transplantation . . . . . . . . . . . . . . . . . . . . . . . . . . . . . . . . . 911


W

Localized Scleroderma . . . . . . . . . . . . . . . . . . . . . . . . . . . . . . . . 915


Low-Oxygen-Affinity Hemoglobins . . . . . . . . . . . . . . . . . . . . . . . 917
L
IA

Lung Abscess . . . . . . . . . . . . . . . . . . . . . . . . . . . . . . . . . . . . . . . 918


ER

Lung Cancer . . . . . . . . . . . . . . . . . . . . . . . . . . . . . . . . . . . . . . . . 921


Lung Fluke . . . . . . . . . . . . . . . . . . . . . . . . . . . . . . . . . . . . . . . . . 925
AT

Lyme Disease . . . . . . . . . . . . . . . . . . . . . . . . . . . . . . . . . . . . . . . 927


M

Lymphadenitis and Lymphangitis . . . . . . . . . . . . . . . . . . . . . . . . 930


SE

Lymphomas . . . . . . . . . . . . . . . . . . . . . . . . . . . . . . . . . . . . . . . . 932
Lysosomal Diseases . . . . . . . . . . . . . . . . . . . . . . . . . . . . . . . . . . 937
UR

Magnesium Deficiency . . . . . . . . . . . . . . . . . . . . . . . . . . . . . . . . 940


Malaria . . . . . . . . . . . . . . . . . . . . . . . . . . . . . . . . . . . . . . . . . . . . 942
CO

Malignant Tumors of The Liver . . . . . . . . . . . . . . . . . . . . . . . . . . 947


Marasmus . . . . . . . . . . . . . . . . . . . . . . . . . . . . . . . . . . . . . . . . . 949
LL

Marfan’s Syndrome . . . . . . . . . . . . . . . . . . . . . . . . . . . . . . . . . . . 952


FU

Measles . . . . . . . . . . . . . . . . . . . . . . . . . . . . . . . . . . . . . . . . . . . 954
Mediastinal Masses . . . . . . . . . . . . . . . . . . . . . . . . . . . . . . . . . . . 957
Megacolon . . . . . . . . . . . . . . . . . . . . . . . . . . . . . . . . . . . . . . . . . 960
Melanoma . . . . . . . . . . . . . . . . . . . . . . . . . . . . . . . . . . . . . . . . . 964
P1: SBT
0521779407pre CUNY1086/Karliner 0 521 77940 7 June 27, 2007 10:22

Contents xxi

Menopause . . . . . . . . . . . . . . . . . . . . . . . . . . . . . . . . . . . . . . . . 967

10
Menstrual Cycle Disorders for the Generalist . . . . . . . . . . . . . . . . 970

59
Metabolic Acidosis . . . . . . . . . . . . . . . . . . . . . . . . . . . . . . . . . . . 973

65
Metabolic Alkalosis . . . . . . . . . . . . . . . . . . . . . . . . . . . . . . . . . . . 977

48
Methemoglobinemia . . . . . . . . . . . . . . . . . . . . . . . . . . . . . . . . . 980
Migraine Headache . . . . . . . . . . . . . . . . . . . . . . . . . . . . . . . . . . . 982

88
Miliaria . . . . . . . . . . . . . . . . . . . . . . . . . . . . . . . . . . . . . . . . . . . 984

91
Mineralocorticoid Disorders . . . . . . . . . . . . . . . . . . . . . . . . . . . . 986

00
Miscellaneous Intestinal Protozoa . . . . . . . . . . . . . . . . . . . . . . . . 989
Mitral Insufficiency (MR) . . . . . . . . . . . . . . . . . . . . . . . . . . . . . . 990

AP
Mitral Stenosis (MS) . . . . . . . . . . . . . . . . . . . . . . . . . . . . . . . . . . 993
TS
Mitral Valve Prolapse . . . . . . . . . . . . . . . . . . . . . . . . . . . . . . . . . . 996
Molluscum Contagiosum . . . . . . . . . . . . . . . . . . . . . . . . . . . . . . 997
HA

Motor Neuron Diseases . . . . . . . . . . . . . . . . . . . . . . . . . . . . . . . . 998


W

Mucopolysaccharidoses . . . . . . . . . . . . . . . . . . . . . . . . . . . . . . 1000
Mucormycosis . . . . . . . . . . . . . . . . . . . . . . . . . . . . . . . . . . . . . 1004
L
IA

Multifocal Atrial Tachycardia (MAT) . . . . . . . . . . . . . . . . . . . . . . 1007


ER

Multiple Endocrine Neoplasia 1 . . . . . . . . . . . . . . . . . . . . . . . . . 1008


Multiple Endocrine Neoplasia 2 . . . . . . . . . . . . . . . . . . . . . . . . . 1012
AT

Multiple Sclerosis . . . . . . . . . . . . . . . . . . . . . . . . . . . . . . . . . . . 1016


M

Mumps . . . . . . . . . . . . . . . . . . . . . . . . . . . . . . . . . . . . . . . . . . . 1017
SE

Muscular Dystrophies . . . . . . . . . . . . . . . . . . . . . . . . . . . . . . . . 1020


Musculoskeletal Problems . . . . . . . . . . . . . . . . . . . . . . . . . . . . . 1021
UR

Myasthenia Gravis . . . . . . . . . . . . . . . . . . . . . . . . . . . . . . . . . . . 1025


Myasthenic (Lambert-Eaton) Syndrome . . . . . . . . . . . . . . . . . . . 1027
CO

Myelodysplastic Syndrome . . . . . . . . . . . . . . . . . . . . . . . . . . . . 1028


Myeloma and Gammopathies . . . . . . . . . . . . . . . . . . . . . . . . . . 1033
LL

Myeloproliferative Disorders . . . . . . . . . . . . . . . . . . . . . . . . . . . 1037


FU

Myoclonus . . . . . . . . . . . . . . . . . . . . . . . . . . . . . . . . . . . . . . . . 1039
Narcolepsy . . . . . . . . . . . . . . . . . . . . . . . . . . . . . . . . . . . . . . . . 1040
Nephrogenic Diabetes Insipidus . . . . . . . . . . . . . . . . . . . . . . . . 1042
Neurofibromatosis . . . . . . . . . . . . . . . . . . . . . . . . . . . . . . . . . . 1043
P1: SBT
0521779407pre CUNY1086/Karliner 0 521 77940 7 June 27, 2007 10:22

xxii Contents

Nevi and Pigmented Lesions . . . . . . . . . . . . . . . . . . . . . . . . . . . 1047

10
Niacin Deficiency . . . . . . . . . . . . . . . . . . . . . . . . . . . . . . . . . . . 1050

59
Nocardiosis . . . . . . . . . . . . . . . . . . . . . . . . . . . . . . . . . . . . . . . . 1051

65
Nonalcoholic Fatty Liver Disease . . . . . . . . . . . . . . . . . . . . . . . . 1053

48
Nongranulomatous Systemic Vasculitis . . . . . . . . . . . . . . . . . . . 1056
Nonmelanoma Skin Cancers: Basal Cell Carcinoma . . . . . . . . . . 1060

88
Nonmelanoma Skin Cancers: Squamous Cell Carcinoma . . . . . . 1063

91
Non-steroidal Anti-Inflammatory Drugs . . . . . . . . . . . . . . . . . . 1066

00
Nontuberculous Mycobacterial Infections . . . . . . . . . . . . . . . . . 1069
Obesity . . . . . . . . . . . . . . . . . . . . . . . . . . . . . . . . . . . . . . . . . . . 1071

AP
Obstructive Sleep Apnea . . . . . . . . . . . . . . . . . . . . . . . . . . . . . . 1075
TS
Occupational Pulmonary Disease . . . . . . . . . . . . . . . . . . . . . . . 1078
Onychomycosis – Tinea Unguium . . . . . . . . . . . . . . . . . . . . . . . 1083
HA

Oral Cancer . . . . . . . . . . . . . . . . . . . . . . . . . . . . . . . . . . . . . . . 1085


W

Oral Lichen Planus . . . . . . . . . . . . . . . . . . . . . . . . . . . . . . . . . . 1086


Orbital Cellulitis . . . . . . . . . . . . . . . . . . . . . . . . . . . . . . . . . . . . 1087
L
IA

Orbital Tumors . . . . . . . . . . . . . . . . . . . . . . . . . . . . . . . . . . . . . 1088


ER

Orchitis and Epididymitis . . . . . . . . . . . . . . . . . . . . . . . . . . . . . 1089


Osteoarthritis . . . . . . . . . . . . . . . . . . . . . . . . . . . . . . . . . . . . . . 1092
AT

Osteogenesis Imperfecta . . . . . . . . . . . . . . . . . . . . . . . . . . . . . . 1096


M

Osteomalacia and Rickets . . . . . . . . . . . . . . . . . . . . . . . . . . . . . 1099


SE

Osteomyelitis . . . . . . . . . . . . . . . . . . . . . . . . . . . . . . . . . . . . . . 1103
Osteonecrosis . . . . . . . . . . . . . . . . . . . . . . . . . . . . . . . . . . . . . . 1105
UR

Osteoporosis . . . . . . . . . . . . . . . . . . . . . . . . . . . . . . . . . . . . . . . 1108
Other Cardiomyopathies . . . . . . . . . . . . . . . . . . . . . . . . . . . . . . 1112
CO

Other Clotting Factor Deficiencies . . . . . . . . . . . . . . . . . . . . . . . 1114


Otitis Externa . . . . . . . . . . . . . . . . . . . . . . . . . . . . . . . . . . . . . . 1118
LL

Otitis Media . . . . . . . . . . . . . . . . . . . . . . . . . . . . . . . . . . . . . . . 1120


FU

Paget Disease . . . . . . . . . . . . . . . . . . . . . . . . . . . . . . . . . . . . . . 1122


Pain Syndromes . . . . . . . . . . . . . . . . . . . . . . . . . . . . . . . . . . . . 1124
Pancreatic Cancer . . . . . . . . . . . . . . . . . . . . . . . . . . . . . . . . . . . 1125
Pancreatic Cysts . . . . . . . . . . . . . . . . . . . . . . . . . . . . . . . . . . . . 1128
P1: SBT
0521779407pre CUNY1086/Karliner 0 521 77940 7 June 27, 2007 10:22

Contents xxiii

Papillary Muscle Dysfunction and Rupture . . . . . . . . . . . . . . . . . 1130

10
Paracoccidioidomycosis . . . . . . . . . . . . . . . . . . . . . . . . . . . . . . 1132

59
Parainfluenza . . . . . . . . . . . . . . . . . . . . . . . . . . . . . . . . . . . . . . 1134

65
Paralytic Poliomyelitis . . . . . . . . . . . . . . . . . . . . . . . . . . . . . . . . 1136

48
Parkinson’s Disease and Parkinsonism . . . . . . . . . . . . . . . . . . . . 1137
Paronychia . . . . . . . . . . . . . . . . . . . . . . . . . . . . . . . . . . . . . . . . 1139

88
Parvovirus B19 . . . . . . . . . . . . . . . . . . . . . . . . . . . . . . . . . . . . . 1140

91
Patent Ductus Arteriosus . . . . . . . . . . . . . . . . . . . . . . . . . . . . . . 1142

00
Pediculosis . . . . . . . . . . . . . . . . . . . . . . . . . . . . . . . . . . . . . . . . 1144
Pelvic Inflammatory Disease . . . . . . . . . . . . . . . . . . . . . . . . . . . 1145

AP
Pemphigus Vulgaris and Pemphigus Foliaceus . . . . . . . . . . . . . . 1149
TS
Peptic Ulcer Disease . . . . . . . . . . . . . . . . . . . . . . . . . . . . . . . . . 1151
Pericardial Tamponade . . . . . . . . . . . . . . . . . . . . . . . . . . . . . . . 1154
HA

Peripheral Neuropathies . . . . . . . . . . . . . . . . . . . . . . . . . . . . . . 1155


W

Perirectal Abscesses and Fistulas . . . . . . . . . . . . . . . . . . . . . . . . 1157


Peritoneal Tumors . . . . . . . . . . . . . . . . . . . . . . . . . . . . . . . . . . . 1159
L
IA

Peritonitis . . . . . . . . . . . . . . . . . . . . . . . . . . . . . . . . . . . . . . . . . 1161
ER

Persistent Vegetative State . . . . . . . . . . . . . . . . . . . . . . . . . . . . . 1163


Pharyngitis . . . . . . . . . . . . . . . . . . . . . . . . . . . . . . . . . . . . . . . . 1164
AT

Pheochromocytoma . . . . . . . . . . . . . . . . . . . . . . . . . . . . . . . . . 1165
M

Phimosis and Paraphimosis . . . . . . . . . . . . . . . . . . . . . . . . . . . . 1169


SE

Phosphate Deficiency . . . . . . . . . . . . . . . . . . . . . . . . . . . . . . . . 1171


Photosensitivity . . . . . . . . . . . . . . . . . . . . . . . . . . . . . . . . . . . . 1172
UR

Pituitary Tumors . . . . . . . . . . . . . . . . . . . . . . . . . . . . . . . . . . . . 1174


Pityriasis Rosea . . . . . . . . . . . . . . . . . . . . . . . . . . . . . . . . . . . . . 1177
CO

Pleural Diseases: Effusion/Empyema . . . . . . . . . . . . . . . . . . . . . 1178


Pleural Tumors . . . . . . . . . . . . . . . . . . . . . . . . . . . . . . . . . . . . . 1183
LL

Pneumothorax . . . . . . . . . . . . . . . . . . . . . . . . . . . . . . . . . . . . . 1186
FU

Polymyositis and Related Disorders . . . . . . . . . . . . . . . . . . . . . . 1189


Porphyria, Acute . . . . . . . . . . . . . . . . . . . . . . . . . . . . . . . . . . . . 1193
Portal Hypertensive Bleeding . . . . . . . . . . . . . . . . . . . . . . . . . . . 1195
Postconcussion Syndrome or Postconcussive Syndrome . . . . . . . 1198
P1: SBT
0521779407pre CUNY1086/Karliner 0 521 77940 7 June 27, 2007 10:22

xxiv Contents

Preeclamptic Liver Disease/HELLP . . . . . . . . . . . . . . . . . . . . . . 1199

10
Pregnancy Complications for the Internist . . . . . . . . . . . . . . . . . 1201

59
Pressure Ulcers . . . . . . . . . . . . . . . . . . . . . . . . . . . . . . . . . . . . . 1204

65
Priapism . . . . . . . . . . . . . . . . . . . . . . . . . . . . . . . . . . . . . . . . . . 1205

48
Primary Biliary Cirrhosis . . . . . . . . . . . . . . . . . . . . . . . . . . . . . . 1209
Primary Hyperparathyroidism . . . . . . . . . . . . . . . . . . . . . . . . . . 1211

88
Primary Sclerosing Cholangitis . . . . . . . . . . . . . . . . . . . . . . . . . 1214

91
Prion Disorders (Creutzfeldt-Jakob Disease) . . . . . . . . . . . . . . . . 1217

00
Prolactinoma and Galactorrhea . . . . . . . . . . . . . . . . . . . . . . . . . 1219
Prostate Cancer . . . . . . . . . . . . . . . . . . . . . . . . . . . . . . . . . . . . . 1222

AP
Prostatitis . . . . . . . . . . . . . . . . . . . . . . . . . . . . . . . . . . . . . . . . . 1225
TS
Protein-Losing Enteropathy . . . . . . . . . . . . . . . . . . . . . . . . . . . . 1227
Pruritus . . . . . . . . . . . . . . . . . . . . . . . . . . . . . . . . . . . . . . . . . . 1228
HA

Pseudohypoaldosteronism . . . . . . . . . . . . . . . . . . . . . . . . . . . . 1230
W

Pseudomonas Infections . . . . . . . . . . . . . . . . . . . . . . . . . . . . . . 1231


Psittacosis . . . . . . . . . . . . . . . . . . . . . . . . . . . . . . . . . . . . . . . . . 1234
L
IA

Psoriasis . . . . . . . . . . . . . . . . . . . . . . . . . . . . . . . . . . . . . . . . . . 1236
ER

Pulmonary Embolism . . . . . . . . . . . . . . . . . . . . . . . . . . . . . . . . 1240


Pulmonary Hypertension . . . . . . . . . . . . . . . . . . . . . . . . . . . . . 1244
AT

Pulmonary Stenosis (PS) . . . . . . . . . . . . . . . . . . . . . . . . . . . . . . 1249


M

Pulmonary Valve Insufficiency (PI) . . . . . . . . . . . . . . . . . . . . . . . 1250


SE

Purine and Pyrimidine Metabolic Disorders . . . . . . . . . . . . . . . . 1252


Pyogenic Granuloma . . . . . . . . . . . . . . . . . . . . . . . . . . . . . . . . . 1256
UR

Pyogenic Liver Abscess . . . . . . . . . . . . . . . . . . . . . . . . . . . . . . . 1257


Pyridoxine Deficiency . . . . . . . . . . . . . . . . . . . . . . . . . . . . . . . . 1258
CO

Rabies . . . . . . . . . . . . . . . . . . . . . . . . . . . . . . . . . . . . . . . . . . . 1259
Radiation Enteritis and Colitis . . . . . . . . . . . . . . . . . . . . . . . . . . 1262
LL

Raynaud’s Syndrome . . . . . . . . . . . . . . . . . . . . . . . . . . . . . . . . . 1265


FU

Recurrent Aphthous Stomatitis . . . . . . . . . . . . . . . . . . . . . . . . . 1269


Red Cell Enzymes . . . . . . . . . . . . . . . . . . . . . . . . . . . . . . . . . . . 1270
Refractive Disorders (Ametropias) . . . . . . . . . . . . . . . . . . . . . . . 1272
Renal Artery Stenosis . . . . . . . . . . . . . . . . . . . . . . . . . . . . . . . . . 1277
P1: SBT
0521779407pre CUNY1086/Karliner 0 521 77940 7 June 27, 2007 10:22

Contents xxv

Renal Calculi . . . . . . . . . . . . . . . . . . . . . . . . . . . . . . . . . . . . . . . 1280

10
Renal Glucosuria . . . . . . . . . . . . . . . . . . . . . . . . . . . . . . . . . . . . 1284

59
Renal Masses and Tumors . . . . . . . . . . . . . . . . . . . . . . . . . . . . . 1285

65
Renal Osteodystrophy . . . . . . . . . . . . . . . . . . . . . . . . . . . . . . . . 1289

48
Renal Vein Thrombosis . . . . . . . . . . . . . . . . . . . . . . . . . . . . . . . 1292
Repaired Congenital Heart Disease . . . . . . . . . . . . . . . . . . . . . . 1294

88
Restless Legs Syndrome . . . . . . . . . . . . . . . . . . . . . . . . . . . . . . . 1295

91
Rhegmatogenous Retinal Detachment . . . . . . . . . . . . . . . . . . . . 1296

00
Rheumatoid Arthritis . . . . . . . . . . . . . . . . . . . . . . . . . . . . . . . . 1299
Rhinitis . . . . . . . . . . . . . . . . . . . . . . . . . . . . . . . . . . . . . . . . . . . 1303

AP
Rhinovirus . . . . . . . . . . . . . . . . . . . . . . . . . . . . . . . . . . . . . . . . 1306
TS
Riboflavin Deficiency . . . . . . . . . . . . . . . . . . . . . . . . . . . . . . . . 1307
Rocky Mountain Spotted Fever . . . . . . . . . . . . . . . . . . . . . . . . . 1308
HA

Rosacea . . . . . . . . . . . . . . . . . . . . . . . . . . . . . . . . . . . . . . . . . . 1311
W

Rotor Syndrome . . . . . . . . . . . . . . . . . . . . . . . . . . . . . . . . . . . . 1312


RSV/Respiratory Syncytial Virus . . . . . . . . . . . . . . . . . . . . . . . . 1313
L
IA

Rubella . . . . . . . . . . . . . . . . . . . . . . . . . . . . . . . . . . . . . . . . . . . 1314
ER

Salmonella Infections Other than Gastroenteritis . . . . . . . . . . . . 1316


Sarcoidosis . . . . . . . . . . . . . . . . . . . . . . . . . . . . . . . . . . . . . . . . 1319
AT

Scabies . . . . . . . . . . . . . . . . . . . . . . . . . . . . . . . . . . . . . . . . . . . 1322
M

Schistosomiasis . . . . . . . . . . . . . . . . . . . . . . . . . . . . . . . . . . . . 1324
SE

Scleroderma . . . . . . . . . . . . . . . . . . . . . . . . . . . . . . . . . . . . . . . 1328
Seborrheic Dermatitis . . . . . . . . . . . . . . . . . . . . . . . . . . . . . . . . 1331
UR

Seborrheic Keratosis . . . . . . . . . . . . . . . . . . . . . . . . . . . . . . . . . 1332


Selenium Deficiency . . . . . . . . . . . . . . . . . . . . . . . . . . . . . . . . . 1333
CO

Severe Acute Respiratory Syndrome . . . . . . . . . . . . . . . . . . . . . . 1333


Sex Differentiation Disorders . . . . . . . . . . . . . . . . . . . . . . . . . . . 1335
LL

Shock . . . . . . . . . . . . . . . . . . . . . . . . . . . . . . . . . . . . . . . . . . . . 1339
FU

Short Bowel Syndrome . . . . . . . . . . . . . . . . . . . . . . . . . . . . . . . 1343


Sickle Cell Syndromes . . . . . . . . . . . . . . . . . . . . . . . . . . . . . . . . 1345
Sigmoid Volvulus . . . . . . . . . . . . . . . . . . . . . . . . . . . . . . . . . . . 1348
Sinoatrial Block . . . . . . . . . . . . . . . . . . . . . . . . . . . . . . . . . . . . . 1349
P1: SBT
0521779407pre CUNY1086/Karliner 0 521 77940 7 June 27, 2007 10:22

xxvi Contents

Sinusitis . . . . . . . . . . . . . . . . . . . . . . . . . . . . . . . . . . . . . . . . . . 1351

10
Small Bowel Tumors . . . . . . . . . . . . . . . . . . . . . . . . . . . . . . . . . 1356

59
Spermatocele . . . . . . . . . . . . . . . . . . . . . . . . . . . . . . . . . . . . . . 1359

65
Spine & Spinal Cord Injury . . . . . . . . . . . . . . . . . . . . . . . . . . . . . 1360

48
Spondyloarthropathies . . . . . . . . . . . . . . . . . . . . . . . . . . . . . . . 1361
Spontaneous Bacterial Peritonitis . . . . . . . . . . . . . . . . . . . . . . . 1364

88
Sporotrichosis . . . . . . . . . . . . . . . . . . . . . . . . . . . . . . . . . . . . . . 1368

91
Staphylococcal Infections . . . . . . . . . . . . . . . . . . . . . . . . . . . . . 1370

00
Stasis Dermatitis . . . . . . . . . . . . . . . . . . . . . . . . . . . . . . . . . . . . 1375
Status Epilepticus . . . . . . . . . . . . . . . . . . . . . . . . . . . . . . . . . . . 1376

AP
Strabismus . . . . . . . . . . . . . . . . . . . . . . . . . . . . . . . . . . . . . . . . 1377
TS
Streptococcal Infections . . . . . . . . . . . . . . . . . . . . . . . . . . . . . . 1380
Strongyloidiasis . . . . . . . . . . . . . . . . . . . . . . . . . . . . . . . . . . . . 1385
HA

Subarachnoid Hemorrhage (SAH) . . . . . . . . . . . . . . . . . . . . . . . 1388


W

Sunburn . . . . . . . . . . . . . . . . . . . . . . . . . . . . . . . . . . . . . . . . . . 1390
Superficial Thrombophlebitis . . . . . . . . . . . . . . . . . . . . . . . . . . 1391
L
IA

Syncope . . . . . . . . . . . . . . . . . . . . . . . . . . . . . . . . . . . . . . . . . . 1393
ER

Syphilis . . . . . . . . . . . . . . . . . . . . . . . . . . . . . . . . . . . . . . . . . . . 1394
Systemic Lupus Erythematosus . . . . . . . . . . . . . . . . . . . . . . . . . 1399
AT

Takayasu Arteritis . . . . . . . . . . . . . . . . . . . . . . . . . . . . . . . . . . . 1403


M

Tapeworm Infections . . . . . . . . . . . . . . . . . . . . . . . . . . . . . . . . 1404


SE

Tardive Dyskinesia . . . . . . . . . . . . . . . . . . . . . . . . . . . . . . . . . . 1406


Tension-Type Headache . . . . . . . . . . . . . . . . . . . . . . . . . . . . . . 1408
UR

Testis Tumors . . . . . . . . . . . . . . . . . . . . . . . . . . . . . . . . . . . . . . 1409


Tetanus . . . . . . . . . . . . . . . . . . . . . . . . . . . . . . . . . . . . . . . . . . . 1412
CO

Thalassemia . . . . . . . . . . . . . . . . . . . . . . . . . . . . . . . . . . . . . . . 1414
Thiamine Deficiency . . . . . . . . . . . . . . . . . . . . . . . . . . . . . . . . . 1418
LL

Thoracic Aortic Aneurysm . . . . . . . . . . . . . . . . . . . . . . . . . . . . . 1419


FU

Thrombophilia . . . . . . . . . . . . . . . . . . . . . . . . . . . . . . . . . . . . . 1421
Thyroid Nodules and Cancer . . . . . . . . . . . . . . . . . . . . . . . . . . . 1426
Tics . . . . . . . . . . . . . . . . . . . . . . . . . . . . . . . . . . . . . . . . . . . . . 1429
Tinea Capitis . . . . . . . . . . . . . . . . . . . . . . . . . . . . . . . . . . . . . . . 1430
P1: SBT
0521779407pre CUNY1086/Karliner 0 521 77940 7 June 27, 2007 10:22

Contents xxvii

Tinea Cruris/Corporis/Pedis . . . . . . . . . . . . . . . . . . . . . . . . . . . 1431

10
Torsion of Appendix Testis . . . . . . . . . . . . . . . . . . . . . . . . . . . . . 1433

59
Torsion Of Testis And Spermatic Cord . . . . . . . . . . . . . . . . . . . . 1434

65
Torticollis . . . . . . . . . . . . . . . . . . . . . . . . . . . . . . . . . . . . . . . . . 1435

48
Tourette Syndrome . . . . . . . . . . . . . . . . . . . . . . . . . . . . . . . . . . 1436
Toxoplasmosis . . . . . . . . . . . . . . . . . . . . . . . . . . . . . . . . . . . . . 1437

88
Transfusion Reactions . . . . . . . . . . . . . . . . . . . . . . . . . . . . . . . . 1439

91
Transient Ischemic Attacks . . . . . . . . . . . . . . . . . . . . . . . . . . . . 1443

00
Trichinosis . . . . . . . . . . . . . . . . . . . . . . . . . . . . . . . . . . . . . . . . 1444
Trichuriasis . . . . . . . . . . . . . . . . . . . . . . . . . . . . . . . . . . . . . . . . 1446

AP
Tricuspid Valve Insufficiency (TR) . . . . . . . . . . . . . . . . . . . . . . . 1448
TS
Tricuspid Valve Stenosis (TS) . . . . . . . . . . . . . . . . . . . . . . . . . . . 1450
Trousseau Syndrome . . . . . . . . . . . . . . . . . . . . . . . . . . . . . . . . . 1452
HA

Trypanosomiasis, African . . . . . . . . . . . . . . . . . . . . . . . . . . . . . 1454


W

Trypanosomiasis, American (Chagas’ Disease) . . . . . . . . . . . . . . 1457


Tuberculosis . . . . . . . . . . . . . . . . . . . . . . . . . . . . . . . . . . . . . . . 1459
L
IA

Tubulointerstitial Renal Disease . . . . . . . . . . . . . . . . . . . . . . . . . 1462


ER

Tumors of Paratesticular and Spermatic Cord Structures . . . . . . . 1470


Tumors, Intracranial . . . . . . . . . . . . . . . . . . . . . . . . . . . . . . . . . 1470
AT

Tumors, Spinal . . . . . . . . . . . . . . . . . . . . . . . . . . . . . . . . . . . . . 1472


M

Typhus Fevers . . . . . . . . . . . . . . . . . . . . . . . . . . . . . . . . . . . . . . 1473


SE

Ulcerative Colitis . . . . . . . . . . . . . . . . . . . . . . . . . . . . . . . . . . . . 1477


Unstable Hemoglobins . . . . . . . . . . . . . . . . . . . . . . . . . . . . . . . 1481
UR

Upper Urinary Tract Obstruction . . . . . . . . . . . . . . . . . . . . . . . . 1484


Urethritis . . . . . . . . . . . . . . . . . . . . . . . . . . . . . . . . . . . . . . . . . 1488
CO

Urinary Incontinence . . . . . . . . . . . . . . . . . . . . . . . . . . . . . . . . 1491


Urticaria and Angioedema (Dermatology) . . . . . . . . . . . . . . . . . 1495
LL

Urticaria and Angioedema (Rheumatology) . . . . . . . . . . . . . . . . 1498


FU

Uveitis . . . . . . . . . . . . . . . . . . . . . . . . . . . . . . . . . . . . . . . . . . . 1502
Vaginitis . . . . . . . . . . . . . . . . . . . . . . . . . . . . . . . . . . . . . . . . . . 1506
Valvular Heart Disease Aortic Stenosis (AS) . . . . . . . . . . . . . . . . . 1510
Valvular Injury . . . . . . . . . . . . . . . . . . . . . . . . . . . . . . . . . . . . . 1513
P1: SBT
0521779407pre CUNY1086/Karliner 0 521 77940 7 June 27, 2007 10:22

xxviii Contents

Varicella-Zoster . . . . . . . . . . . . . . . . . . . . . . . . . . . . . . . . . . . . . 1514

10
Varicocele . . . . . . . . . . . . . . . . . . . . . . . . . . . . . . . . . . . . . . . . . 1516

59
Varicose Veins . . . . . . . . . . . . . . . . . . . . . . . . . . . . . . . . . . . . . . 1517

65
Vascular Disease of Spinal Cord . . . . . . . . . . . . . . . . . . . . . . . . . 1519

48
Ventilator Management in the ICU . . . . . . . . . . . . . . . . . . . . . . . 1520
Ventricular Fibrillation (VF) and Sudden Death . . . . . . . . . . . . . 1524

88
Ventricular Premature Complexes . . . . . . . . . . . . . . . . . . . . . . . 1526

91
Ventricular Septal Defect (VSD) . . . . . . . . . . . . . . . . . . . . . . . . . 1527

00
Ventricular Tachycardia . . . . . . . . . . . . . . . . . . . . . . . . . . . . . . . 1528
Viral Arthritis . . . . . . . . . . . . . . . . . . . . . . . . . . . . . . . . . . . . . . 1531

AP
Visceral Larva Migrans . . . . . . . . . . . . . . . . . . . . . . . . . . . . . . . 1534
TS
Vitamin A Deficiency . . . . . . . . . . . . . . . . . . . . . . . . . . . . . . . . . 1535
Vitamin C Deficiency . . . . . . . . . . . . . . . . . . . . . . . . . . . . . . . . 1536
HA

Vitamin E Deficiency . . . . . . . . . . . . . . . . . . . . . . . . . . . . . . . . . 1537


W

Vitamin K Deficiency . . . . . . . . . . . . . . . . . . . . . . . . . . . . . . . . . 1538


Vitiligo . . . . . . . . . . . . . . . . . . . . . . . . . . . . . . . . . . . . . . . . . . . 1539
L
IA

Volume Depletion and Edema . . . . . . . . . . . . . . . . . . . . . . . . . . 1542


ER

Von Willebrand Disease . . . . . . . . . . . . . . . . . . . . . . . . . . . . . . . 1546


Waldenstrom’s Macroglobulinemia . . . . . . . . . . . . . . . . . . . . . . 1548
AT

Warts . . . . . . . . . . . . . . . . . . . . . . . . . . . . . . . . . . . . . . . . . . . . 1549
M

Whipple’s Disease . . . . . . . . . . . . . . . . . . . . . . . . . . . . . . . . . . . 1552


SE

Wilson’s Disease . . . . . . . . . . . . . . . . . . . . . . . . . . . . . . . . . . . . 1554


X-linked Hypophosphatemia . . . . . . . . . . . . . . . . . . . . . . . . . . . 1557
UR

Zinc Deficiency . . . . . . . . . . . . . . . . . . . . . . . . . . . . . . . . . . . . . 1558


Zollinger-Ellison Syndrome . . . . . . . . . . . . . . . . . . . . . . . . . . . . 1559
CO
LL
FU
Hospital-Based Medicine
Questions

46. You are covering a busy hospital service at night when you are paged to evaluate a 78-year-old man with sudden onset of dyspnea. A quick review of the patient’s chart

0
reveals that he was diagnosed with small cell lung cancer 2 months earlier. In spite of a regimen of radiation and chemotherapy, he was admitted to the hospital 3 days earlier

1
with a suspected pathologic fracture to the right femur. He has no other known metastases. T hirty minutes ago he became acutely short of breath. Current vital signs include

59
a heart rate of 115 beats/minute, blood pressure of 92/69, and respiratory rate of 32. Oxygen saturation is 94% on 4 L of oxygen via nasal cannula. He is anxious and
tachypneic, but lung sounds are clear and symmetric. T he heart rhythm is regular and no murmurs are appreciated. What is the best next step in the management of this
patient?

65
a. Immediately administer empiric antibiotics for coverage of hospital-acquired pneumonia.

48
b. Immediately administer therapeutic dose of intravenous heparin.
c. Arrange for synchronized electrical cardioversion.
d. Order a ventilation/perfusion (V/Q) scan of the chest.

88
e. Administer a benzodiazepine.

47. You respond to the cardiopulmonary arrest of a 72-year-old woman in the intensive care unit. She has no palpable pulse, but the cardiac monitor shows sinus tachycardia

91
at 124/minute. Breath sounds are symmetric with bag-mask positive-pressure ventilation. What is the best next step in management of this patient?

00
a. Immediate electrical cardioversion
b. Immediate transthoracic cardiac pacing
c. Immediate administration of high-volume normal saline
d. Immediate large-bore pericardiocentesis

AP
e. Immediate administration of extended-spectrum antibiotics

48. A 64-year-old man presents with acute exacerbation of chronic obstructive pulmonary disease. T he patient had a long smoking history before quitting 2 years ago. In

TS
spite of his poor baseline lung function, he has been able to maintain an independent lifestyle. T he patient is in obvious respiratory distress and appears tired. He has difficulty
greeting you secondary to shortness of breath. Respiratory rate is 32/minute. Auscultation of the lungs reveals minimal air movement. ABGs show
HA ,
, and . One dose of IV methylprednisolone has already been administered. What is the best next step in the management of this patient’s
disease?
W

a. Urgent institution of BiPAP (bilevel positive airway pressure)


b. Urgent endotracheal intubation
c. Administration of 100% FiO2 by face mask
AL

d. Start inhaled tiotropium immediately


e. IV levofloxacin
I

49. A 71-year-old woman is brought to the emergency room by her daughter because of sudden onset of right-sided weakness and slurred speech. T he patient, a recent
ER

immigrant from Southeast Asia, has not seen a doctor in two decades. Her symptoms began 75 minutes ago while she was eating breakfast. A stat noncontrast CT scan of the
head is normal. Labs are normal. Physical examination reveals an anxious appearing woman with dense hemiplegia of the R upper and lower extremities. Deep tendon
reflexes are not discernible on the R side and 2+ on the left. Aspirin has been given. What is the best next step in management of this patient?
AT

a. Immediate intravenous unfractionated heparin


b. Immediate thrombolytic therapy
M

c. Immediate administration of interferon-beta


d. Emergent MRI/MRA of head
SE

e. Emergent cardiac catheterization

50. You are asked to see a 78-year-old woman prior to surgical repair of a femoral neck fracture. Her medical problems include hypertension, osteoporosis, and
hypothyroidism. Morphine is the only medication ordered so far. She is comfortable at rest. Her BP is 136/82, HR 88, and RR 16. Her cardiac examination is normal and her
UR

lungs are clear. What is the best recommendation to prevent postoperative venous thrombosis?

a. Postoperative low-dose ASA


CO

b. Postoperative SCDs (sequential compression devices)


c. Early mobilization and ambulation
d. Postoperative subcutaneous low-molecular-weight heparin
e. Postoperative intravenous unfractionated heparin
LL

51. An 84-year-old woman develops confusion and agitation after surgery for hip fracture. Her family reports that prior to her hospitalization she functioned independently
at home, but sometimes needed help balancing her checkbook and paying bills. Her current medications include intravenous fentanyl for pain control, lorazepam for control
FU

of her agitation, and DVT prophylaxis. She has also been started on ciprofloxacin for pyuria (culture pending). In addition to frequent reorientation of the patient, which of
the following series of actions would best manage this patient’s delirium?

a. Increase lorazepam to more effective dose, repeat urinalysis.


b. Discontinue lorazepam, remove Foley catheter, add haloperidol for severe agitation, and change to nonfluoroquinolone antibiotic.
c. Continue lorazepam at current dose, discontinue fentanyl, add soft restraints.
d. Continue lorazepam at current dose, add alprazolam 0.25 mg for severe agitation, repeat urinalysis, restrain patient to prevent self harm.
e. Discontinue lorazepam, remove Foley catheter, add alprazolam 0.25 mg for severe agitation, place the patient on telemetry.

52. You are caring for a 72-year-old man admitted to the hospital with an exacerbation of congestive heart failure. Two weeks prior to admission, he was able to ambulate
two blocks before stopping because of dyspnea. He has now returned to baseline and is ready for discharge. His preadmission medications include aspirin, metoprolol, and
furosemide. Systolic blood pressure has ranged from 110 to128 mm Hg over the course of his hospitalization. Heart rate was in 120s at the time of presentation, but has been
consistently around 70/minute over the past 24 hours. An echocardiogram performed during this hospitalization revealed global hypokinesis with an ejection fraction of 30%.
Which of the following medications, when added to his preadmission regimen, would be most likely to decrease his risk of subsequent mortality?

a. Digoxin
b. Enalapril
c. Hydrochlorothiazide
d. Propranolol

0
e. Spironolactone

1
53. A 64-year-old woman presents to the emergency room with flank pain and fever. She noted dysuria over the past 3 days. Blood and urine cultures are obtained, and she is

59
started on intravenous ciprofloxacin. Six hours after admission, she becomes tachycardic and her blood pressure drops. Her intravenous fluid is NS at 100 mL/h. Her current
blood pressure is 79/43 mm Hg, heart rate is 128/minute, respiratory rate is 26/minute and temperature is 39.2°C (102.5°F). She seems drowsy yet uncomfortable. Extremities

65
are warm with trace edema. What is the best next course of action?

a. Administer IV hydrocortisone at stress dose.

48
b. Begin norepinephrine infusion and titrate to mean arterial pressure greater than 65 mm Hg.
c. Add vancomycin to her antibiotic regimen for improved gram-positive coverage.
d. Administer a bolus of normal saline.

88
e. Place a central venous line to monitor central venous oxygen saturation.

91
54. An 84-year-old woman presents to the ED with shortness of breath. She has been coughing for the past 2 to 3 days. T he patient has a history of mild dementia, but has
been able to maintain independent living at home with the assistance of her daughters and a home health agency. Her daughter denies any fever at home. Vital signs include a
heart rate of 102/minute, respiratory rate of 24/minute, blood pressure 142/58 mmHg, and temperature of 37.8°C with a weight of 52 kg. Oxygen saturation is 93% on room

00
air. Upon examination, she appears to be in mild respiratory distress. She is pleasant but oriented only to self. Chest auscultation reveals few crackles in the left upper lung
field. WBC count is 12,500, BUN is 30 mg/dL, and creatinine is 1.3 mg/dL. A chest radiograph shows an infiltrate in the left upper lung lobe. What is the best initial course of
therapy for this patient?

AP
a. Begin a third-generation cephalosporin and admit her to the hospital.
b. Begin a renal-dosed third-generation cephalosporin and a macrolide, and admit her to the hospital.

TS
c. Begin a respiratory fluoroquinolone and discharge her home for follow-up.
d. Begin a loop diuretic and monitor her oxygen saturation.
e. Begin bronchodilator therapy with an inhaled betaagonist. HA
55. A 78-year-old man presents to the emergency department with acute onset of bright red blood per rectum. Symptoms started 2 hours earlier, and he has had three bowel
movements since then with copious amounts of blood. He denies prior episodes of rectal bleeding. He notes dizziness with standing but denies abdominal pain. He has had no
vomiting or nausea. A nasogastric lavage is performed and shows no coffee ground emesis or blood. Lab evaluation reveals hemoglobin of 10.5 g/dL. What is the most likely
W

source of the bleeding?

a. Internal hemorrhoids
AL

b. Dieulafoy lesion
c. Diverticulosis
d. Mallory-Weiss tear
I
ER

e. Sessile polyp

56. You are covering the general medical service one evening when contacted by the nursing staff about a “ critical” lab test on a patient. T he patient in question is a 62-year-
old man who was admitted to the hospital with community-acquired pneumonia. His comorbidities include diabetes mellitus and chronic kidney disease. T he patient had a
AT

scheduled chemistry panel, which showed potassium of 6.5 mEq/L. You immediately order an EKG (noted below). What is the next best step in management of this patient’s
hyperkalemia?
M
SE
UR
CO
LL
FU

a. Administer IV calcium gluconate


b. Administer oral sodium polystyrene sulfonate (Kayexalate)
c. Administer subcutaneous insulin
d. Administer IV bicarbonate
Hospital-Based Medicine
Answers

46. The answe r is b. (Fauci, pp 1651-1657.) Although there are many causes of acute dyspnea in the hospitalized patient, the most likely etiology in this patient is

0
pulmonary embolism. In addition to the rapid onset of symptoms, the patient’s risk factors for development of a venous thromboembolism (malignancy, bone fracture,

1
immobility, and advanced age) are suggestive of a PE. Virchow triad predisposing to clot formation includes hypercoagulability, blood stasis, and endothelial injury. Specific

59
risk factors for venous thromboembolism include recent surgery, trauma or pregnancy, prior thromboembolic event, obesity, and hypercoagulable state. Potential etiologies
of the hypercoagulable state include prothrombin gene mutation, antiphospholipid antibody, activated protein C resistance, hyperhomocysteinemia, and deficiencies in
protein C, S, or antithrombin III. Assuming no absolute contraindication, the first-line therapy for a PE is immediate anticoagulation. Because the majority of deaths from

65
PE occur within 1 hour of onset of symptoms, it would be inappropriate to withhold treatment until confirmatory testing (CT or V/Q scan) is completed. Evaluation of a V/Q
scan (answer d) may be complicated by the likelihood that he has an abnormal chest x-ray given his history of lung cancer and thoracic radiation. In this circumstance, a CT

48
pulmonary angiogram would be the preferred test.
Although a diagnosis of pneumonia could be considered (answer a) the rapidity of onset of symptoms, the lack of purulent sputum and the clear lung fields make this
diagnosis less likely than PE. T here should be time to evaluate for pneumonia once the patient is stabilized. Answer c is incorrect because the patient likely has sinus

88
tachycardia as a result the PE; sinus tachycardia will improve with treatment of the underlying cause. Although the patient may symptomatically improve in the short term
with anxiolytic therapy (answer e), his low blood pressure may limit the use of benzodiazepines. If the patient were having an “ anxiety attack” rather than a PE, the blood

91
pressure would usually be elevated rather than depressed.

47. The answe r is c. (Fauci, pp 1707-1713.) Pulseless electrical activity (PEA) is a common cause of cardiopulmonary arrest in the hospital setting. Etiologies of PEA

00
include hypovolemia, hypoxia, hyperkalemia, severe acidosis, pulmonary embolism, cardiac tamponade, and tension pneumothorax. T he loss of cardiac output results from
decreased ventricular filling (hypovolemia, pulmonary embolism, cardiac tamponade, or tension pneumothorax) or electromechanical dissociation (hypoxia, hyperkalemia,
or severe acidosis). Management of PEA arrest requires rapid establishment of vascular access, airway stabilization, and administration of IV fluids. Physical examination

AP
focuses on potential correctable etiologies. Electrical cardioversion will not benefit a patient in sinus rhythm. Similarly, cardiac pacing will not help, since the problem is not
associated with severe bradycardia. Sudden pericardial tamponade is uncommon, but, if suspected (proper setting, jugular distension, low-voltage ECG), pericardiocentesis is
performed. Rapid saline bolus is more likely to be effective and can be given immediately. If sepsis is suspected, broad-spectrum antibiotics would be appropriate, but antibiotic

TS
administration will not affect the immediate outcome of the cardiopulmonary arrest.

48. The answe r is a. (Fauci, pp 1665-1668, 1684-1688.) Bilevel positive airway pressure (BiPAP) ventilation has found increased favor in acute lung or heart disease,
HA
especially in those with acute CO2 retention. T he use of BiPAP may prevent the need for endotracheal intubation with its concomitant risks. BiPAP is contraindicated in
patients with severe respiratory acidosis, decreased level of consciousness, bradypnea, or hemodynamic instability, for whom endotracheal intubation is the best treatment.
Although oxygen should never be withheld from a hypoxic patient, caution must be exercised in patients with chronic CO2 retention. Overly aggressive oxygen therapy may
W

actually increase PaCO2. In patients with chronic CO2 retention, a targeted oxygen saturation of 88% to 92% is appropriate. Although effective in the chronic management
of COPD, inhaled tiotropium will not help acutely. Nebulized albuterol and ipratiotropium are beneficial in COPD exacerbation but in the absence of wheezing would be less
AL

effective than BiPAP. Antibiotics are given for severe COPD exacerbations (especially if the patient is producing purulent sputum) but will not affect the immediate outcome
of his respiratory failure.
I
ER

49. The answe r is b. (Fauci, pp 2513-2531.) T his patient presents with an acute left middle cerebral artery stroke. T ime is of the essence if thrombolytic therapy is to be
beneficial. Intravenous thrombolytics may be administered up to 3 hours after the onset of symptoms. Recent studies have suggested expanding the window of opportunity to
4.5 hours. Fortunately, this patient was brought to the ER promptly. CT scan of the brain shows no evidence of bleed. Evidence of ischemia may not become apparent until
AT

48 to 72 hours. A prior history of intracranial hemorrhage, recent surgery, bleeding diathesis, onset of symptoms greater than 3 to 4.5 hours prior to therapy, and unknown
time of onset of symptoms are contraindications to thrombolytic therapy. T his patient should be given intravenous tissue-type plasminogen activator (t-PA).
Anticoagulation in acute stroke (answer a) is not currently recommended. In most trials of anticoagulation, any benefit of therapy is matched by an increase in
M

hemorrhagic transformation. Interferon-beta (answer c) is used to treat multiple sclerosis, not ischemic stroke. Emergent scanning with MRI (answer d) wastes precious time
and is not always available. Patients with acute stroke often have mild elevation in cardiac biomarkers. Cardiac catheterization (answer e) is unnecessary, and may very well
prove harmful in the setting of a stroke.
SE

50. The answe r is d. (Fauci, pp 731-735). After orthopedic injury, patients are at high risk of development of deep vein thrombosis. Other risk factors for DVT formation
include advanced age, immobility, malignancy, hypercoagulable states, and prior history of DVT. Appropriate options for DVT prophylaxis after hip fracture include
UR

subcutaneous unfractionated heparin, low-molecular-weight heparin, or fondaparinux. SCDs (answer b) may be used in addition to chemoprophylaxis, but SCDs by themselves
are not effective in hip fracture patients. Early ambulation is recommended as tolerated for all patients at risk for DVT, but is not enough to fully attenuate risk after a hip
fracture. Aspirin (answer a) is never recommended by itself for inpatient DVT prophylaxis. Intravenous heparin is used for DVT therapy, not prophylaxis.
CO

51. The answe r is b. (Fauci, pp 158-162.) Delirium is a common complication in the hospital setting. Delirium may be differentiated from dementia by its acute onset and
waxing and waning mental state. Elderly patients, especially those with a history of dementia, and the severely ill are at greatest risk of developing delirium. Delirium may be
precipitated by medications, postsurgical state, infection, or electrolyte imbalance. T he management of delirium relies on nonpharmacologic approaches, including frequent
LL

reorientation, discontinuation of any unnecessary noxious stimuli (eg, urinary catheters, unnecessary oxygen delivery systems or telemetry monitors, and restraints),
environmental modification to establish day/night sleep cycles, and discontinuation of all unnecessary medications. T his patient likely will continue to need pain control, but
FU

the dose of fentanyl should be minimized to the smallest effective dose. Benzodiazepines frequently induce a delirium and their continued use or escalation may impair
recovery. Fluoroquinolones can worsen mental status in the elderly. Physical or chemical restraints actually impair recovery from delirium and should be used only as last
resort to prevent serious harm to self or others. A repeat urinalysis would provide no useful information since the original urine culture is still pending.

52. The answe r is b. (Fauci, pp 1443-1455.) Inhibition of the renin-angiotensin-aldosterone system by either angiotensin-converting enzyme inhibitors (ACEi) or
angiotensin receptor blockers (ARB) has been proven to decrease mortality in patients with symptoms of congestive heart failure and a depressed ejection fraction. All
patients with a history of congestive heart failure should be maintained on a beta-blocker and an ACEi or ARB. Most patients will require a diuretic for symptom control.
Digitalis glyco-sides decrease rehospitalization rate but have not been shown to improve mortality. T hiazide diuretics are excellent medications for blood pressure control.
Our patient, however, has well-controlled blood pressure. T he patient is already on a selective beta-blocker and the addition of a non-selective beta-blocker is unlikely to be
helpful. Spironolactone provides mortality benefit in patients with NYHA class III or IV heart failure. T he patient in this scenario was able to walk two blocks before
stopping and would be classified as NYHA class II.
Rheumatology
Questions

61. A 40-year-old woman complains of 7 weeks of pain and swelling in both wrists and knees. She has several months of fatigue. After a period of rest, resistance to

0
movement is more striking. On examination, the meta-carpophalangeal joints and wrists are warm and tender. T here are no other joint abnormalities. T here is no alopecia,

1
photosensitivity, kidney disease, or rash. Which of the following is correct?

59
a. T he clinical picture suggests early rheumatoid arthritis, and a rheumatoid factor and anti-CCP (anti-cyclic citrullinated peptide) should be obtained.
b. T he prodrome of lethargy suggests chronic fatigue syndrome.

65
c. Lack of systemic symptoms suggests osteoarthritis.
d. X-rays of the hand are likely to show joint space narrowing and erosion.

48
e. An aggressive search for occult malignancy is indicated.

62. A 70-year-old man complains of fever and pain in his left knee. Several days previously, he suffered an abrasion of his knee while working in his garage. T he knee is red,

88
warm, and swollen. An arthrocentesis is performed, which shows 200,000 leukocytes/µL and a glucose of 20 mg/dL. No crystals are noted. Which of the following is the most
important next step?

91
a. Gram stain and culture of joint fluid
b. Urethral culture

00
c. Uric acid level
d. Antinuclear antibody
e. Antineutrophil cytoplasmic antibody

AP
63. A 60-year-old woman complains of dry mouth and a gritty sensation in her eyes. She states it is sometimes difficult to speak for more than a few minutes. T here is no
history of diabetes mellitus or neurologic disease. T he patient is on no medications. On examination, the buccal mucosa appears dry and the salivary glands are enlarged
bilaterally. Which of the following is the best next step in evaluation?

TS
a. Lip biopsy
b. Schirmer test and measurement of autoantibodies HA
c. IgG antibody to mumps virus
d. A therapeutic trial of prednisone for 1 month
e. Administration of a benzodiazepine
W

64. A 40-year-old man complains of acute onset of exquisite pain and tenderness in the left ankle. T here is no history of trauma. T he patient is taking hydrochlorothiazide
for hypertension. On examination, the ankle is very swollen and tender. T here are no other physical examination abnormalities. Which of the following is the best next step
AL

in management?

a. Begin colchicine and broad-spectrum antibiotics.


I

b. Perform arthrocentesis.
ER

c. Begin allopurinol if uric acid level is elevated.


d. Obtain ankle x-ray to rule out fracture.
e. Apply a splint or removable cast.
AT

65. A 48-year-old woman complains of joint pain and morning stiffness for 4 months. Examination reveals swelling of the wrists and MCPs as well as tenderness and joint
effusion in both knees. T he rheumatoid factor is positive, antibodies to cyclic citrullinated protein are present, and subcutaneous nodules are noted on the extensor surfaces of
M

the forearm. Which of the following statements is correct?

a. Prednisone 60 mg per day should be started.


SE

b. T he patient should be evaluated for disease-modifying antirheumatic therapy.


c. A nonsteroidal anti-inflammatory drug should be added to aspirin.
d. T he patient’s prognosis is highly favorable.
UR

e. T he patient should receive a 3-month trial of full-dose nonsteroidal anti-inflammatory agent before determining whether and/or what additional therapy is indicated.

66. A 45-year-old woman with long-standing, well-controlled rheumatoid arthritis develops severe pain and swelling in the left elbow over 2 days. She is not sexually active.
Arthrocentesis reveals cloudy fluid. Synovial fluid analysis reveals greater than 100,000 cells/mL; 98% of these are PMNs. What is the most likely organism to cause this
CO

scenario?

a. Streptococcus pneumoniae
b. Neisseria gonorrhoeae
LL

c. Escherichia coli
d. Staphylococcus aureus
FU

e. Pseudomonas aeruginosa

67. A 66-year-old man complains of a 1-year history of low back and buttock pain that worsens with walking and is relieved by sitting or bending forward. He has
hypertension and takes hydrochlorothiazide but has otherwise been healthy. T here is no history of back trauma, fever, or weight loss. On examination, the patient has a
slightly stooped posture, pain on lumbar extension, and has a slightly wide based gait. Pedal pulses are normal and there are no femoral bruits. Examination of peripheral
joints and skin is normal. What is the most likely cause for this patient’s back and buttock pain?

a. Lumbar spinal stenosis


b. Herniated nucleus pulposus
c. Atherosclerotic peripheral vascular disease
d. Facet joint arthritis
e. Prostate cancer
e. T esting for rheumatoid factor and anti-cyclic citrullinated peptide (CCP)

74. A 75-year-old man complains of headache. On one occasion he transiently lost vision in his right eye. He also complains of aching in the shoulders and neck. T here are
no focal neurologic findings. Carotid pulses are normal without bruits. Laboratory data show a mild anemia. Erythrocyte sedimentation rate (ESR) is 85. Which of the
following is the best approach to management?

a. Begin glucocorticoid therapy and arrange for temporal artery biopsy.

0
b. Schedule temporal artery biopsy and begin corticosteroids based on biopsy results and clinical course.
c. Schedule carotid angiography.

1
d. Follow ESR and consider further studies if it remains elevated.

59
e. Start aspirin and defer any invasive studies unless further symptoms develop.

75. A 53-year-old woman presents with pain in the fingers bilaterally. Examination reveals inflammation of the synovium of multiple DIP and PIP joints. Larger joints are

65
spared. Skin examination reveals the following lesion (figure). What is the most likely diagnosis?

48
88
91
00
AP
TS
HA
W
I AL
ER
AT

Re produce d, with pe rmission, from Wolff K, e t al. Fitzpatrick’s Color Atlas & Synopsis of Clinical Dermatology. 6th e d. Ne w York: McGraw-Hill, 2009. Fig. 3-3.

a. Hemochromatosis
M

b. Rheumatoid arthritis
c. Osteoarthritis
SE

d. Systemic lupus erythematosus


e. Psoriatic arthritis
UR

76. A 65-year-old man develops the onset of severe knee pain over 24 hours. T he knee is red, swollen, and tender. T he patient does not have fever or systemic symptoms.
He has a history of diabetes mellitus and cardiomyopathy. Definitive diagnosis is best made by which of the following?

a. Serum uric acid


CO

b. Serum calcium
c. Arthrocentesis and identification of positively birefringent rhomboid crystals
d. Rheumatoid factor
e. ANA
LL

77. A 35-year-old woman complains of aching all over. She sleeps poorly and all her muscles and joints hurt. Her symptoms have progressed over several years. She reports
she is desperate because pain and weakness often cause her to drop things. Physical examination shows multiple points of tenderness over the neck, shoulders, elbows, and
FU

wrists. T here is no joint swelling or deformity. A complete blood count and erythrocyte sedimentation rate are normal. Rheumatoid factor is negative. Which of the
following is the best therapeutic option in this patient?

a. Graded aerobic exercise


b. Prednisone
c. Weekly methotrexate
d. Hydroxychloroquine
e. A nonsteroidal anti-inflammatory drug

78. A 38-year-old man has pain and stiffness of his right knee. T his began 2 weeks ago after he fell while skiing. On two occasions he had the sense that his knee was locked
in a semiflexed position for a few seconds. He has noted a popping sensation when he bends his knee. On examination there is tenderness over the medial joint line of the
knee. Marked flexion and extension of the knee are painful. T he Lachman test (anterior displacement of the lower leg with the knee at 20° of flexion) and the anterior
Pulmonary Disease
Questions

96. A 50-year-old patient with long-standing chronic obstructive lung disease develops the insidious onset of aching in the distal extremities, particularly the wrists bilaterally.

0
T here is a 10-lb weight loss. T he skin over the wrists is warm and erythematous. T here is bilateral clubbing. Plain film of the forearms reveals bilateral periosteal thickening,

1
possible osteomyelitis, but no joint abnormality. Which of the following is the most appropriate management of this patient?

59
a. Start vancomycin.
b. Obtain chest x-ray.

65
c. Aspirate both wrists.
d. Begin methotrexate therapy.

48
e. Obtain erythrocyte sedimentation rate.

97. A 76-year-old woman presents with worsening dyspnea for the past 4 weeks. She has noticed fatigue, 10-lb weight loss, and occasional night sweats. On examination, she

88
is in mild respiratory distress. Her RR is 22, and her BP is 134/76. She has mild generalized lymphadenopathy, with the largest node measuring 1.5 cm. Lung examination
reveals bibasilar dullness without rales or wheezes. Her neck veins are not distended. CXR shows moderate left-sided pleural effusion. A thoracentesis is performed, revealing
milky fluid. Pleural fluid protein and LDH demonstrate an exudative effusion. T he pleural fluid cell count is 4800/mm 3 with 14% neutrophils, 12% mesothelial cells, and 74%

91
lymphocytes. Pleural fluid triglyceride is 170 mg/dL. What is the likely cause of this patient’s illness?

00
a. T uberculosis
b. Lung cancer
c. Lymphoma
d. Congestive heart failure

AP
e. Pneumonia with parapneumonic effusion

98. A 40-year-old alcoholic develops cough and fever. Chest x-ray, shown below, shows an air-fluid level in the superior segment of the right lower lobe. Which of the

TS
following is the most likely etiologic agent?
HA
W
I AL
ER
AT
M
SE
UR

Re produce d, with pe rmission, from Fauci A e t al. Harrison’s Principles of Internal Medicine, 17th e d. Ne w York, NY: McGraw-Hill; 2008.

a. Streptococcus pneumoniae
CO

b. Haemophilus influenzae
c. Legionella pneumophila
d. Anaerobes
e. Mycoplasma pneumoniae
LL

99. A 30-year-old man is admitted to the hospital after a motorcycle accident that resulted in a fracture of the right femur. T he fracture is managed with traction. T hree days
later the patient becomes confused and tachypneic. A petechial rash is noted over the chest. Lungs are clear to auscultation. Arterial blood gases show P O2 of 50, P CO2 of 28,
FU

and pH of 7.49. Which of the following is the most likely diagnosis?

a. Unilateral pulmonary edema


b. Hematoma of the chest
c. Fat embolism
d. Pulmonary embolism
e. Staphylococcus aureus pneumonia

100. A 70-year-old patient with chronic obstructive lung disease requires 2 L/minute of nasal O 2 to treat his hypoxia, which is sometimes associated with angina. T he patient
develops pleuritic chest pain, fever, and purulent sputum. While using his oxygen at an increased flow of 5 L/minute, he becomes stuporous and develops a respiratory acidosis
with CO2 retention and worsening hypoxia. What would be the most appropriate next step in the management of this patient?
4
88
CARDIOLOGY
Cardiomyopathy
-A group of diseases of the myocardium associated with mechanical or electrical dysfunction that usually exhibit ventricular hypertrophy or dilation

91
-Current major society definitions of cardiomyopathies exclude heart disease secondary to CV disorders such as HTN, CAD, or valvular disease
-Etiologies may be genetic, inflammatory, metabolic, toxic, or idiopathic
Type Info Signs & Symptoms Workup Management Prognosis

00
Dilated Cardiomyopathy: -Common etiologies: viral, -CHF -Treat CHF symptoms
dilation and impaired contraction genetic, alcoholism -Arrhythmias -ICDs
of one or both ventricles -Systolic dysfunction -Sudden death -Eval for transplant
-Exercise intolerance

AP
-Fatigue or weakness
-Dyspnea
Hypertrophic Cardiomyopathy: -Caused by genetic mutations -Varied presentation, may be asymptomatic -Differential: athlete’s -β-blockers to reduce O2 -Annual
disorganized hypertrophy of left -Diastolic dysfunction -CHF heart (physiologic LVH), demand mortality of 1%

TS
ventricle and occasionally right -Usually asymptomatic until -DOE: the most common sx HTN, aortic stenosis -CCB to reduce -May progress
ventricle childhood or adolescence -Orthopnea and PND -Valsalva will increase contractility and improve to dilated
-Athletes with underlying -Exertional chest pain HCM murmur and diastolic relaxation cardiomyopathy
HOCM at greater risk for lethal -Atypical chest pain decrease aortic stenosis -Pacer or AICD

HA
arrhythmia during exertion -Syncope and presyncope murmur -Surgical myectomy,
-May have abnormal SAM -Palpitations -EKG: prominent Q mitral valve surgery, or
movement of mitral valve -Postural hypotension waves, P wave ethanol ablation to
-Fatigue abnormalities, LAD destroy thickened

W
-Edema -Echo septum
-Arrhythmias -Holter monitor
-Harsh crescendo systolic murmur ± mitral -Exercise stress test
regurg -Screen relatives

L
-S4
-Displaced apical impulse or thrill

IA
-Sudden death
-Stroke
Restrictive Cardiomyopathy: -Etiologies: scleroderma, -R CHF as pulmonary pressures must increase to -Differential: constrictive
ER
diastolic dysfunction  normal amyloidosis, genetic, HOCM, deliver blood pericarditis
contractility but rigid and stiff DM, chemo, HIV
ventricular walls -Uncommon in US
Arrhythmogenic Right -Genetic cause -Ventricular arrhythmias
AT

Ventricular
Cardiomyopathy/Dysplasia: RV
wall replaced with fibrous tissue
M

Unclassified Cardiomyopathies -Includes stress-induced


cardiomyopathy and left
ventricular noncompaction
SE
UR

1
CO
4
88
Giant Cell Arteritis (Temporal Arteritis)
-Rheumatic disease, most often affects med-large head & neck Signs & symptoms Workup
vessels -Weight loss -Arterial biopsy

91
-Usually in white patients over 50 -Night sweats -↑ ESR or CRP
-Often co-exists with polymyalgia rheumatica -Fever
-Jaw claudication Management
-Temple tenderness -Immediate steroids while awaiting biopsy results in order to prevent blindness

00
-Vision loss with pale optic disc -Monitor for thoracic aortic aneurysm (increased risk)
-New headache
-Scalp tenderness

AP
Arrhythmias
Bradyarrhythmias
-Either a delay in impulse formation or conduction Signs & symptoms Management

TS
-Syncope or presyncope -Treat underlying cause
Etiologies -Dyspnea from CHF -Meds
-Hypoxia -Angina pectoris -Pacemaker or transcutaneous pacing
-↑ ICP -Hypotension -Emergent: atropine

HA
-Hypothermia
-Hypothyroid Differential
-Hyperkalemia -Regular rate  sinus brady, complete heart
-Sarcoidosis or amyloidosis block, 2:1 AV block, sinus arrest with escape

W
-Degenerative disease of conduction system rhythm, “regularized” slow a-fib
-Ischemia -Irregular rate  sick sinus syndrome, 2° AV
-Lyme block (Wenckebach or Mobitz type II), slow a-fib
-Rheumatic heart disease

L
-Drugs: CCBs, β-blockers, digoxin

IA
Tachyarrhythmias
-Either a result of abnormal impulse formation or Signs & symptoms Workup
abnormal propagation (reentry) -Syncope or presyncope -Labs to exclude underlying exacerbating conditions: BMP, Mg, digoxin levels, TSH
ER
-Risk factors for SVT: hyperthyroid, HTN, mitral -Palpitations
valve disease, COPD, post cardiac surgery -Diaphoresis Management
-Risk factors for VT: prior MI, ischemia, long QT -Chest pain -Cardioversion for any tachyarrhythmias if there is hemodynamic instability; add
syndrome, antiarrhythmics, TCAs, hypoMg, hypo amiodarone if VT is involved
AT

or hyperK Differential -Transcutaneous pacing for torsades


-Narrow complex afib, multifocal atrial -If stable SVT, can try vagal maneuvers, adenosine, beta blockers, CCBs, or digoxin
tachycardia, aflutter, atrial tachycardia, sinus -For stable VT, lidocaine is DOC
tachycardia, WPW, junctional tachycardia, -Atrial fibrillation or flutter: control ventricular response with CCB, beta blockers, or
M

AVNRT digoxin, and consider anticoagulation with warfarin; can electrically or chemically
-Wide complex  torsades, polymorphic VT, AF cardiovert but afib may recur
with aberrant conduction, VT, SVT -VT: beta blockers if asymptomatic and nonsustained, ICD placement if more severe due to
risk of death
SE

-Torsades: IV magnesium and phenytoin


UR

2
CO
4
88
AV Block
Type Signs & Symptoms/Info/Workup Management EKG
First degree -Asymptomatic -Treat reversible

91
-EKG showing lengthened PR interval causes such as
-Determine site of block using EKG ischemia, increased
findings, atropine, exercise, or vagal vagal tone, or meds

00
maneuvers -Pacemaker usually not
recommended

AP
Second Wenckebach -Typically asymptomatic -Treat reversible
degree (Mobitz type -EKG shows progressive PR prolongation causes such as
I) for several beats prior to nonconducted P ischemia, increased
wave vagal tone, or meds

TS
-Beats classically occur in ratios of 3:2, 4:3, -Pacemaker if there is
or 5:4 symptomatic
-Can be a result of inferior MI bradycardia

HA
Mobitz type II -May be asymptomatic or have signs of -Treat reversible
hypoperfusion or HF causes such as
-PR interval remains unchanged prior to a ischemia, increased

W
nonconducted P wave vagal tone, or meds
-Most patients will
require a pacemaker

L
IA
ER
Third degree -May have dizziness, presyncope, syncope,
v-tach, v-fib, worsening HF, or angina
AT

-P waves don’t correlate to QRS


-Escape rhythm takes over for QRS
(junctional or ventricular)
M
SE
UR

3
CO
4
88
Peripheral Vascular Disease
-Risk factors: smoker, DM, HTN, ↑ lipids, obesity Workup
-Ankle/brachial index: PVD if <0.9, will have intermittent claudication if <0.7, pain at risk if <0.4,

91
Signs & symptoms impending necrosis if <0.1
-LE disease: claudication (ppt by exercise, relieved by rest) in butt, hip, thigh, calf, or foot;
diminished peripheral pulses; femoral bruits; nighttime pain from ischemia Management
-UE disease: difference in systolic BPs between aroms, arm pain with exertion, dizziness -Smoking cessation

00
during arm exertion (subclavian steal syndrome) -Walking program
-Cool skin or abnormal skin color -Antiplatelet therapy
-Poor hair growth -Revascularization if necessary via open surgery or stent
-Ulceration or tissue necrosis -BP, lipid, sugar control

AP
-Signs of atherosclerosis elsewhere in the body
Deep Venous Thrombosis
Signs & symptoms

TS
-Palpable cord
-Calf pain
-Ipsilateral edema, warmth,
tenderness, erytema

HA
Workup
-Homan’s is only + 50% of the time
-Determine probability with Well’s
criteria  < 2 indicates unlikely, >

W
6 highly likely
-Further investigation using D-
dimer

L
-US for at least moderate Well’s
score

Management
-Immediate anticoagulation with IA
ER
heparin, LMWH, or fondaparinux
-Lytics or thrombectomy for select
cases
-3 months of anticoagulation for
AT

initial distal DVT or consider IVC


filter if not a good candidate
Aortic Stenosis (Aortic Valve Stenosis)
-Causes obstruction  ↑ LV
M

Signs & symptoms Workup


pressure  LVH  CHF -May have asymptomatic murmur early in disease: harsh systolic < > -EKG for LVH
murmur at RUSB, with radiation to the carotids bilaterally -CXR for cardiomegaly
-Echo is diagnostic
Etiologies -Late: DOE, SOB, angina, syncope, CHF, PND, orthopnea, carotid pulsus
SE

-Age-related calcification parvus et tardus Management


(inflammation + lipids, -Sudden cardiac death -Proven benefit with statins
accelerated with bicuspid -Arrhythmias -Valve replacement if symptomatic
aortic valve) -Endocarditis -Aortic balloon valvotomy as bridge to surgery or for palliation
UR

-Rheumatic fever -Bleeding predisposition


Prognosis
-High risk of sudden death without valve replacement (life expectancy of 2-3 years)
4
CO
4
88
Mitral Stenosis
-Causes elevated LA pressure  LA hypertrophy  Signs & symptoms Workup

91
transmission of high pressures to pulmonary vasculature  -Dyspnea is commonly the only symptom -Echo to stage
pulmonary edema  possible R-sided CHF -A-fib from disruption of electrical conduction in hypertrophied
tissue Management

00
Etiologies -Fatigue -If asymptomatic, only watchful waiting
-Most commonly due to rheumatic heart disease -Apex murmur: loud S1 with post S2 opening snap, followed by -HTN management
-Congenital malformation low-pitched rumble -Afib management
-Connective tissue disease -RV heave with progression to pulmonary HTN -β-blockers to prevent pulmonary edema

AP
-Pregnancy may bring on symptoms -Anticoagulation for any embolic event
-Follow with echoes every 1-5 years depending on stage
-Mitral valve replacement or balloon valvuloplasty
Hypertension

TS
-95% of cases are essential hypertension Pharmacologic treatment options: single agents only lower by 10-20 mm Hg, may need a 2nd agents
-Secondary cause workup: renal disease, renal stenosis, aortic Thiazides: HCTZ, chlorthalidone -DOC for HTN but can’t use once CrCl < 30
coarctation, hyperaldosteronism from tumor or hyperplasia, -Ca sparing = good for osteoporosis pts
Cushing’s, pheochromocytoma, OSA -Need to check BMP before and after starting

HA
-May hear S4 from forceful atrial contraction -Chlorthalidone has the most evidence but my preceptor thinks
it causes a lot of hypokalemia
JNC- 7 classifications and initial treatment of HTN Loops: furosemide
-Pre-HTN is 120/80 to 139/89  lifestyle changes K-sparing: spironolactone, eplerenone Not very potent

W
-HTN stage I is 140/90 to 159/99  thiazide (or loop if renal pt)
-HTN stage II is >160/>100  thiazide + 2nd agent ACEIs: benazepril, enalapril, lisinopril -Cough
-Can cause renal failure = need to monitor BMP 1 week and 1
Hypertensive urgency = stable or no end organ damage, no month after starting and periodically after that, STOP if serum
Cr ↑ by 30%

L
raised ICP
-May have SOB, headache, BPs usually > 220/110 -Ok to use in patients with no renal function left

IA
-Tx is to lower BP in clinic slowly over several hours -Pregnancy D
(≤160/100) with labetalol, clonidine, captopril ARBs: irbesartan, losartan, olmesartan, valsartan -Same AEs as ACEIs and also pregnancy D
CCBs: dihydropyridine (nifedipine, amlodipine) and non- -Useful in the elderly
ER
-Close follow-up
dihydropyridine (verapamil and diltiazem) -FDA warning about amlodipine, verapamil, and diltiazem use
Hypertensive emergency = rapidly progressing end organ with simvastatin
damage -Contraindicated in heart failure
-Papilledema if malignant Other direct vasodilators: hydralazine, minoxidil
AT

-Chest pain, AMS, weakness, MI, acute CHF, renal failure, α-blockers -Clonidine: only for refractory HTN due to risk of falls
ICH, eclampsia, aortic dissection -Methyldopa: DOC for HTN in pregnancy
-Don’t lower BP by more than 25% original value
β-blockers
M

-Questionable role in the treatment of essential HTN unless


Monitoring HTN: patient also has CHF or MI
-Annual urine microalbumin -Need strict β-1 blockers for asthma/COPD patients so that
-Annual BMP bronchial relaxation is not blocked (atenolol or metoprolol)
SE

-Annual lipids -Propranolol and labetalol block at multiple sites


-Baseline EKG, look for LVH -Can mask signs of hypoglycemia
-Contraindications: heart block
UR

5
CO
4
88
Depressive Disorders
-Episodes are classified as mild, moderate, or severe Screening Workup Management
-Half of cases are missed by PCPs -USPSTF grade B for nonpregnant -MMSE -Referral to psychiatry indicated for pts

91
-First episode most common in ages 30-40, with smaller adults as long as there are supports in -TSH, CMP and Ca to r/o metabolic disturbance, with severe depression, depression
peak at 50-60 place for effective treatment RPR, CBC to r/o systemic infection, folate, vit D, unresponsive to initial treatment,
-PHQ-2 or PHQ-9 vit B12, drug screen psychotic depression, depression with
-Geriatric depression scale -DSMI-IV diagnostic criteria: 5+ of the following other psychiatric diagnosis

00
Subtypes
-Melancholic depression: loss of pleasure in most symptoms present nearly every day for a minimum -Psychotherapy recommended in
activities, nonreaction to pleasurable stimuli, worsening of Differential of 2 weeks (at least 1 symptom must be depressed combination with pharmacotherapy for
sx in early morning hours, psychomotor retardation, -Dysthymia: a chronic, milder mood mood or loss of interest or pleasure) patients with severe chronic or
excessive weight loss or guilt disturbance that has been present for at • Depressed mood recurrent depression

AP
-Atypical depression: mood reactivity, weight gain, least 2 years • Loss of interest or pleasure in most or all -Exercise
excessive sleep, heavy sensation, significant social -Adjustment disorder with depressed activities -Electroconvulsive therapy is last resort
impairment mood: occurs when identifiable • Insomnia or hypersomnia -Melancholic subtype responds best to
-Catatonic depression: rare form involving disturbances of psychosocial stressor(s) that has • TCAs or MAOIs

TS
Change in appetite or weight gain
motor behavior occurred within the last 3 months can • Psychomotor retardation or agitation -Atypical subtype responds best to
-Postpartum depression: intense, sustained depression be attributed to depressed mood, and • Low energy SSRIs or SNRIs
experienced within 1 month of giving birth depressed mood resolves within 6 • Poor concentration -For refractory depression, confirm dx
-Seasonal affective disorder: resolves in spring months after the stressor has ended and medication adherence and r/o

HA
• Thoughts of worthlessness or guilt
-Major depression with psychotic features: delusions or (BUT if pt meets criteria for MDD this organic causes of depression, switch to
• Recurrent thoughts about death or
hallucinations are present diagnosis will supersede dx of another antidepressant or augment, and
suicide
adjustment disorder) consider less common regimens such as
Etiology -Minor depressive disorder: pt meets tranylcypromine or venlafaxine +

W
-Multifactorial, involving biologic, psychologic, and social criteria for 2-4 of MDD criteria mirtazapine
factors -Bipolar disorder
-Dementia

L
Pharmacologic Management
-Effectiveness of SSRIs, SNRIs, bupropion, TCAs, and MAOIs is generally comparable

IA
-Begin dose at lowest effective dose and increase incrementally until patient achieves remission
-Switch after 8 weeks of therapy if no response
-If partial response at 8 weeks, can switch or augment therapy with bupropion or buspirone
ER
-May need to use doses higher than what is FDA approved
-Duration of treatment should be continued until 4-9 months after remission of symptoms for 1st episode, an additional year after that for 2nd episode, and possibly continue treatment indefinitely for
3rd episode or more
Class & MOA Agents Info Risks & AEs
AT

SSRIs: inhibit reuptake -Fluoxetine: longest half-life, many drug -AEs: GI, CNS, sexual, sedation, fatigue, dry mouth, hypotension, withdrawal if
of serotonin as well as interactions, lowest weight gain = good for d/c abruptly, prolonged QT, rash, insomnia, asthenia, seizure, tremor,
slight effects on eating disorders, highest risk for serotonin somnolence, mania, suicidal ideation, worsened depression
histamine-R, α1-R, and syndrome
M

muscarinic-R -Citalopram and escitalopram: low risk of


sexual AEs
-Fluvoxamine
SE

-Sertraline: few drug interactions


-Paroxetine: shortest half-life, most sedating,
greatest weight gain, greatest sexual AEs,
greatest anticholinergic activity
UR

55
CO
4
88
SNRIs: inhibits -Venlafaxine: ER available -Equally effective as SSRIs for -AEs: GI, HTN, CNS, permanent sexual?, diaphoresis, dizziness, fatigue,
reuptake of both -Duloxetine: less AEs than venlafaxine treating major depression insomnia, blurred vision, suicidal ideation, dysuria, worsened depression
serotonin and -Desvenlafaxine -May be more effective in the -Fewer drug interactions

91
norepinephrine setting of diabetic neuropathy,
fibromyalgia, msk pain, stress
incontinence, sedation, fatigue, and
patients with comorbid anxiety

00
Atypical Bupropion -May increase sexual function -AEs: lower seizure threshold, insomnia, nervousness, agitation, anxiety, tremor,
Antidepressants -Has stimulant effects = good for comorbid arrhythmias, HTN, tachycardia, S-J, weight loss, GI, arthralgia or myalgia,
ADHD but don’t use if comorbid anxiety confusion, dizziness, HA, psychosis, suicidal ideation
Mirtazapine -Less nausea and sexual AEs -AEs: the most sedating antidepressant, weight gain, orthostatic hypotension,

AP
-Overdose is generally safe dizziness, dry mouth
Nefazodone
Trazodone -AEs: arrhythmia, hyper or hypotension, diaphoresis, GI, hemolytic anemia,
leukocytosis, dizziness, HA, insomnia, lethargy, memory impairment, seizure,

TS
somnolence, priapism, weight gain
Tricyclic -Amitriptyline -AEs: anticholinergic, CV, CNS, weight gain, sexual dysfunction, decreased
Antidepressants: -Clomipramine seizure threshold

HA
inhibits reuptake of both -Desipramine -Overdose can be lethal
serotonin and -Doxepin
norepinephrine -Imipramine
-Nortriptyline
MAOIs: block -Phenelzine: irreversible -MAO-A acts on norepinephrine -AEs: anticholinergic, lower seizure threshold, weight gain, rash, orthostasis,

W
destruction of -Tranylcypromine: irreversible and serotonin sexual dysfunction, insomnia or somnolence, HA, HTN crisis in presence of
monoamines centrally -Selegiline: reversible -MAO-B acts on phenylethylamine monoamines,
and peripherally and DA -Overdose is lethal

L
-2 week washout period of other antidepressants needed before starting in order
to prevent serotonin syndrome

IA
Alcohol Dependence
-Strong genetic risk factors, with heritability similar to DM or Withdrawal Management
HTN -Triggered by abrupt cessation or reduction of intake in -Inpatient management needed for possible withdrawal if there
ER
-Associated health risks include HTN, a-fib, cardiomyopathy, dependent individuals is h/o seizures, delirium, mental instability, suicidal or
esophagitis, gastritis, upper GIB, pancreatitis, hepatitis, cirrhosis -S/S: tremor, tachycardia, HTN, sweating, insomnia, nausea, homicidal ideation, psychosis, unstable environment, or no
-Related illnesses include pneumonia, TB, and cancers of the vomiting, photophobia, hallucinations, hyperreflexia, irritability, support or transportation available
breast, liver, throat, and esophagus alcohol craving, seizures -Can consider outpatient management if there are no risk factors
AT

-Onset in 12-24 hours after last drink, with peak intensity at 24- -Thiamine supplementation
48 hours
-Lasts 4-7 days
-Mortality is 10-15%
M
SE
UR

56
CO
4
88
Acute Myeloid Leukemia
-Cancer of the myeloid progenitor (gives rise to all WBCs other Signs & symptoms Workup
than B/T and NK cells), where cells do not mature and do not -From cell deficiencies: pallor, fatigue, dyspnea, -Differentiate from ALL by peripheral smear showing Auer

91
die and take up the bone marrow space of other needed cells thrombocytopenia, petechiae, hematomas, bleeding, neutropenia rods
-Most common in first 2 years at life, peaks again in with sepsis, cellulitis, pneumonia
adolescence -From hyperleukocytosis: obstruction of capillaries and small Management
arteries with high numbers of blasts -Aggressive chemo

00
-From CNS involvement: HA, AMS, CN issues
-Leukemia cutis lesions Prognosis
-DIC -Overall survival of 30%
-Tumor lysis syndrome

AP
Chronic Lymphocytic Leukemia
-Clonal proliferation and accumulation of mature-appear B cells Signs & symptoms Management
-The most commonly occurring leukemia -Fatigue, night sweats, weight loss, persistent infections, -Observation

TS
-Mostly occurs in those > 50, and more common in males lymphadenopathy, hepatomegaly, splenomegaly -Chemo with tumor lysis prophylaxis
-RAI system for staging -BMT
Workup -Radiation for lymphadenopathy
-CBC showing lymphocytosis with WBCS > 20k with

HA
concomitant anemia and peripheral smear showing mature small Prognosis
lymphocytes and cobblestone-appearing smudge cells -Typically slow-growing, but has potential for Richter’s
-Coexpression of CD19 and CD5 transformation to aggressive disease
-High IgG -Worse prognosis with deletion of chromosome 17

W
-Average 5 year survival rate of 50%
Chronic Myelogenous Leukemia
-Excess proliferation of the myeloblast or its progeny with no Signs & symptoms Management
negative feedback -Fever, bone pain, LUQ pain with splenomegaly, weakness, -Chemo

L
-Usually occurs in young to middle age adults night seats, bleeding & bruising, petechiae -BMT

Categories
1.) Chronic: < 15% blast component of bone marrow or blood
Workup

IA
-Detection of Philadelphia chromosome via FISH or RT-PCR
Prognosis
-Average survival is 6 years with treatment
ER
2.) Accelerated: peripheral blood > 15% blasts or > 30% blasts -CBC showing leukocytosis and thrombocytopenia
+ promyelocytes, or > 20% basophils
3.) Acute: when blasts comprise > 30% of BM
AT

Multiple Myeloma
-Malignancy of plasma cells where replacement of Signs & symptoms Workup Management
bone marrow leads to failure -Forms lytic lesions on bone  bone -Bone marrow biopsy shows > 5% plasma cells -Chemotherapy
M

-Etiology is unknown, but there is increased incidence pain, pathologic fx, and -Lytic lesions on metastatic bone survey x-ray series -Local radiation for pain control
with h/o pesticides, paper production, lather tanning, hypercalcemia -Spikes in M protein in protein electrophoresis (differentiate -Autologous BMT
nuclear radiation exposure, and abnormalities of -Renal failure from excretion of from MGUS, where M protein level will still be WNL) -Bisphosphonates for
SE

chromosome 13 proteins -IgG and IgA spikes on electrophoresis hypercalcemia


-Multi-hit hypothesis that development of MM -Fatigue -Peripheral smear showing rouleaux formations (poker chips)
requires 2 oncologic events: MGUS (a common, age- -Recurrent infections -Urine has Bence-Jones proteins (produced by malignant Prognosis
related medical condition characterized by -Spinal cord compression plasma cells) -Average survival with chemo is
UR

accumulation of monoclonal plasma cells in the BM -Hyperviscosity syndrome from high -Hypercalcemia 3 years, 7 years for BMT
 moderate IgG spike on electrophoresis) + 2nd hit circulating Ig of all kinds -Anemia
causing transition of MGUS to severe MM
68
CO
4
88
Lymphoma
Hodgkin Lymphoma Non-Hodgkin Lymphoma
-A group of cancers characterized by orderly spread of disease from one lymph node to another -A diverse group of blood cancers that include any kind of lymphoma except Hodgkin (includes

91
and by the development of systemic symptoms with advanced disease CLL, Waldenstrom’s, and multiple myeloma)
-Extranodal presentation in the lung, liver, or BM in some cases -Associated with congenital or acquired immunodeficiency
-Peaks in adolescence and young adulthood, and in ages 50+ -Single or multiple areas of involvement

00
-Association with EBV -Low, intermediate, and high grades
-Incidence increases with age
Signs & symptoms
-Painless, firm lymphadenopathy (often supraclavicular and cervical areas), mediastinal mass Signs & symptoms
causing cough or SOB, fever, weight loss -Lymphadenopathy  hydronephrosis, bowel obstruction, jaundice, wasting, SVC syndrome

AP
-Abdominal pain
Workup -Fever, weight loss, night sweats
-Peripheral smear showing Reed-Sternberg cells -Edema
-CT scans of chest, abdomen, and pelvis

TS
-PET scan Workup
-BM biopsy -CBC and smear are usually normal
-Lymph node biopsy -Lymph node biopsy

HA
-CT scans of chest, abdomen, and pelvis
Management
-Chemo Management
-Low dose radiation -Systemic chemo

W
Prognosis Prognosis
-Overall survival 90% but there are 3 separate risk groups -70-90% survival rate

L
INFECTIOUS DISEASE

IA
Cryptococcosis
-Cryptococcus neoformans Signs & symptoms Workup
-Invasive fungal infection that is becoming increasingly -Pulmonary infections  solitary, non-calcified nodules -Must culture organism from CSF for definitive dx of
ER
prevalent in the immunocompromised population (AIDS, -Meningoencephalitis: seen in HIV, sx occur over 1-2 weeks, meningitis but can presumptively ID with CSF Ag testing
prolonged steroids, organ transplant, malignancy, sarcoidosis)
-Associated with soil frequented by birds and with rotting Treatment
vegetation -Amphotericin B and flucytosine for meningitis
AT

-Worldwide distribution
Pulmonary Histoplasmosis
-Histoplasma capsulatum Signs & symptoms Differential
-The most prevalent endemic mycosis in the US -Symptoms begin weeks to months following exposure -Sarcoidosis
M

-Associated with bird and bat droppings, chicken coops, farm -Pneumonia with mediastinal or hilar lymphadenopathy -TB
-Malignancy
buildings, abandoning buildings, caves, wood lots -Mediastinal or hilar masses
-Most infections will be asymptomatic or self-limiting -Pulmonary nodule Workup
SE

-Cavitary lung disease -CXR looks just like sarcoid


-Pericarditis with mediastinal lymphadenopathy -Histo serologies
-Arthritis or arthralgia + erythema nodosum -Special stains on biopsy specimens
-Dysphagia from esophageal narrowing -EIA: urine, blood, or BAL
UR

-SVC syndrome
-With dissemination: fever, fatigue, weight loss, GI, CNS, Management
-Itraconazole for moderate infection, amphotericin B for severe
adrenal manifestations
69
CO
4
88
HIV
-Progresses from primary infection with seroconversion  Signs & symptoms Workup
clinical latency  early symptomatic disease  AIDs -Only lymphadenopathy during asymptomatic disease -Serologies are + 3-7 weeks after infection

91
-Transmission is mostly heterosexual in developing countries -May have mononucleosis-like syndrome during primary -Drug resistance testing
while both MSM and heterosexual in the US infection -Definition of AIDS is when CD4 count drops to < 200
-Patients are most infectious during primary infection -Febrile illness
-Aseptic meningitis

00
Management
-Large debate about aggressive treatment of primary infection
vs waiting until disease is symptomatic
Lyme Disease

AP
-Borrelia burgdorferi with a tick vector Workup
-Transmission usually does not occur until 72 hours after attachment -Dx can be clinical if erythema migrans is present (serologies will be neg)
-Serologies will be + during early disseminated disease
Signs & symptoms

TS
-Early localized disease: erythema migrans ~1 mo after exposure, nonspecific viral syndrome Management
-Disseminated disease: acute neurologic or cardiac involvement weeks to months after tick bite -Treat with doxycycline, amoxicillin, or cefuroxime, for 10-21 days for erythema migrans, 14-21
(AV blocks days for facial nerve palsy, 28 days for meningitis or arthritis
-Late disease: years after disease; arthritis, subtle encephalopathy or polyneuropathy -IV antibiotics needed for patients with cardiac symptoms or late neurologic disease

HA
-HA, fatigue, arthralgias may persist for months after treatment but don’t represent active -No evidence for extended-course antibiotics for presumed chronic Lyme
disease -Can give single dose doxycycline for prophylaxis if attached tick is identified, estimated to be
present > 36 hours, local tick Borrelia infection rate > 20%
Cholera

W
-Vibrio cholerae Signs & symptoms Workup
-US cases are only acquired overseas or via consumption of -Severe, watery diarrhea -Stool Gram stain for curved Gram neg rods
contaminated seafood -Vomiting -PCR for toxinogenic strains

L
Prevention Management
-Dukoral vaccine available -Begin treatment before definitive diagnosis!

IA -Rehydration
-Antibiotics: doxycycline, FQ if resistant
ER
Mycobacterial Disease
-Mycobacterium tuberculosis Signs & symptoms Workup
-Mycobacterium leprae -MAC: pulmonary disease with cough, fatigue, malaise, -Sputum or BAL culture
-Non-tuberculous mycobacteria: MAC, Mycobacterium weakness, dyspnea, chest discomfort, occasional hemoptysis
AT

kansasii, Mycobacterium abscessus -M. kansasii presents as lung disease that is very similar to TB Management
-Superficial lymphadenitis -3 drug regimen for 12 months+
-Disseminated disease in the immunocompromised
-Skin and soft tissue infection from direct inoculation
M

Amebiasis
-Entamoeba histolytica Workup
-Serology or Ag testing along with parasitic stool exam
SE

Signs and symptoms


-Intestinal amebiasis has a subacute onset of 1-3 weeks with mild diarrhea or dysentery, Management
abdominal pain, bloody stools, can have fulminant colitis with bowel necrosis  perf and -Treat with metronidazole, then paromomycin to kill the cysts
peritonitis or toxic megacolon
UR

-Extraintestinal manifestations present as liver abscesses or pulmonary, cardiac, or brain


involvement

70
CO
4
88
Toxoplasmosis
-Toxoplasma gondii Signs & symptoms Workup
-Acquired via ingestion of contaminated meat or cat poop, -Majority of adult infections are asymptomatic -Serology (IgM will be + within 1 week and may persist for

91
through vertical transmission, or via blood transfusion or organ -Fevers, chills, sweats years, IgG persists for lifetime)
transplantation -Cervical lymphadenopathy -Brain MRI for HIV patients showing ring-enhancing lesions
-High amount of seroprevalence in US as 30-40% of household -Can have reactivation illness during times of
cats are infected immunosuppression

00
Management
-In HIV can have encephalitis -Pyrimethamine + sulfadiazine for cerebral toxoplasmosis
Prevention -With vertical transmission there is congenital toxo syndrome
-AIDS pts should be given prophylaxis
Syphilis

AP
-Treponema pallidum Signs & symptoms Workup
-Most cases are MSM -Primary/acute infection lasts 5-6 weeks: contagious chancre, -Remember that negative tests do not exclude a diagnosis of
painless rubbery regional lymphadenopathy, followed by syphilis

TS
generalized lymphadenopathy -Darkfield microscopy of chancre sample
-Secondary infection 6 weeks-6 months after exposure (not all -LP for neurosyphilis
pts will develop this): fever, malaise, HA, arthralgias, bilateral -Direct fluorescent antibody testing
papulosquamous rash on the palms and soles, alopecia, denuded -Serology: RPR (has a 3-6 week latency period)

HA
tongue, condyloma lata -HIV test recommended as syphilis facilitates this infection
-Tertiary infection occurs in disease > 4 years’ duration: end
organ manifestations, CV symptoms, gummas, neurosyphilis Management
-Latent infection has no clinical manifestations but serology will -Mandatory reporting within 24 hours

W
be reactive -Penicillin G
-Recheck serologies at 6 and 12 months after treatment to look
for fourfold reduction in titer
Cytomegalovirus

L
-Transmission may be sexual, close contact, or blood and tissue Signs & symptoms Workup
exposure -Generally asymptomatic or nonspecific in immunocompetent -CBC shows lymphocytosis
-HIV patients and transplant patients are at increased risk of
reactivation disease
host

IA
-Can have CMV mononucleosis with fever (distinguish from
-PCR test
-Serologies
ER
EBV by absence of lymphadenopathy and pharyngitis) -Viral culture
-Rare associations with colitis, encephalitis, myocarditis
-Can have reactivation in critically ill patients Management
-Antivirals only for immunocompromised with severe
manifestations
AT
M
SE
UR

71
CO
Questions: 14–24 5

syncopal event 1 month ago. Her echocardio- (C) it is generally performed with
gram reveals a cardiac tumor in the left atrium, cardiopulmonary bypass

0
it is pendunculated and attached to the endo- (D) it may cause renal failure

91
cardium. Which of the following is the most (E) it requires carotid artery puncture
likely cause of this lesion?

55
(A) myxoma 22. A 23-year-old man develops sharp left-sided
chest pain, fever, and a friction rub heard at

86
(B) sarcoma
the lower left sternal border, unaffected by res-
(C) rhabdomyoma
piration. The pain is also aggevated by lying

4
(D) fibroma down and relieved by sitting up. He is other-

88
(E) lipoma wise well with no other symptoms and the

91
remaining physical examination is normal.
19. A 72-year-old woman has new-onset atrial flut- Which of the following is the most likely cause
ter with a ventricular rate of 150/min. She is

00
for his symptoms?
hemodynamically stable with a blood pressure
of 155/90 mm Hg, but is experiencing palpita- (A) rheumatic fever

P
tions. Which of the following drugs is the best (B) tuberculosis (TB)

SA
intravenous choice for controlling the heart rate? (C) herpes simplex virus
(A) diltiazem (D) MI
(B) lidocaine T (E) coxsackievirus
HA
(C) aminophylline
23. A 72-year-old woman with angina undergoes
(D) magnesium cardiac catheterization. The pulmonary capil-
W

(E) atropine lary “wedge” pressure is an approximation of


the pressure in which of the following structures?
AL

20. Several of the older patients in your practice


intend to pursue exercise programs. They have (A) pulmonary artery (PA)
RI

no cardiac symptoms, but some do have vas- (B) pulmonary vein


cular risk factors such as diabetes or hyperten- (C) left atrium
E

sion. In these patients, which of the following (D) right atrium


AT

is true about exercise electrocardiography? (E) vena cava


(A) it is an invasive procedure
M

(B) it is contraindicated in patients over 24. A 58-year-old man with hypertension is


65 years of age brought to the emergency room after sudden-
SE

onset chest pain that radiates to his back and


(C) it detects latent disease
arms. He is in moderate distress with a blood
(D) it has a morbidity of approximately 5%
UR

pressure of 160/90 mm Hg in the left arm and


(E) it is used in pulmonary embolism 120/70 mm Hg in the right arm. Cardiac exam-
ination reveals a soft second heart sound and a
CO

21. A 58-year-old man is undergoing cardiac murmur of aortic insufficiency. His ECG shows
catheterization for evaluation of chest pain sinus tachycardia but no acute ischemic changes,
symptoms. He is worried about the risks, and and the chest x-ray (CXR) is shown in Fig. 1–4.
LL

as part of obtaining informed consent, you Which of the following is the most appropriate
advise him about the risks and benefits of the next step in confirming the diagnosis?
FU

procedure. Which of the following aspects of


angiography is true? (A) coronary angiography
(B) transthoracic echocardiography
(A) it is contraindicated in the presence of
(C) computerized tomography (CT) chest
cyanosis
(D) exercise stress test
(B) it is considered noninvasive
(E) cardiac troponin level
Questions: 6–14 37

12. A 27-year-old man develops warts on his hand.


Which of the following is a correct statement

0
concerning these skin lesions?

91
(A) are viral in etiology

55
(B) may be premalignant lesions
(C) are found mainly in patients with

86
lymphoma
(D) are contagious in children only

4
(E) may be treated with griseofulvin

88
13. A 27-year-old man develops a painless 1 cm

91
sore on his penis. It appears ulcerated with a
raised margin and minimal serous exudates.

00
Which of the following is the most appropriate
next step in the diagnosis? (See Fig. 2–4.)

P
(A) biopsy

SA
(B) Gram stain
(C) serology
T (D) ultrasound
HA
Figure 2–3. (E) skin test
(Reproduced, with permission, from Wolff K and
W

Johnson RA, Fitzpatrick’s Color Atlas & Synopsis


of Clinical Dermatology, 5th ed. New York: 14. A 64-year-old woman notices bullous-type
McGraw-Hill, 2005:87.) lesions over her thighs and axilla. They are
AL

itchy, but not painful, and she has no other


10. A 22-year-old woman develops an acute con- symptoms. On examination, there are large
RI

tact dermatitis to a household-cleaning agent. tense, serous-filled bullae on the affected areas.
Which of the following treatments is most A biopsy confirms the diagnosis of bullous
E

appropriate during the bullous, oozing stage? pemphigoid. Which of the following histologic
AT

features is typical of this condition?


(A) wet dressings
(A) nonspecific changes
M

(B) systemic corticosteroids


(B) immunoglobulin A (IgA) deposits
(C) topical anesthetics
SE

(C) lesions within the epidermis


(D) systemic antibiotics
(acantholysis)
(E) antihistamines
(D) basement membrane lesions
UR

11. Which of the following is a characteristic of (E) immunoglobulin M (IgM) deposits.


ringworm of the scalp as compared with other
CO

dermatophytoses?
(A) more frequent occurrence in childhood
LL

(B) high degree of contagiousness


(C) ability to invade the dermis
FU

(D) sensitivity to penicillin


(E) ability to spread to other organs
Answers: 1–17 93

9. (F, J) Whipple’s disease is caused by infection esophagus, most are adenocarcinomas. The
with a Gram-positive bacillus called Tropheryma adenocarcinomas develop more commonly

0
whippelii. The disease, previously invariably from columnar epithelium in the distal esoph-

91
fatal, can be controlled with long-term antibi- agus (Barrett’s esophagus). Adenocarcinomas
otic therapy (at least 1 year), and some patients of the esophagus have the biologic behavior of

55
seem to be cured. Arthritis and central nervous gastric cancers. The incidence of squamous cell
system (CNS) involvement are other manifes- cancer of the esophagus is decreasing while

86
tations of this rare disease. (Kasper, pp. 1774–1775) adenocarcinoma is increasing. Currently, over
50% of esophageal cancer is adenocarcinoma.

4
10. (B) Zollinger-Ellison syndrome is caused by a The 5-year survival for esophageal cancer is

88
nonbeta islet cell tumor of the pancreas. It may <5%. Combination therapy seems to be more
be associated with the syndrome of multiple effective than surgery alone. (Kasper, pp. 523–524)

91
endocrine neoplasia, type I (MEN I). The syn-
drome should be suspected in patients with 14. (D) Fat malabsorption demonstrated on stool

00
multiple ulcers, ulcers resistant to therapy, collection for 72 hours is the gold standard, but
ulcers in unusual locations, strong family his- does not indicate the exact cause. The Schilling

P
tory of ulcers, unexplained diarrhea, or evi- test is useful in testing for vitamin B12 absorp-

SA
dence of MEN I. (Kasper, p. 1758) tion. X-rays can be helpful in diagnosing
underlying disorders, but are nonspecific.
11. (D) Diverticula are present in over 50% of octo- Small intestinal biopsy is useful in determining
genarians. Most patients remain asymptomatic. T the cause of malabsorption. (Kasper, p. 1768)
HA
They are most common in the sigmoid colon
and decrease in frequency in the proximal 15. (A) The antibody can be demonstrated in
W

colon. The relative scarcity of diverticula in 80–90% of patients, usually late in convales-
underdeveloped nations has led to the hypoth- cence, and indicates relative or absolute immu-
AL

esis that low fiber diets result in decreased fecal nity. In contrast, HBsAg occurs very early and
bulk, narrowing of the colon, and an increased disappears in <6 months. Persistence of HBsAg
RI

intraluminal pressure to move the small fecal indicates chronic infection. The pattern in this
mass. This results in thickening of the muscu- patient is also seen postvaccination, and per-
E

lar coat, and eventually herniations or diver- haps as a consequence of remote infection.
AT

ticula of the mucosa at the points where (Kasper, p. 1834)


nutrient arteries penetrate the muscularis.
16. (D) Gilbert’s syndrome may be associated with
M

(Kasper, p. 1798)
impaired hepatic uptake of bilirubin. It is
SE

12. (C) Amylase accumulates in the setting of renal caused by hereditary decrease in the activity
failure and thus becomes a less-valuable diag- of glucoronosyltransferase in the uridine
nostic test. Numerous other conditions involv- diphosphate glycosyltransferase 1 (UGT1)
UR

ing the pancreas, the gut, and the salivary family. More severe enzyme deficits are the
glands can raise amylase levels. Sulfonamides cause of the two variants of Crigler-Najjar syn-
CO

cause pancreatitis; therefore, an elevated amy- drome. (Kasper, pp. 1819–1820)


lase is not confusing, but rather a useful test
for pancreatitis in patients taking the drug. 17. (A) Intestinal polyposis is a possible indication
LL

Morphine can elevate amylase levels in the of Peutz-Jeghers syndrome associated with
absence of pancreatitis. (Kasper, p. 1893, 1897) dark brown spots on the lips and palate. There
FU

is characteristic distribution of pigment around


13. (D) This presentation is typical of esophageal lips, nose, eyes, and hands. Tumors of the ovary,
cancer. Lesions in the upper two-thirds of the breast, pancreas, and endometrium are associ-
esophagus are squamous, but in the distal ated with this syndrome. (Kasper, pp. 198, 553)
Answers: 29–49 195

ESR cannot make the diagnosis, a normal ESR and this condition may create difficulties at
helps in excluding the diagnosis. C-ANCA is a operation. Habitual dislocation of joints is also

0
diagnostic tool for Wegener’s granulomatosis. a characteristic of this syndrome. (Kasper, p. 2328)

91
Elevated CPK is not seen in temporal arteritis,
even with associated PMR. Normochromic, or 46. (C) In traumatic arthritis, swellings, ecchymoses,

55
slightly hypochromic, anemia often seen in muscular spasms, and tenderness tend to be
temporal arteritis is too nonspecific to be of present, but fractures must be excluded. This

86
much diagnostic help. (Kasper, p. 2009) man has calcium pyrophosphate (CPPD) crystal-
induced monoarthritis, so called “pseudogout.”

4
41. (C) About 20–30% of patients with PAN have It is most common in the elderly and can be

88
hepatitis B antigenemia. Circulating immune precipitated by minor trauma. The crystals have
complexes containing hepatitis B antigen and a rhomboid shape, and the clinical presenta-

91
immunoglobulin have been detected, and tion can mimic that of gout. It can be associated
immunofluorescence of blood vessel walls with metabolic abnormalities such as hyper-

00
have also demonstrated hepatitis B antigen. parathyroidism or hemochromatosis. Treatment
Antiviral therapy has been used in these cases. is with an NSAID for 7–10 days. If there are

P
(Kasper, p. 2007) multiple joints involved, then steroids can be

SA
considered. An alternative to oral NSAIDs is
42. (C) This patient has a sacroiliac form of arthritis intra-articular steroids for single joint disease.
seen in patients with psoriasis. Patients with Allopurinol is not effective in CPPD. Synovial
psoriasis can develop five different patterns of T fluid is bloody, but the fluid is of normal vis-
HA
musculoskeletal symptoms. These include cosity, so a “string” test is usually positive.
arthritis of the DIP joints; asymmetric oligoarthri- (Kasper, pp. 2047–2048)
W

tis; symmetric polyarthritis similar to RA; spine


and sacroiliac type; and arthritis mutilans, a 47. (A) The course of muscle necrosis in dermato-
AL

highly destructive form. Most patients with myositis can be best followed by repeated CK
psoriatic arthritis also have nail involvement. determinations. Repeated muscle biopsies are
RI

Only about a quarter actually develop a pro- rarely required. However, the goal of therapy is
gressive, destructive disease. Uric acid may be to increase muscle strength and function, so fol-
E

elevated because of high tissue turnover but is lowing muscle strength is the key clinical assess-
AT

not part of the pathogenesis of joint disease. ment of response to therapy. (Kasper, p. 2544)
(Kasper, p. 1998)
48. (B) This patient with SLE has developed autoim-
M

43. (A) The presence of RF has little predictive mune hemolytic anemia. Treatment consists of
SE

power in determining the diagnosis of RA. high-dose oral steroids. The positive Coombs’
However, it is useful in determining prognosis, test (direct antiglobulin test), high indirect biliru-
as high titers of RF are associated with more bin, high LDH, and high retic count are all sup-
UR

severe and progressive disease, as well as with portive of hemolytic anemia. A normal AST and
extra-articular manifestations. (Kasper, p. 1972) ALT rule out hepatitis. (Kasper, p. 1964)
CO

44. (D) The disease is relapsing polychondritis and 49. (E) This story is typical of Lyme disease. The
is characterized by frequent remissions and spirochete involved (B. burgdorferi) is transmit-
LL

exacerbations of lesions and is rarely fatal. ted by ixodic ticks and is most common in the
Auricular chondritis and nasal chondritis are Northeastern and Midwestern parts of the
FU

the most common manifestations. It can also be United States. The host animal varies depend-
secondary to SLE, RA, Sjögren’s syndrome, and ing on the exact type of tick. Treatment is
vasculitis. (Kasper, pp. 2015–2016) briefer and more effective earlier in the course
of the disease. The treatment of choice is doxy-
45. (E) In Ehlers-Danlos syndrome, skin hyperex- cycline 100 mg bid for 1–2 months. Amoxicillin
tensibility, fragility, and bruisability are marked, is a second choice drug. (Kasper, pp. 997–998)
290 14: Comprehensive Review

98. A 94-year-old female nursing home resident is


referred for evaluation of anemia of 8 g/dL.

0
She has dementia, and adequate documenta-

91
tion of her past medical history is not avail-
able. She eats well and is cooperative.

55
Examination reveals evidence of cognitive
impairment, primitive reflexes, and a well-

86
healed midline abdominal scar. Her blood film
is shown in Fig. 14–8. You presume a relation-

4
ship between the anemia and the previous sur-

88
gery. Which of the following surgical
procedures is most likely to cause her current

91
anemia?

00
(A) gastrectomy
(B) vagotomy and pyloroplasty

P
(C) cholecystectomy

SA
(D) right hemicolectomy
(E) common bile duct exploration

T99. A 42-year-old man suffers a myocardial infarc-


HA
tion during coronary angiography. Which of
the following is the approximate risk of this
W

complication?
(A) 1 in 10,000
AL

Figure 14–6. (B) 1 in 100


RI

(C) 1 in 100,000
(D) 1 in 1000
E

(E) unknown
AT
M
SE
UR
CO
LL
FU

Figure 14–7.

You might also like